You are on page 1of 100

Part 1 : 07/07/14 10:28:01

Question 1 - CMA 1294 H4 - Budget Methodologies

A. A continuous budget represents the same firm commitment that other types of budgets represent.

B. A continuous budget can be prepared for different levels of activities. The budget that is prepared for only one level
of activity is a static budget.

C. A continuous budget can be prepared for only one level of activity. The budget that prepared for different levels of
activity is called a flexible budget.

D. A continuous budget, also called a rolling budget, is one that is prepared for a certain period of time ahead
of the present. For example, a 1-year continuous budget will be prepared at the end of every month for the
next 12 months.

Question 2 - ICMA 10.P1.062 - Budget Methodologies

A.

This is not the correct answer. Please see the correct answer for an explanation.

We have been unable to determine how to calculate this incorrect answer choice. If you have calculated it, please let
us know how you did it so we can create a full explanation of why this answer choice is incorrect. Please send us an
email at support@hockinternational.com. Include the full Question ID number and the actual incorrect answer choice
-- not its letter, because that can change with every study session created. The Question ID number appears in the
upper right corner of the ExamSuccess screen. Thank you in advance for helping us to make your HOCK study
materials better.

B.

The question tells us that beginning finished goods inventory of dolls on hand is 1,2000 dolls and the target
ending units of finished dolls in inventory is 15,000 dolls. The company plans to sell 200,000 dolls. The
inventory formula is:

Beginning Inventory + Units Produced/Purchased − Units Sold/Used = Ending Inventory

Letting P stand for Units Produced, we have the following formula:

Beginning Inventory of 12,000 finished dolls + P − 200,000 dolls sold = Ending Inventory of 15,000 dolls.

Solving for P, we get P = 203,000 dolls.

Since each doll produced requires 2 shoes, the total shoes required for production of 203,000 dolls = 203,000
× 2 = 406,000 shoes.

We now use the shoes needed for current production along with the beginning and ending inventories of
shoes to calculate the number of shoes that will need to be purchased. We use the same inventory formula:
Beginning Inventory + Units Produced/Purchased − Units Sold/Used = Ending Inventory.

Letting P stand for Purchases of shoes, we have the following formula:

Beginning Inventory of 20,000 shoes + P − 406,000 shoes Used in Production = Ending Inventory of 18,000
shoes.

Solving for P, we get P = 404,000 shoes.

(c) HOCK international, page 71


Part 1 : 07/07/14 10:28:01

C. This answer results from using the number of dolls to be sold instead of the number to be produced in calculating
the number of shoes needed.

D. This answer results from reversing the beginning and ending inventories when calculating both the number of dolls
to be produced and the number of shoes to be purchased.

Question 3 - CMA 692 3.11 - Planning and Budgeting Concepts

A. A budget is a realistic plan for the future expressed in quantitative terms. A budget is a very useful tool and
can serve as a tool in a number of areas: planning, control, evaluation, motivation, communication, identifying
future problems. Developing a budget is an important step in the process of control. However, a formal
budgetary process alone will not ensure improved cost control or prevent inefficiencies. After the budget has
been developed, it must be used as a tool by management to accomplish those objectives.

B. A budget provides criteria against which to measure performance during the upcoming period. Therefore, it does
provide a formal benchmark to be used for feedback and performance evaluation, and that is an advantage of
budgeting.

C. One of the advantages of a budget is that it forces management to evaluate the reasonableness of assumptions
used in the budgetary process and goals that have been set.

D. The budget communicates and authorizes actions and also promotes coordination within an organization. This is
an advantage of a budget.

Question 4 - CMA 691 H1 - Planning and Budgeting Concepts

A. Because the company already has a budget, there should already be a budget director.

B. Because the company already has a budget, there should already be a budget committee.

C. In order to use the budget as both a planning and control tool, the budgeting and accounting systems need
to be synchronized. The responsibility centers used for budgeting need to be the same as the responsibility
centers used for accounting; the chart of accounts used for budgeting need to be the same as the chart of
accounts used for accounting; and so forth. This enables management to compare the budget with the actual
levels of activity, revenues and expenditures and calculate variances.

D. The existing budget process should have forecasting procedures already in place.

Question 5 - ICMA 10.P1.026 - Learning Curves

A.

This is the number of direct labor hours required for the first unit multiplied by 7 and the product multiplied by .80. This
is not the correct way to find the number of direct labor hours required for the additional seven units.

Find the total number of direct labor hours required for all eight units. The number of direct labor hours required for the
additional seven units (units 2 through 8) will be that number minus the number of direct labor hours required for the
first unit.

(c) HOCK international, page 72


Part 1 : 07/07/14 10:28:01

B. This is the number of direct labor hours required for the first unit multiplied by 7. This would be correct only if no
learning had taken place. Since learning did take place, the number of direct labor hours required for the additional
seven units must be less than the number of hours for the first unit multiplied by 7.

C. This is the total number of direct labor hours required for all eight units. The number of direct labor hours required
for the additional seven units (units 2 through 8) will be this number minus the number of direct labor hours required
for the first unit.

D.

To find the number of direct labor hours required for the additional seven units, first find the total direct labor
hours required for all eight units. Then, to find the number of hours required for the seven additional units,
subtract the 10,000 hours required for the first unit from the total number of hours required for all eight units.

The learning curve rate is given as 80%. Therefore, the formula to calculate the total direct labor hours
required for 8 units (3 doublings) is: 10,000 (2 × .8) (2 × .8) (2 × .8), or 10,000 (2 × .8)3, which is equal to 40,960.
The first unit required 10,000 direct labor hours, so we subtract the 10,000 hours required for the first unit
from the 40,960 hours required for all 8 units. The result, 30,960 hurs, is the number of hours required for
units 2-8, the seven additional units.

Question 6 - HOCK CMA.P1A5.07 - Top-Level Planning and Analysis

A. These will cause the company's requirements for external financing to increase, but they are not the only events
from among those listed that will have that effect.

B. Increasing the retention ratio will cause the company's requirements for external financing to decrease, not
increase.

C. Negotiating a lower price and longer terms with a major supplier and increasing its retention ratio will both cause
the company's requirements for external financing to decrease, not increase.

D. These events will all cause the company's requirements for external financing to increase. When the
dividend payout ratio increases, it means the company is paying out more of its net income in dividends and
so it will have less retained earnings and cash available. When the company changes its credit terms to
increase the time it gives customers to pay, this will cause accounts receivable to increase and collections
and cash to decrease. Lower prices will decrease the profit margin and will decrease collections from sales,
which will result in less cash available.

Question 7 - ICMA 10.P1.079 - Budget Methodologies

A. This is the amount of the receivables at December 31. That is not the same as the amount that will be collected
during January, because (1) not all of those receivables will be collected during January; and (2) some of the
collections during January will come from sales made in January.

B.

Collections during January will include the following:


The $162,000 outstanding at December 31 from December sales represents 90% of December sales,
since 10% of sales are collected during the month of sale. Therefore, December sales were $162,000 ÷
.90, or $180,000. 60% of that, or $108,000, will be collected during January.

(c) HOCK international, page 73


Part 1 : 07/07/14 10:28:01

The $49,500 outstanding at December 31 from November sales will all be collected during January,
since January is the second month after the sale.
10% of January sales of $170,000, or $17,000, will be collected during January.

The total collected during January will be $108,000 from December sales plus $49,500 from November sales
plus $17,000 from January sales, for a total of $174,500.

C. This is 60% of the $162,000 in receivables remaining from December plus 30% of the $49,500 in receivables
remaining from November plus 10% of January sales of $170,000. The amount collected during January from
December sales will be 60% of December sales, not 60% of the receivables remaining from December sales. And the
amount collected during January from November sales will be 30% of November sales, not 30% of the receivables
remaining from November sales. Using the informaiton given in the problem, calculate what November sales were and
what December sales were.

D. This is the $211,500 of receivables outstanding at December 31 plus 10% of the sales made during January. The
amount of receivables outstanding at December 31 will not all be collected during January.

Question 8 - CMA 1296 H5 - Budget Methodologies

A. Management by exception lets managers concentrate their attention on areas where problems are. This will likely
be a motivating item for managers as they will feel that their efforts are being directed where they are needed.

B. Participation in the budgetary process would motivate managers by giving them a sense of ownership of the budget.

C. A budget is a very useful tool in a number of areas: planning, control, evaluation, motivation,
communication, identifying future problems. However, when top management sets the budget without any
participation from those who will be responsible for meeting the budgeted goals, employees will not have a
sense of ownership of the plan. This will not be motivating and thus, not very effective.

D. Holding subordinates accountable for the items they control would most likely motivate managers and encourage a
better performance.

Question 9 - CMA 1294 H6 - Planning and Budgeting Concepts

A. An annual profit plan should be based on a combination of financial, quantitative, qualitative measures. This answer
includes only qualitative measures.

B. An annual profit plan should be based on a combination of financial, quantitative, qualitative measures. This answer
includes only financial measures.

C. An annual profit plan should be based on a combination of financial, quantitative, qualitative measures. This
answer includes only quantitative measures.

D. An annual profit plan should be based on a combination of financial, quantitative and qualitative measures.
The development of goals and objectives is the first step in the planning process. Top management must
establish the major goals and objectives, set priorities and communicate these priorities to the people within
the organization. Lower levels of the organization bear a part of the responsibility for the overall
organizational goals and objectives, which become subunits' goals and objectives. However, specific
departments' goals and objectives may conflict with other departments' goals and objectives. For example, a
goal of an increase in market size may conflict with a goal of profitability of sales. Thus, profit plans should
be based on multiple measures.

(c) HOCK international, page 74


Part 1 : 07/07/14 10:28:01

Question 10 - CMA 683 5.8 - Probability

A. Since 50% of time a recall of a product is required, 50 of the numbers 1-100 need to be assigned to that alternative.
The number of numbers in an interval is calculated as the highest number minus the lowest number plus 1. The
numbers 40 - 90 represent 51 random numbers, not 50.

B. Since 50% of time a recall of a product is required, 50 of the numbers 1-100 need to be assigned to that alternative.
The number of numbers in an interval is calculated as the highest number minus the lowest number plus 1. The
numbers 1 - 40 represent 40 random numbers, not 50.

C. Since 50% of time a recall of a product is required, 50 of the numbers 1-100 need to be assigned to that
alternative. The number of numbers in an interval is calculated as the highest number minus the lowest
number plus 1. Numbers 11 - 60 represent 50 random numbers, and this is the only answer choice from
among the given possibilities that contains 50 numbers in the interval.

D. Since 50% of time a recall of a product is required, 50 of the numbers 1-100 need to be assigned to that alternative.
The number of numbers in an interval is calculated as the highest number minus the lowest number plus 1. The
numbers 61 - 100 represent 40 random numbers, not 50.

Question 11 - CMA 690 5.17 - Risk, Uncertainty and Expected Value

A. The expected value of 50 bags of peanuts is $29.60, which is not the optimal expected value.

B. The expected value of 30 bags of peanuts is $27.60, which is not the optimal expected value.

C.

Expected value is the criterion for selecting the best course of action. The highest possible expected value is
$30.40, and it is related to stocking 40 bags of peanuts.

D. The expected value of 20 bags of peanuts is $20.00, which is not the optimal expected value.

Question 12 - CMA 692 3.13 - Planning and Budgeting Concepts

A. A manager's performance valuation should be based on the factors controllable by the manager. Gross profit is
equal to sales revenue minus COGS. Cost of goods sold includes part of the fixed manufacturing overheads.
However, fixed manufacturing overheads usually include items that are not controlled by a division manager, such as
depreciation. Thus, gross profit should not be used as a division manager performance evaluation tool.

B. A manager's performance valuation should be based on the factors controllable by the manager. Contribution
margin does not include fixed costs and some of the fixed costs may be controllable by the division manager.

C. A manager's performance valuation should be based on the factors controllable by the manager. A
contribution income statement that presents net revenue minus controllable division costs can be used to
isolate the controllable costs of a business unit from its non-controllable costs such as depreciation or
allocated central costs. According to the contribution income statement approach to evaluation, a division
manager usually controls the division's revenues, variable costs and a portion of its fixed costs.

(c) HOCK international, page 75


Part 1 : 07/07/14 10:28:01

D. A manager's performance valuation should be based on the factors controllable by the manager. Net income minus
the division's fixed costs would (a) include a reduction for the fixed costs twice, since fixed costs are a reduction to net
income; and (b) only some of the fixed costs, not all, may be controllable by the division manager.

Question 13 - CIA 1190 IV.15 - Budget Methodologies

A. This is the amount of ending inventory of materials.

B. This is the amount of materials requirements for the period's production.

C. This is not taking into account that only 22,000 finished units are needed to be produced, not 24,000 units.

D.

First, we need to determine the production requirements for the given period. The formula is:

Units in Beginning Inventory + Units Manufactured – Units Sold = Units in Ending Inventory.

Let X stand for Units Manufactured:

14,000 + X - 24,000 = 12,000


X = 22,000

Next, we take those 22,000 units to be produced and use it to calculate the number of pounds of direct
materials that will be needed for production. 4 units (pounds) of direct materials are needed for each unit of
finished goods produced. Thus, we will need 22,000 × 4, or 88,000 pounds of direct materials.

Now, we use that in the direct materials inventory formula to calculate the number of units of direct materials
that will need to be purchased.

Units in Beginning Inventory + Net Units Purchased – Units Used in Production = Units in Ending Inventory

Let X stand for Units Purchased:

44,000 + X - 88,000 = 48,000


X = 92,000

Question 14 - ICMA 10.P1.043 - Budget Methodologies

A. Zero-based budgeting is a budgeting method in which the budget is prepared without any reference to, or
use of, the current period’s budget and the likely operating results for the current period. In zero-based
budgeting the manager must start from scratch and justify all incomes and expenses proposed.

B. Program budgeting is used mainly by non-profit organizations and governmental bodies. It involves planning and
budgeting for a specific program which may have not only expenses but income as well. If the program has income,
the goal is to have the program income not only cover the program expenses but to exceed them. Program budgeting
is limited to the activities that relate to one specific program and does not extend to the entire budget.

C. Performance budgets use statements of missions, goals and objectives to explain why the money is being spent. It
is a way to allocate resources to achieve specific objectives based on program goals and measured results.
Performance budgeting starts with a goal, then determines strategies and activities to accomplish that goal and justify

(c) HOCK international, page 76


Part 1 : 07/07/14 10:28:01

the expenses. However, that justification is limited to the activities that accomplish a particular goal and does not
extend to the entire budget.

D. Incremental budgeting simply takes historical information and adjusts it for anticipated increases or decreases in
the coming year. This type of budgeting does not involve justifying the entire budget for each budget period.

Question 15 - CIA 1194 II.46 - Forecasting Techniques

A. The coefficient of correlation is a numerical measure that measures both the direction (positive or
negative) and the strength of the linear association between the dependent and independent variables. The
coefficient of correlation lies between -1.0 and +1.0. When the correlation coefficient is positive (between 0
and +1), it means the dependent and independent variables move in the same direction. When the correlation
coefficient is negative (between 0 and −1), it means they move in opposite directions, i.e., when the
independent variable goes up, the dependent variable goes down. When the coefficient of correlation is 0, it
means either that there is no correlation between the two variables, or that the relationship between them is
not linear. To identify the strongest correlation we need to determine the coefficient of correlation that is
closest to either +1 or −1. In this case it is −.89.

B. This is not the strongest correlation. The strongest relationship between the independent and dependent variables
is represented by a correlation coefficient that is closest to either +1 or −1. See the correct answer for a complete
explanation.

C. The correlation coefficient representing the strongest relationship between the independent and dependent variable
is the one that is closest to either +1 or −1. This is the weakest correlation among the answer choices.

D. The coefficient of correlation lies between -1.0 and +1.0. Therefore, the coefficient of correlation could not be 1.03.

Question 16 - ICMA 10.P1.030 - Risk, Uncertainty and Expected Value

A. This is an unweighted average of the possible incomes (losses). The expected value of the additional monthly
income (or loss) is a weighted average of the possible monthly incomes or losses, with the probabilities serving as the
weights.

B. This is an average of the monthly sales in units (both weighted and unweighted). The question asks for the
expected value of the additional monthly income. The expected value of the additional monthly income (or loss) is a
weighted average of the possible monthly incomes or losses, with the probabilities serving as the weights.

C.

The expected value of the additional monthly income (or loss) is a weighted average of the possible monthly
incomes or losses, with the probabilities serving as the weights. Thus, the expected value of the additional
monthly income (loss) is:

(.2 × $[4,000]) + (.3 × $10,000) + (.3 × $30,000) + (.2 × $60,000) = $23,200.

D. This answer results from calculating the expected value of the additional monthly income (or loss) without using a
negative amount for the first amount in the calculation. The $4,000 is a $4,000 loss, and it should be a negative
number in the calculation of the expected value.

(c) HOCK international, page 77


Part 1 : 07/07/14 10:28:01

Question 17 - CIA 594 III.68 - Budget Methodologies

A.

We need to determine the beginning raw materials inventory, the ending raw materials inventory and the
amount of raw materials needed for production, so we can determine how much raw material will need to be
purchased for the given production period of July through September.

Beginning inventory of raw materials as of July 1 needs to be 30% of July's production requirements: 10,000 ×
2 pounds per unit × 30% = 6,000 lb. The question tells us that the company forecasts that there will be 6,000
pounds of raw material on hand at the end of June, so the beginning inventory for July will be sufficient (i.e.,
no additional raw materials will need to be purchased to adjust the beginning inventory). Ending inventory of
raw materials for September needs to be 30% of October's production requirements: 11,000 × 2 pounds per
unit × 30% = 6,600 lb.

The number of units to be produced from July through September is 35,000 units (10,000 + 12,000 + 13,000).
The amount of raw materials required for production in these three months is 70,000 lb. (35,000 × 2 pounds
per unit). Now we can calculate the amount of materials that the company will need to purchase during these
three months. The standard calculation for inventory is as follows, and if we know three of the four numbers,
we can always find the fourth one:
Beginning Inventory 6,000
Plus: Purchases ?
Minus: Materials Used −70,000
Equals: Ending Inventory 6,600

We can solve this with an algebraic equation, or we can simply "back into" the missing purchases number:

6,000 + X − 70,000 = 6,600


X − 64,000 = 6,600
X = 70,600

We know that the difference in the price of materials purchased before June 30th and after June 30th is $0.10
($1.10 − $1.00). Thus, the effect of the price increase for raw materials for the given period is $7,060 ($0.10 ×
70,600).

B. This answer results from using the total number of finished goods units to be produced to calculate the ending raw
materials inventory and the amount of raw materials to be purchased, as if one unit of raw materials were required for
each unit of finished goods. This is incorrect because each unit of finished goods requires two pounds of raw
materials, not one pound.

C.

This is not the correct answer. See the correct answer for a complete explanation.

We have been unable to determine how to calculate this incorrect answer choice. If you have calculated it, please let
us know how you did it so we can create a full explanation of why this answer choice is incorrect. Please send us an
email at support@hockinternational.com. Include the full Question ID number and the actual incorrect answer choice
-- not its letter, because that can change with every study session created. The Question ID number appears in the
upper right corner of the ExamSuccess screen. Thank you in advance for helping us to make your HOCK study
materials better.

D. This is the number of pounds of raw material in beginning inventory on July 1 (6,000) multiplied by the amount of
the price increase (10% of $1.00, or $.10). To answer this question, it is necessary to find the number of pounds of
raw material that will need to be purchased during the three-month period and multiply that by the $.10 per pound
price increase.

(c) HOCK international, page 78


Part 1 : 07/07/14 10:28:01

Question 18 - HOCK CMA P3A H3 - Planning and Budgeting Concepts

A. The actions the company might take based on its strengths and weaknesses, along with its profit objectives, can be
a result of the mission statement and the strategic planning process, but they are not what the mission statement
defines.

B. The mission statement defines why the company exists and also prioritizes and communicates the
company's overall objectives.

C. The company's profit objectives can be a result of the mission statement and the strategic planning process, but
they are not what the mission statement defines.

D. A recognition of the firm's weaknesses can be a result of the mission statement and the strategic planning process,
but it is not what the mission statement defines.

Question 19 - CMA 1294 3.7 - Budget Methodologies

A. This is 60% of the November 30 outstanding accounts receivable plus 60% of the December sales. The collection
pattern is that 60% of sales are collected in the month of the sale and the remainder are collected in the following
month. The $150,000 in accounts receivable at November 30 represents November sales that were not collected in
November and so will be collected in December. Therefore, 100% of the November 30 accounts receivable should be
included in the December collections figure.

B. This is the budgeted level of sales for December. See the correct answer for a complete explanation.

C. Cash collections are equal to 60% of the December sales plus all of the accounts receivable from the
beginning of the period: ($520,000 × 60%) + $150,000 = $462,000.

D. This is 40% of the budgeted December sales. See the correct answer for a complete explanation.

Question 20 - CMA 1289 5.24 - Probability

A.

It is impossible to sell more products that are supplied, so the conditional profit of having 4,000 pretzels and a demand
of 5,000 is the same as having 4,000 pretzels available and a demand of 4,000.

We have been unable to determine how to calculate this incorrect answer choice. If you have calculated it, please let
us know how you did it so we can create a full explanation of why this answer choice is incorrect. Please send us an
email at support@hockinternational.com. Include the full Question ID number and the actual incorrect answer choice
-- not its letter, because that can change with every study session created. The Question ID number appears in the
upper right corner of the ExamSuccess screen. Thank you in advance for helping us to make your HOCK study
materials better.

B.

The meaning of the word "conditional" in "conditional profit" is similar to the meaning of the word
"conditional" in "conditional probability." The conditional probability of two events is the probability that the
second event will occur when it is known that the first event has already occurred. Conditional profit is
conditional because a certain amount of profit (or loss) is associated with each possible course of action,

(c) HOCK international, page 79


Part 1 : 07/07/14 10:28:01

such as purchasing a certain amount of inventory and selling a certain amount of inventory.

In this problem, the first event is the purchase of 4,000 pretzels. So given that we know that 4,000 pretzels
have been purchased, what is the profit from that course of action if demand is 5,000 pretzels? In other
words, in this problem, there are actually two conditions that are known: (1) 4,000 pretzels are supplied, and
(2) demand is 5,000 pretzels. Since the amount supplied is given and the amount demanded is given, the
frequency distribution of the demand for pretzels is irrelevant.

It is impossible to sell more products than are supplied. Furthermore, conditional profit does not take into
consideration any opportunity loss of lost sales because of not being able to fulfill all that is demanded. So
the conditional profit of having 4,000 pretzels and a demand of 5,000 is the same as having 4,000 pretzels
available and having a demand of 4,000. If 4,000 are sold at a price of $1.00 each and the cost is $.30 each, the
profit is ($1 − $.30) × 4,000 = $2,800.

C. This is the profit if 5,000 pretzels are available to sell and demand is 5,000 pretzels. It is impossible to sell more
products that are supplied, so the conditional profit of having 4,000 pretzels and a demand of 5,000 is the same as
having 4,000 pretzels available and a demand of 4,000.

D. It would not be possible to have a loss if they purchase 4,000 pretzels and have a demand for 5,000 pretzels. The
cost of each pretzel is less than the revenue received from selling it, they have no fixed costs to cover, and they will
not have any unsold pretzels.

Question 21 - CMA 690 5.21 - Sensitivity Analysis

A. Sensitivity analysis is a process of changing key variables to determine the possible change in the optimal
solution because of changes in the variables. It is used to define how sensitive the project (sales for example)
is to a change in those variables.

B. PERT is a project scheduling technique used to plan and control projects.

C. Linear programming is used to either maximize or minimize some quantity (called the objective function). At the
same time, this maximizing or minimizing must be accomplished in the presence of constraints, or restrictions, such as
limited quantities of labor or materials. The maximization or minimization must be done without violating any of the
constraints.

D. Expected value analysis is used to determine expected return or cost. Expected value is calculated by multiplying
each projected outcome by its corresponding probability and adding the products together. In other words, expected
value is the weighted average of the probable outcomes.

Question 22 - ICMA 10.P1.057 - Budget Methodologies

A.

This is an answer that we have to work backwards to get. It requires the use of the formulas for calculating
Cost of Goods Sold and Cost of Goods Manufactured.

Direct Labor Used is one of the components of Cost of Goods Manufactured. To find Direct Labor Used, we
will need to know what Cost of Goods Manufactured is. Cost of Goods Manufactured is one component of the
calculation of Cost of Goods Sold. To know what Cost of Goods Manufactured is, we need to use the Cost of
Goods Sold formula, because COGM is the unknown in that formula.

(c) HOCK international, page 80


Part 1 : 07/07/14 10:28:01

The formula is: COGS = Beginning Finished Goods Inventory + Cost of Goods Manufactured – Ending
Finished Goods Inventory. Using the numbers given in the problem and letting X stand for Cost of Goods
Manufactured, we have the following equation:

$10,000 + X – 25,000 = $400,000.


Solving for X, we get Cost of Goods Manufactured = $415,000.

We then work through the Cost of Goods Manufactured formula to find out what Direct Labor Used is. We
know what COGM is, because we worked it out in the COGS formula above.

COGM = Beginning WIP Inventory + Direct Materials Used + Direct Labor Used + Overhead Applied – Ending
WIP Inventory. We have two unknowns here, Direct Labor Used and Overhead Applied, but we know that
Overhead Applied is two times the amount of Direct Labor Used. Therefore, we can let X stand for Direct
Labor Used and 2X stand for Overhead Applied.

Using the numbers we do know and X and 2X for Direct Labor Used and Overhead Applied, the formula is: $0
+ $100,000 + X + 2X – $0 = $415,000.
Solving for X, we get X = $105,000, and that is the amount that should be budgeted for Direct Labor.

B.

This is Cost of Goods Sold minus Direct Material Used plus Ending Inventory minus Beginning Inventory.

Direct Labor is a component of Cost of Goods Manufactured, and Cost of Goods Manufactured is, in turn, a
component in the calculation of Cost of Goods Sold. Therefore, it is necessary to first calculate Cost of Goods
Manufactured using the Cost of Goods Sold formula and from there, calculate Direct Labor Used.

C.

This is Cost of Goods Sold minus Direct Material Used plus Ending Inventory minus Beginning Inventory, the quantity
divided by 2.

Direct Labor is a component of Cost of Goods Manufactured, and Cost of Goods Manufactured is, in turn, a
component in the calculation of Cost of Goods Sold. Therefore, it is necessary to first calculate Cost of Goods
Manufactured using the Cost of Goods Sold formula and from there, calculate Direct Labor Used.

D. This is the amount of overhead to be applied, not the amount of direct labor to be used.

Question 23 - CMA 1296 H13 - Budget Methodologies

A. There is no difference of new project treatment under both of these budget development approaches.

B. This is an advantage of zero-based budgeting, not incremental budgeting.

C. Periodic review of business functions is required regardless the type of budget development approach used.

D. Zero-based budgeting is the budgeting method in which the current year's budget is prepared without any
reference to, or use of, the prior period's budget or actual amounts. Incremental budgeting assumes that the
previous period's budgeted or actual results are satisfactory, and the budget is calculated by adjusting the
previous period budgeted or actual amount by a number, for example 1.1, to allow for changes planned for
the new budgeting period. Thus, it is easier to prepare an ncremental budget and less managerial effort is
consumed than when the budget is prepared under the ZBB concept.

(c) HOCK international, page 81


Part 1 : 07/07/14 10:28:01

Question 24 - CIA 1188 IV.51 - Planning and Budgeting Concepts

A. Budgets are not used check managerial discretion.

B. A budget is a realistic plan for the future expressed in quantitative terms. A budget serves as a planning,
control, and evaluation tool. As such, the use of a budget helps to allocate resources efficiently.

C. Budgets do not provide an automatic corrective mechanism for errors.

D. Budgets help to provide effective and efficient use of recourses, not just the basic usage use of all resources.

Question 25 - ICMA 10.P1.055 - Budget Methodologies

A. This calculation has all of the numbers correct but it reverses the beginning inventory and ending inventory
balances in the inventory equation. The inventory equation is Beginning Inventory + Purchases − Sales = Ending
Inventory.

B.

Needed information to be calculated is beginning inventory for February, sales for February and ending
inventory for February. These are needed in units in order to calculate purchases for February. The inventory
equation is Beginning Inventory + Purchases − Sales = Ending Inventory. When any three of these amounts
are available or can be calculated, the fourth amount can always be calculated. This equation can be used
with either monetary amounts or number of units. Here we have monetary amounts and will convert it to
number of units using the product price of $20 per unit.

The beginning inventory for February will depend upon February sales. February sales can be calculated from
the cash receipts given as $66,000 + $44,000, or $110,000. $110,000 in sales divided by the $20 price per unit =
5,500 units sold during February. Beginning inventory for February will be 30% of 5,500, or 1,650 units.

Ending inventory for February will be based on the number of sales in March, which is $150,000 divided by
$20, or 7,500 units. Ending inventory for February will be 30% of 7,500, or 2,250 units.

The number of units to be purchased in February, using the inventory equation, will be:

1,650 + X − 5,500 = 2,250

X = 6,100 units

C. This answer assumes that beginning inventory was zero. Beginning inventory should be 30% of February's sales,
or 1,650 units.

D. This answer assumes that ending inventory is zero. Ending inventory should be 30% of March sales, or 2,250.

Question 26 - ICMA 10.P1.070 - Budget Methodologies

A. This is 15% of March sales plus 98% of 40% of April sales. The April sales amount still outstanding in receivables
at the end of April should not be decreased by the 2% discount.

B. This answer fails to take into account collections of 15% of February sales and 25% of March sales that are

(c) HOCK international, page 82


Part 1 : 07/07/14 10:28:01

received during April.

C.

May 1 is the same as April 30. On April 30, the company will have outstanding (unpaid) receivables from its
March sales and its April sales only. All sales made during January and February will have been collected by
April 30.

The amount outstanding on April 30 from March sales will be 15% of March sales, because 60% of March
sales will have been collected during March and 25% of March sales will have been collected during April, for
a total of 85% collected. 100% − 85% collected leaves 15% yet to be collected as of April 30.

The amount outstanding on April 30 from April sales will be 40% of April sales, because 60% of April sales
will have been collected during April. 100% − 60% leaves 40% yet to be collected as of April 30.

Receivable balance as of April 30 from March sales: 15% of $280,000 sales = $42,000.

Receivable balance as of April 30 from April sales: 40% of $260,000 sales = $104,000.

The total receivable balance as of April 30 (May 1) is $42,000 + $104,000, which equals $146,000.

D.

This is the portion of April sales that was still outstanding in receivables at the end of April, increased by 2% of the
amount of April sales that were collected during April. This answer is incorrect for two reasons.

One, it fails to take into account the portion of March sales that were also still outstanding at the end of April. 15% of
sales made in March had not been collected as of the end of April.

Two, it fails to recognize that when payments were received in April for sales made in April and the customers took the
2% discount, the dollar amount of the discounts taken did not remain in accounts receivable as outstanding
receivables. As the discounts are honored, the unpaid 2% is debited to an expense account and credited to accounts
receivable, so it is removed from outstanding accounts receivable.

Question 27 - ICMA 13.P1.007 - Forecasting Techniques

A. This is the forecasted sales for June.

B.

This is not the correct answer. Please see the correct answer for an explanation.

We have been unable to determine how to calculate this incorrect answer choice. If you have calculated it, please let
us know how you did it so we can create a full explanation of why this answer choice is incorrect. Please send us an
email at support@hockinternational.com. Include the full Question ID number and the actual incorrect answer choice
-- not its letter, because that can change with every study session created. The Question ID number appears in the
upper right corner of the ExamSuccess screen. Thank you in advance for helping us to make your HOCK study
materials better.

C.

The formula to calculate a forecast using exponential smoothing is

Ft+1 = α Yt + (1 − α) Ft
Where:

(c) HOCK international, page 83


Part 1 : 07/07/14 10:28:01

Ft+1 =forecast for the next period


Yt =actual value for period t
Ft =forecasted value for period t
α =smoothing constant (0-1)

To calculate forecasted sales for July, we first need to calculate forecasted sales for June, because that figure
is needed to calculate forecasted sales for July.

June forecasted sales = (0.7 × $22,000,000) + (0.3 × $21,400,000) = $21,820,000.

July forecasted sales = (0.7 × $22,500,000) + (0.3 × $21,820,000) = $22,296,000.

D. This answer results from reversing the 0.7 smoothing constant and (1 − 0.7) in the exponential smoothing formula.

Question 28 - CMA 1293 H2 - Budget Methodologies

A. Interest was $1,000, not $900. Additional borrowings are needed as there is a cash deficit in May. See the correct
answer for a complete explanation.

B.

First, we need to determine the beginning cash balance for May and the amount of interest that has to be paid
in May for April borrowings, since interest is paid monthly. To do this we need to determine if there were any
borrowings in April.

To determine what the April borrowings were, we need to first determine the cash collections for April: 50% of
April sales and 50% of March sales (or $25,000 + $20,000 = $45,000) will be collected in April. Then, we need to
determine the amount paid for accounts payable in April: 75% of April A/P and 25% of March A/P (or $30,000 +
$7,500 = $37,500) will be paid in April. Other disbursements are paid in the month they occur, and for April
they are: $70,000 for payroll plus $30,000 for other disbursements, totaling $100,000.

Subtracting the amount of cash outflows from cash inflows we get a $92,500 negative net cash flow for the
month. We assume that April's beginning cash was $100,000. Therefore, the company's April ending cash
balance before any borrowings was $100,000 − $92,500, or $7,500.

The company needed to have $100,000 in cash at the end of April. Since borrowings for cash deficits must be
made in $10,000 increments, the company needed to borrow $100,000 to cover the $92,500 cash deficit so it
could end the month with at least $100,000 in cash. After borrowing $100,000, the April ending cash balance
was $107,500; but the extra $7,500 in the cash account is unavoidable because of the $10,000 incremental
borrowing requirement.

The company will need to pay $1,000 of interest on May 31 ($100,000 × [12% ÷ 12]) for the April borrowing.

Next, we need to determine the cash inflows and outflows for May as we did for April. Cash collections in May
are 50% of the April and May sales (50% × $50,000) + (50% × $100,000) = $75,000. Accounts payable that will
be paid in May are 75% of May's AP and 25% of April's AP: ($40,000 × 75%) + ($40,000 × 25%) = $40,000. Other
disbursements total $61,000 ($50,000 for payroll + $10,000 in other disbursements + $1,000 in interest for
borrowings during April). Subtracting the total disbursements from the collections in May we get a $26,000
negative cash flow ($75,000 − $40,000 − $61,000).

At the beginning of the month, the company had a cash balance of $107,500. $107,500 minus the $26,000
negative net cash flow during May will result in a May ending cash balance before any borrowing of $81,500.
But remember the company needs to end the month with a cash balance of $100,000. They are $18,500 short.
Since borrowings for cash deficits must be made in $10,000 increments, the company needs to borrow

(c) HOCK international, page 84


Part 1 : 07/07/14 10:28:01

$20,000 to cover the $18,500 cash deficit for May and end the month with at least $100,000 in cash. In fact,
they will end the month of May with $101,500 in cash. ($107,500 + $75,000 - $40,000 - $61,000 + $20,000 =
$101,500).

C. Principal repayments are to be made in any month in which there is a surplus of cash. There was no cash surplus
in May so no principal would be repaid. Also the interest is not $100. See the correct answer for a complete
explanation.

D. Principal repayments are to be made in any month in which there is a surplus of cash. There was no cash surplus
in May so no principal would be repaid. See the correct answer for a complete explanation.

Question 29 - CMA 1280 5.15 - Probability

A. Statistical sampling is used to calculate the condition of a population based on a sample from that population.

B. Probability gives a means of measuring numerically how likely it is that an event will occur. It enables us to
quantify and analyze uncertainties. Usage of probability technique can provide an estimate of what condition
the tires of the fleet are in at any point in time and the expected value of replacing all tires at once versus
replacing them as they wear out.

C. Learning curves describe the fact that the more experience people have with something, the more efficient they
become in doing that task. Higher costs per unit early in production are part of the start-up costs. Usually new
products and production processes experience a period of low productivity followed by increased productivity.
However, the rate of productivity improvement declines over time until it reaches a level where it remains, until another
change in production occurs.

D. Linear programming is used to either maximize or minimize some quantity (called the objective function) in the
presence of constraints, or restrictions, such as limited quantities of labor or materials.

Question 30 - ICMA 10.P1.069 - Budget Methodologies

A. First, we need to find the August selling price per unit, so we can calculate what the price will be after the
price cut. To find the August selling price per unit, we begin by calculating the total sales revenue for the
month of August. The variable costs are the sales commissions, which are 6.2% of sales revenue. Therefore,
sales revenue for August was $372,000 ÷ .062, or $6,000,000. Since that revenue was for 20,000 units, the
price per unit in August was $6,000,000 ÷ 20,000, or $300 per unit.

The price will be cut by 10%, to $270 per unit ($300 × .90). Number of units sold will increase to 24,000.
Therefore, sales revenue for September will be $270 × 24,000, or $6,480,000. The variable cost is sales
commissions at 6.2%, so the variable cost will be $6,480,000 × .062, or $401,760.

Step costs are based on the number of salespeople. In August, when there were 17 salespeople, the total step
costs were $85,000. In September, there will be only 15 salespeople. So step costs will be $85,000 ÷ 17 × 15,
or $75,000.

Fixed costs will not change, because the fixed costs will stay the same until volume exceeds 30,000, and
volume will not exceed 30,000 in September. Therefore, fixed costs are $176,000.

The sum of all of these costs is $401,760 + $75,000 + $176,000 = $652,760.

B. This answer results from two incorrect calculations: (1) the assumption is made that more salespeople will be

(c) HOCK international, page 85


Part 1 : 07/07/14 10:28:01

required to sell more product. I.e., it is assumed that since 17 salespeople were required in August to sell 20,000
units, then 20.4 salespeople will be required in September to sell 24,000 units (17 ÷ 20,000 × 24,000). At a step cost
of $5,000 per salesperson ($85,000 August expense divided by 17 salespeople), this would mean the step costs for
September would be $5,000 × 20.4 salespeople, or $102,000. And (2) the fixed costs are assumed to increase with
the increased production, to $8.80 × 24,000, or to $211,200. However, the problem tells us that two salespeople will
be leaving at the end of August and will not be replaced, so the number of salespeople in September will be only 15 at
$5,000 each, for a total cost of $75,000. Furthermore, the fixed cost does not change with the increased sales,
because the sales volume is still within the relevant range.

C.

This answer results from assuming that more salespeople will be required to sell more product. I.e., it is assumed that
since 17 salespeople were required in August to sell 20,000 units, then 20.4 salespeople will be required in
September to sell 24,000 units (17 ÷ 20,000 × 24,000). At a step cost of $5,000 per salesperson ($85,000 August
expense divided by 17 salespeople), this would mean the step costs for September would be $5,000 × 20.4
salespeople, or $102,000. However, the problem tells us that two salespeople will be leaving at the end of August and
will not be replaced, so the number of salespeople in September will be only 15 at $5,000 each, for a total cost of
$75,000.

Furthermore, the nature of step costs is that they stay at one level for a while and then make a large increase to the
next level. In this case, each time a salesperson is added, the total cost increases by $5,000; and each time a
salesperson leaves, the total cost decreases by $5,000. The company does not hire part-time salespeople, so it could
not possibly have 20.4 salespeople.

D. This is the total per unit cost at a sales level of 20,000 multiplied by 24,000 units. This fails to take into
consideration the facts that the step costs are based on the number of salespeople, not the number of units sold, and
that fixed costs in total will not change because the anticipated sales volume is within the relevant range.

Question 31 - CMA 692 H7 - Budget Methodologies

A. The budgeted statement of financial position (balance sheet) is based on a number of elements of the master
budget including the budgeted income statement, so it is not an independent item in the preparation of the master
budget.

B. The desired level of ending inventory not independent, as it is used in the development of the production budget.

C. The budgeted income statement is a critical element of the master budget. It is one of the last budgets created from
the operating budgets. The budgeted income statement is based on the sales budgets, expense budgets and other
elements of the master budget, so it is not an independent item in the preparation of the master budget.

D. The capital budget is the budget in which all capital (property, plant and equipment) expenditures are
planned. This budget is not directly connected to the current period budgets and it is often prepared years in
advance so that the company can plan to obtain the necessary financing or accumulate the necessary cash to
carry out its capital expansion plans. The capital budget is often considered to be independent from the
master budget.

Question 32 - CMA 1292 4.21 - Probability

A. This is not possible, as the maximum the beverage stand can obtain is $2,500 if it sells soft drinks when the
weather is hot.

B. This is the expected payoff for selling coffee when the weather is cold multiplied by the probability of cold weather

(c) HOCK international, page 86


Part 1 : 07/07/14 10:28:01

of .60, plus the expected payoff for selling soft drinks when the weather is hot multiplied by the probability of hot
weather of .40. That is not the expected payoff for selling coffee.

C.

The beverage stand can sell either soft drinks or coffee on any given day − not both. Since the question asks
for the expected payoff for selling coffee, the proprietor is asking for the answer to this question: "Assuming I
take coffee with me to sell today, what is my expected payoff for doing that?" Thus, the probability is 100%
that coffee will be served.

To solve this problem we have to identify the expected payoff of selling coffee when we know the
probabilities of the weather being hot and cold. That will be the weighted average of the expected payoffs for
serving coffee, weighted according to the probabilities of cold weather and hot weather. So we will multiply
each possible payoff for selling coffee by its corresponding probability.

If the stand sells coffee and the weather is hot, it will make $1,900, and the probability of hot weather is 40%
(100% − 60% probability of cold weather). If the stand sells coffee and the weather is cold, it will make $2,000,
and the probability of cold weather is 60%. Thus, the weighted average profit for selling coffee is ($1,900 ×
.40) + ($2,000 × .60), which is $1,960. And that is the expected payoff for selling coffee.

D. This is the expected payoff for selling soft drinks when the weather is cold multiplied by the probability of cold
weather of .60, plus the expected payoff for selling coffee when the weather is hot multiplied by the probability of hot
weather of .40. That is not the expected payoff for selling coffee.

Question 33 - CMA 1292 4.22 - Risk, Uncertainty and Expected Value

A.

With perfect information about whether the weather will be hot or cold, the company would be able to choose
in advance the correct drink to supply for each of the types of weather.

If the beverage stand knew in advance that the weather would be hot, it would supply soft drinks and make
$2,500. If it knew in advance that the weather would be cold, it would supply coffee and make $2,000.

Therefore, given that for each type of weather, the company will choose the best supply alternative, the
expected payoff is the weighted average of the best payoff that can be achieved on a cold day and the best
payoff that can be achieved on a hot day, with the probabilities of each weather condition as the weights.
Since the probability of cold weather on a given day at this time is 60%, we know that the probability of hot
weather is 40%. So the beverage stand's expected profit if it has perfect information is ($2,000 × .6) + ($2,500 ×
.4), which is $2,200.

B. This is the expected payoff for selling soft drinks when the weather is cold multiplied by the probability of cold
weather of .60, plus the expected payoff for selling coffee when the weather is hot multiplied by the probability of hot
weather of .40. However, if the vendor has perfect information, it would not sell soft drinks on a cold day, nor would it
sell coffee on a hot day. It would do just the opposite.

C. This is the profit for selling coffee in hot weather ($1,900) plus the profit for selling coffee in cold weather ($2,000).
This payoff is not possible, as the beverage stand can obtain only one profit payoff. Furthermore, the maximum profit
the beverage stand can obtain is $2,500. That would occur if the weather is hot and it sells soft drinks.

D. This is expected payoff for selling coffee without perfect information, calculated as ($1,900 × .40) + ($2,000 × .60).

(c) HOCK international, page 87


Part 1 : 07/07/14 10:28:01

Question 34 - CMA 691 3.15 - Budget Methodologies

A. The direct labor budget is part of the operating budget. The direct labor budget is developed after the production
budget is developed, but it is not the last budget to be prepared.

B. The manufacturing overhead budget is part of the operating budget. The manufacturing overhead budget is
developed after the production budget, but it is not the last budget to be prepared.

C. Of all the budgets given in the answer choices, the cash budget is the last budget to be prepared because
all other budgets are inputs to it.

D. The cost of goods sold budget is developed after the production budget. It is a part of the operating budget but it is
not the last budget to be prepared.

Question 35 - ICMA 13.P1.009 - Risk, Uncertainty and Expected Value

A.

This is not the correct answer. Please see the correct answer for an explanation.

We have been unable to determine how to calculate this incorrect answer choice. If you have calculated it, please let
us know how you did it so we can create a full explanation of why this answer choice is incorrect. Please send us an
email at support@hockinternational.com. Include the full Question ID number and the actual incorrect answer choice
-- not its letter, because that can change with every study session created. The Question ID number appears in the
upper right corner of the ExamSuccess screen. Thank you in advance for helping us to make your HOCK study
materials better.

B. This is [.70 × $10,000,000] + [.75 × $(4,000,000)]. The expected value of the new product's annual profit is a
weighted average of the expected value of profits with favorable research results and the expected value of profits
with unfavorable research results.

C. This is the average of $10,000,000 and $(4,000,000). The expected value of the new product's annual profit is a
weighted average of the expected value of profits with favorable research results and the expected value of profits
with unfavorable research results.

D.

The expected value of the new product's annual profit is a weighted average of the expected value of profits
with favorable research results and the expected value of profits with unfavorable research results.

The expected value of profits with favorable research results is [.70 × $10,000,000] + [.30 × $(4,000,000)] =
$5,800,000. The probability that the research results will be favorable is 60%.

The expected value of profits with unfavorable research results is [.25 × $10,000,000] + [.75 × $(4,000,000)] =
$(500,000). The probability that the research results will be unfavorable is 40%.

The expected value of the new product's annual profit is [.60 × $5,800,000] + [.40 × $(500,000)] = $3,280,000.

Question 36 - ICMA 10.P1.060 - Budget Methodologies

A.

(c) HOCK international, page 88


Part 1 : 07/07/14 10:28:01

This is not the correct answer. Please see the correct answer for an explanation.

We have been unable to determine how to calculate this incorrect answer choice. If you have calculated it, please let
us know how you did it so we can create a full explanation of why this answer choice is incorrect. Please send us an
email at support@hockinternational.com. Include the full Question ID number and the actual incorrect answer choice
-- not its letter, because that can change with every study session created. The Question ID number appears in the
upper right corner of the ExamSuccess screen. Thank you in advance for helping us to make your HOCK study
materials better.

B. This answer results from using the amount of sales dollars to calculate the amount of purchase dollars needed
during the month. The sales dollar amount includes profit, as well as the cost of the sales. Only the cost of the sales is
used when calculating the amount of purchases to be made and anything else having to do with inventory. Each sales
amount should be reduced to the cost of the sales, which is 65% of the sales amount because the company maintains
a 35% gross profit margin.

C.

Cost of goods sold figures are used in calculating the amount of purchases. Cost of goods sold are 65% of
sales for each month, since the company maintains a 35% gross profit margin (100% − 35% = 65%). COGS for
October is therefore 65% of $42,000, or $27,300. COGS for November is 65% of $58,000, or $37,700. And COGS
for December is 65% of $74,000, or $48,100.

The company carries an ending inventory balance each month that is sufficient to support 30% of the next
month's expected sales (that is, 30% of the COGS of the next month's expected sales, which is 65% of the
next month's expected sales). Therefore, the ending inventory balance at the end of October will be 30% of
November's COGS of $37,700, or $11,310. This will also be the beginning inventory balance for the month of
November. The ending inventory balance for November will be 30% of December's COGS of $48,100, or
$14,430.

The basic inventory formula is Beginning Inventory + Purchases − Cost of Goods Sold = Ending Inventory. We
know beginning inventory for November ($11,310), COGS for November ($37,700) and ending inventory for
November ($14,430), so we can calculate the Purchases for November. Letting P stand for Purchases, the
formula is:

$11,310 + P − $37,700 = $14,430

Solving for P, we get P = $40,820.

D. This answer results from using December's planned Cost of Goods Sold to calculate November's planned
purchases. November's Cost of Goods Sold should be used in calculating November's purchases.

Question 37 - CMA 1286 5.3 - Risk, Uncertainty and Expected Value

A. This is a partial explanation of the way the standard deviation is calculated, but it is not the complete explanation. It
is also not the definition of expected value.

B. The expected value is the weighted average of all the possible outcomes, with the probability of each
possible outcome serving as its weight.

C. This is not the definition of expected value. This answer has no meaning.

D. This is not the definition of expected value. Standard deviation is a measure of how closely together all of the items
in the population are to each other. It is also the square root of the variance. The variance and standard deviation both
give us an idea of the variability of the possible values from the mean.

(c) HOCK international, page 89


Part 1 : 07/07/14 10:28:01

Question 38 - HOCK CMA.P1A5.01 - Top-Level Planning and Analysis

A. Higher sales may lead to higher profits; but increased cash from higher profits is not the only effect the higher sales
will have on cash.

B. The higher the firm's rate of growth in sales, the greater will be its need for additional financing. When
sales increase, firms usually need to purchase more assets to support the increased level of sales. More
inventory will need to be purchased, and additional equipment will be needed to expand manufacturing.
Additional employees will be required to operate the new equipment and also to make and process the
additional sales and to provide customer service after the sale. Some of the required funding can be provided
by increases in profits and increases in accounts payable and accruals (called "spontaneous liabilities"); but
not all of the required funding can be provided in that way. The company will need to raise additional external
financing either in the form of borrowed funds or in the form of new equity.

C. While it is true that more inventory will be sold when sales increase, more inventory will also need to be
manufactured or purchased, not only to replace the inventory that has been sold but also to have enough inventory on
hand to support the increased sales. So the increased cash generated by increased inventory sales will be offset by
the need to acquire more inventory.

D. A firm in a high-growth stage does not necessarily need to decrease its prices in order to keep sales growing, so
this is an incorrect assumption.

Question 39 - CMA 1288 5.20 - Learning Curves

A. Fixed overhead costs per lot (a lot equals 50 units) would decrease since the cost is applied at a rate based on
direct labor hours, which is decreasing.

B. Since there are two doublings, the number of hours required for 200 units using a 70% learning curve is:
1,000 hours × (.7 × 2) × (.7 × 2) = 1,960 hours. 1,960 hours required for 200 units less 1,000 hours required for
the first 50 units = 960 hours required for the last 150 units. 960 hours ÷ 150 units = 6.4 hours required per
unit for the last 150 units.

C. If there were no learning curve the total direct hours would be approximately 4,000 hours, instead of 1,960 hours, a
change of more than 200%.

D. If there were an 80% learning curve instead of 70%, the reduction in labor hours would be more than 25% (based
on the difference of 1,960 hours at an 80% learning curve and 1,560 hours at a 70% learning curve).

Question 40 - CMA 691 3.12 - Budget Methodologies

A. The inflation rate may be considered in various types of budgets, for example a fixed or static budget, a project
budget, or a flexible budget.

B. Flexible budgets are prepared for different levels of activity and do not evaluate the usage of capacity.

C. Flexible budgets are prepared for different levels of activity. In other words, a flexible budget is based on the static
budget, but variable revenues and variable expenses are adjusted upward or downward to reflect the actual activity
level. The actual activity level might well change when prices change, but the flexible budget does not incorporate
changes in prices in its budgeted amounts, only changes in volume.

(c) HOCK international, page 90


Part 1 : 07/07/14 10:28:01

D. Flexible budgets are prepared for different levels of activity. In other words, a flexible budget is based on
the static budget, but variable revenues and variable expenses are adjusted upward or downward to reflect
the actual activity level. A flexible budget is more useful than a static budgetsas it allows the company to
compare the actual results with the budgeted results that have been adjusted for the actual level of activity.

Question 41 - ICMA 10.P1.002 - Planning and Budgeting Concepts

A. Throughout the budget process the various departments within an organization will meet to plan for the next year.
These meetings will foster cooperation, communication and familiarity which will carry over after budgeting season.

B. Budgets provide performance information, in that management can analyze what they thought would happen vs.
what actually happened. From that point they can determine what factors were within their control and what was
outside of their control and use this information for future budgets and operational decisions.

C. Forcing management to focus on the future and make appropriate plans for the organization's success is a
significant advantage of the use of budgets.

D. Even the most advanced and accurate budget cannot prevent unauthorized expenditures. This is
controlled through management oversight, rather than the budgeting process.

Question 42 - CMA 697 4.22 - Risk, Uncertainty and Expected Value

A. $85,000 is the sales estimate with the highest probability level. The problem asks for the expected value of Philip's
budgeted cost of goods sold.

B. $84,000 is the expected value for sales in 2013. The problem asks for the expected value of Philip's budgeted cost
of goods sold.

C. Cost of goods sold averages 80% of sales. In order to determine the expected value of cost of goods sold,
we first need to calculate the expected value for sales. Cost of goods sold will be equal to 80% of the sales
value. The expected sales can be calculated by multiplying each of the possible outcomes by the probability
that it will occur and adding the products together. When we do this, we get $84,000 as the expected sales
[($60,000 × .25) + ($85,000 × .4) + ($100,000 × .35)]. The expected value of cost of goods sold is 80% of this, or
$67,200.

D. $68,000 is the cost of goods sold calculated using the sales estimate with the highest probability level.

Question 43 - CMA 1293 4.24 - Learning Curves

A. To solve this problem first we need to identify the learning curve percentage. The learning curve is a
percentage of time reduction to complete a task for each doubling of cumulative production. In this case it is
60% ($72 ÷ $120). Hence, the cumulative average labor cost of a unit after the fourth batch is $43.20 ($72 ×
60%). There are 400 units in 4 batches. The total labor cost is $17,280 ($43.20 × 400).

B. This is the average cost of one unit after 800 units have been produced multiplied by 400.

C. This is the average cost of one unit after 400 units have been produced multiplied by 100.

(c) HOCK international, page 91


Part 1 : 07/07/14 10:28:01

D. This is the average cost of one unit after 800 units have been produced multiplied by 100.

Question 44 - ICMA 10.P1.071 - Budget Methodologies

A. The payments of property taxes will be made during the last month of each calendar quarter. The last months of
each calendar quarter are March, June, September and December. Since a quarterly property tax payment will not be
made during July, it should not be considered when developing July's cash budget.

B. The payment of federal income tax and social security tax withheld from employees' June paychecks to be
remitted to the Internal Revenue Service in July will affect July cash because it is a July disbursement to be
made. Therefore, it should be considered when developing July’s cash budget.

C. The quarterly cash dividend will be accrued in July. However, it will not be paid until August, so it will not affect
cash during July. Therefore, it should not be considered when developing July's cash budget.

D. The allowance for uncollectible accounts is recorded by debiting Bad Debt Expense and crediting the Allowance for
Uncollectible Accounts account. Since it is not a cash transaction, it should not be considered when developing July's
cash budget.

Question 45 - ICMA 10.P1.051 - Budget Methodologies

A. This answer does not ake into consideration the beginning inventory that does not need to be produced. The
beginning inventory will lower budgeted production by 3,300 units.

B. This answer results from dividing the beginning inventory of 3,500 units by 10 to calculate the 10 days of desired
ending inventory. The ending inventory should be based on the projected sales. To calculate the ending inventory,
divide the 67,500 units that are expected to be sold in the first quarter by 90 days (360 days ÷ 4) to get the number of
units sold per day. Use that to calculate the number of units needed for 10 days of sales, and that will be the ending
inventory that is needed. Also, the beginning inventory of 3,500 units includes 200 units that are obsolete and will
need to be subtracted from the beginning inventory in calculating the budgeted production for the quarter.

C.

The first thing to do is figure out how much ending inventory is needed. The company wants to have an
ending inventory equal to 10 days of sales. The first quarter sales budget is 67,500 units. Since the company
uses a 360 day year, one quarter's sales will be sales for 90 days (360 ÷ 90). Therefore, sales for one day
would be 67,500 ÷ 90, or 750 units. Inventory for 10 days would be 10 days × 750 units, or 7,500 units needed
in ending inventory.

Now we can use the inventory formula to find the number of units to be produced. The inventory formula is:
Beginning Inventory + Units Produced or Purchased − Units Sold = Ending Inventory. Whenever we have 3 of
the 4 numbers, we can find the 4th number.

Beginning inventory = 3,300 good units (3,500 units minus the 200 obsolete units)
Sales = 67,500 units
Ending Inventory = 7,500 unit

3,300 + X − 67,500 = 7,500


X = 71,700

D. This answer results from using a beginning inventory of 3,500 units. 3,500 units includes the 200 units that are

(c) HOCK international, page 92


Part 1 : 07/07/14 10:28:01

obsolete and which should not be included.

Question 46 - ICMA 08.P2.12 - Budget Methodologies

A. Option 2 is correct, but option 3 won't fix the problem of "slack" that Thomas is concerned about.

B. Option 2 and option 4 will generate a budget and that has employee buy in, but it may not eliminate slack.

C. A budget developed by top management and issued to lower level units is unlikely to motivate employees.

D. Options 2, 4 and 5 are the best way to eliminate slack through the rounds of changes and the review of
historical information. Option 4 will also help generate employee buy in and motivate them to help achieve the
overall corporate targets.

Question 47 - CMA 690 5.18 - Risk, Uncertainty and Expected Value

A.

To determine the maximum amount that a company would pay for perfect information, we compare the
maximum profit that the company could achieve with perfect information with the best profit that would be
available if we needed to choose one level when demand is not known.

With perfect information the company would be able to choose the correct level of supply for each of the
levels of demand. For example, if he knew the demand would be for 20 bags, Stan Berry would bring 20 bags
and his profit would be $20. If he knew the demand would be 30 bags, he would bring 30 bags and the profit
would be $30. And so on. However, he would be paying for this "perfect information" in advance, before he
knows what the information will be. So to determine what he is willing to pay for the information in advance,
he will calculate the expected value of his profit, assuming he will be able to bring exactly the right number of
bags once he knows what the demand will be. He will use the various probabilities of demand as the weights
in calculating this expected value.

Therefore, given that for each level of demand the company will choose the best supply alternative, the
expected value of the profit with perfect information is $33 [($20 × .2) + ($30 × .4) + ($40 × .3) + ($50 × .1)].

Without perfect information, Stan Berry would choose to stock 40 bags, because the expected profit for
stocking 40 bags is $30.40, and that is higher than the expected profits for any of the other three potential
supply levels.

Since perfect information would provide an expected profit of $33.00 while the expected profit without the
perfect information is $30.40, Stan Berry would pay a maximum of $2.60 (the difference between $33.00 and
$30.40) for the perfect information.

B. The price that Stan Berry would pay for perfect information is the difference between the expected profit with
perfect information and the expected profit without perfect information. See the correct answer for a complete
explanation.

C. The price that Stan Berry would pay for perfect information is the difference between expected profit with perfect
information and expected profit without perfect information. This answer is the difference between the highest
expected profit he could earn without perfect information ($30.40) and the second highest expected profit he could
earn, also without perfect information ($29.60).

(c) HOCK international, page 93


Part 1 : 07/07/14 10:28:01

D. The price that Stan Berry would pay for perfect information is the difference between expected profit with perfect
information and expected profit without perfect information. This is the difference between the highest expected profit
he could earn without perfect information ($30.40) and the lowest expected profit he could earn, also without perfect
information ($20.00).

Question 48 - CMA 1296 H10 - Budget Methodologies

A.

This question asks for the amount of cash collected in March from sales made during March. Collections
expected during March from credit sales made during previous months are not included.

In March Karmee Company will collect $140,000 (20% × $700,000) from the cash sales made in March and
$168,000 ($700,000 × 80% × 30%) from the credit sales made in March. The total amount of March sales
collected in March is $308,000.

B. This is the total cash collections in March and it includes collections from previous months' sales as well as from
March sales. However, the question asks for only the amount of cash collected in March from the sales made during
March. Collections expected during March from credit sales made during previous months are not included.

C. This is the amount of cash collected in March from credit sales made in March. However, the amount of cash
collected from cash sales made in March must be included as well.

D. This is the amount of cash collected in March from cash sales made in March. However, the amount of cash
collected from credit sales made in March must be included as well.

Question 49 - CMA 691 H2 - Planning and Budgeting Concepts

A. The SEC (Securities Exchange Commission) does not require businesses to budget.

B. Zero-base budgeting does not need to be used, and is rarely used, in governmental budgeting.

C. Governmental budgets differ from business budgets because a governmental budget represents the legal
amount that the government can spend. In order to spend more than the budgeting amount, legislation must
be passed by the government to allow the additional spending.

D. Governmental budgeting can also be used to measure progress in achieving objectives.

Question 50 - CMA 1296 H11 - Budget Methodologies

A. In February the production will be equal to 1/2 of March sales. March sales are expected to be 33,000, so
February will see production of 16,500 units. The variable cost per unit is $7 ($3.50 + $1 + $2 + $.50), so total
variable costs will be $115,500 ($7 × 16,500). We need to add to this the $12,000 of fixed costs giving us a total
production cost of $127,500.

B. This is the production budget for January. See the correct answer for a complete explanation.

C. These are the costs based on production of 150% of January sales. See the correct answer for a complete

(c) HOCK international, page 94


Part 1 : 07/07/14 10:28:01

explanation.

D. This is production budget for April. See the correct answer for a complete explanation.

Question 51 - ICMA 10.P1.044 - Planning and Budgeting Concepts

A. Many companies experience seasonal demand for their products and this can easily be built into a budget. By
straight lining the line items, some of the months will reflect significant unfavorable variances while other months will
be decidedly favorable. These swings can hide true budgetary problems or opportunities as variances will be brushed
off as resulting from seasonal fluctuations.

B.

The use of a flexible budget rather than a fixed budget is not a cause of behavior problems. The way
management chooses to develop the budget and the way management chooses to use the budget can be
causes of behavior problems.

When senior management imposes the budget on employees without receiving any input from the employees
who will be responsible for achieving the budget, this causes a lack of motivation in the employees because
they don't see the budget as "theirs." When managers are held responsible for costs that they have no control
over, that is demotivating as well. Furthermore, just dividing annual budget amounts by 12 and expecting
managers to account for their performance against the budget every month puts the managers in an
impossible situation because it does not incorporate normal seasonal fluctuations that managers cannot
control. All of those things will create behavior problems.

When a flexible budget is used, variable items in the budget are adjusted to actual activity (production and
sales) levels by multiplying per unit budgeted amounts by the actual activity achieved in units. This ensures
that the focus is on variances due to causes other than the volumes of output and sales. Use of a flexible
budget does not guarantee that the budget will not be imposed in an authoritarian manner or that managers
will not be held responsible for costs that they cannot control. Use of a flexible budget does not, by itself,
prevent behavior problems caused by those things. But use of a flexible budget also does not, by itself, cause
behavior problems.

C. The inclusion of depreciation in and of itself isn't the problem. But depreciation is not controllable by the department
management in most cases, because decisions on purchases of fixed assets are usually made at a higher level.
Department managers should not be held accountable for items beyond their control – in this case, that is likely all of
the fixed costs.

D. When workers feel that they are being dictated to, they are less likely to buy into the budget. An attitude of
"Management didn’t ask for or respect my opinion, so why should I respect their wishes?" will lead to morale problems
as well as increased unfavorable budget variances.

Question 52 - CMA 695 H7 - Budget Methodologies

A. This answer results from using the June 30 finished goods inventory given in the problem as the beginning finished
goods inventory for August. The beginning finished goods inventory for August must be calculated as 40% of August
sales.

B.

The beginning inventory level of finished goods in August is 40% of August sales, or 1,000 units (40% ×

(c) HOCK international, page 95


Part 1 : 07/07/14 10:28:01

2,500). The ending inventory of finished goods in August is 40% of the sales in September, or 840 units (2,100
× 40%). Now we can determine the number of units to produce in August using the following basic formula:

Beginning Inventory + Units Produced − Units Sold = Ending Inventory. So the equation is:

1,000 + P − 2,500 = 840.

Solving for P, we get P = 840 − 1,000 + 2,500, and P = 2,340.

C. This is the number of units that will be produced in July. See the correct answer for a complete explanation.

D. This is the June 30 finished goods inventory.

Question 53 - CIA 598 3.21 - Budget Methodologies

A. This is the definition of an operating budget.

B. This is the definition of activity-based budgeting.

C. This is the definition of Kaizen budgeting.

D. This is the definition of a flexible budget.

Question 54 - CIA 1187 III.41 - Learning Curves

A. Under the incremental unit-time learning model, this is the incremental labor cost as a percentage of the first unit
produced that is required for the 4th unit. See the correct answer for a complete explanation.

B. Under the incremental unit-time learning model, this is the incremental labor cost as a percentage of the first unit
produced that is required for the 32nd unit, not the 16th unit. See the correct answer for a complete explanation.

C. In a learning curve situation in which the labor costs are reduced by 20% each time that production is
doubled, this means that if the first unit requires 100% of the labor cost, under the incremental unit-time
learning model, the second unit will require only 80% of the labor cost of the first unit. The 4th unit will require
only 80% of the labor cost of the 2nd unit, or 80% of 80%, which is 64%. The 8th unit will require 80% of the
labor cost of the 4th unit, or 80% of 64%, which is 51%. The 16th unit will require 80% of the labor cost of the
8th unit, or 80% of 51%, which is 41% of the first unit's direct labor cost.

D. Under the incremental unit-time learning model, this is the incremental labor cost as a percentage of the first unit
produced that is required for the 8th unit. See the correct answer for a complete explanation.

Question 55 - CMA 1296 H6 - Budget Methodologies

A. This answer does not include variable expenses or advertising and it includes depreciation. See the correct answer
for a complete explanation.

B. This answer incorrectly includes depreciation. Depreciation is not a cash expenditure. See the correct answer for a
complete explanation.

(c) HOCK international, page 96


Part 1 : 07/07/14 10:28:01

C. This answer does not include variable expenses or advertising. See the correct answer for a complete explanation.

D. Operating expenses other that COGS paid in the month of April are: 10% of March's sales, the advertising
monthly payment, monthly salaries, the quarterly insurance payment and the property tax payment made
twice a year. Depreciation is not a cash expense and is therefore not included in calculation. The
disbursements are: $70,000 for variable operating expenses (March sales of $700,000 × 10%), $60,000 for
advertising ($720,000 ÷ 12), $90,000 for salaries ($1,080,000 ÷ 12), $45,000 for insurance ($180,000 ÷ 4), and
$120,000 for property taxes ($240,000 ÷ 2). The total amount of disbursements in April is $385,000.

Question 56 - ICMA 10.P1.023 - Learning Curves

A. This is $20,000 × 8 × .70. This is not the correct way to calculate the total manufacturing cost for eight propellers.

B.

This is not the correct answer. Please see the correct answer for an explanation.

We have been unable to determine how to calculate this incorrect answer choice. If you have calculated it, please let
us know how you did it so we can create a full explanation of why this answer choice is incorrect. Please send us an
email at support@hockinternational.com. Include the full Question ID number and the actual incorrect answer choice
-- not its letter, because that can change with every study session created. The Question ID number appears in the
upper right corner of the screen. Thank you in advance for helping us to make your HOCK study materials better.

C.

The total cost to produce eight propellers, using the Cumulative Average-Time Learning Model, is

$20,000 (2 × .70) (2 × .70) (2 × .70), or $20,000 (2 × .70)3 = $54,880.

Another way to calculate the total cost under the Cumulative Average-Time Learning Model is to find the
average cost per unit and multiply it by the number of units produced.

To find the average cost per unit using the Cumulative Average-Time Learning Model, multiply the cost for the
first unit by the learning curve rate raised to the appropriate exponent for the number of times doubling will
occur. For eight units, doubling will occur three times (from 1 unit to 2 units; from 2 units to 4 units; and from
4 units to 8 units). $20,000 × .73 = $6,860.

The total cost is the average cost of $6,860 × 8, which equals $54,880.

D.

This answer results from multiplying $20,000 by .7 and getting $14,000; then multiplying $14,000 by .7 and getting
$9,800; then multiplying $9,800 by .7 and getting $6,860; then summing $20,000 + $14,000 + $9,800 + $6,860 =
$50,660.

Under the Incremental Unit-Time Learning Model, $20,000 is the cost of the first unit; $14,000 is the cost of the
second unit; $9,800 is the cost of the fourth unit; and $6,860 is the cost of the eighth unit. This is incorrect for two
reasons: (1) the problem says to use the Cumulative Average-Time Learning Model, not the Incremental Unit-Time
Learning Model; and (2) this would not be correct even for the Incremental Unit-Time Learning Model, because only
the costs of the first, second, fourth and eighth units are included in the total.

(c) HOCK international, page 97


Part 1 : 07/07/14 10:28:01

Question 57 - CMA 1292 H2 - Budget Methodologies

A. In order to solve this problem we need to determine a total fixed cost and the variable cost per unit. The
total fixed costs are $200,000 ($100,000 each of manufacturing and selling costs). The total variable costs in
the 100,000 unit budget are $450,000 ($200,000 raw materials + 100,000 direct labor + $100,000 manufacturing
overhead + $50,000 selling/administrative expense). This gives a standard variable cost of $4.50 per unit.
Therefore, to produce 110,000 units the company will incur $495,000 in variable costs ($4.50 × 110,000 units)
plus $200,000 in fixed costs for a total of $695,000.

B. This answer assumes no fixed costs.

C. This is the variable cost of production only.

D. This is the cost at a production level of 100,000 units.

Question 58 - CMA 1294 3.19 - Budget Methodologies

A.

To solve this question, we use the formula for the physical flow of goods:

Beginning Inventory + Units Produced − Units Sold = Ending Inventory

Beginning inventory for the second quarter is 20% of the second quarter sales, or 1,600 units (8,000 × .20).
The ending inventory for the second quarter is 20% of the third quarter sales, or 2,400 units (12,000 × .20).
Plugging numbers into the formula we will get the following:

1,600 + Units Produced − 8,000 = 2,400

Solving for Units Produced, we get Units Produced = 8,800.

B.

This answer results from using the formula for the physical flow of inventory incorrectly.The correct formula is

Beginning Inventory + Units Produced − Units Sold = Ending Inventory

To get this answer, units sold are added and units produced are subtracted. Instead, the number of units produced
(the unknown) should be added to beginning inventory and the number of units sold should be subtracted.

C. This is the sales level for the second quarter. It does not take into consideration the beginning and ending
inventories. See the correct answer for a complete explanation.

D.

This answer results from calculating the beginning inventory as 2,000 units. The beginning inventory is budgeted as
20% of sales of 8,000 units, which is 1,600 units.

Question 59 - CMA 1289 5.22 - Probability

A. This answer is calculated without regard to the unsold pretzels that are discarded because they will be stale before
the next home game. Those pretzels are a cost that needs to be included in the calculation of any profit.

(c) HOCK international, page 98


Part 1 : 07/07/14 10:28:01

B. The correct answer is given. See the correct answer for a complete explanation.

C. This would be the profit if it were assumed that all 4,000 pretzels were sold. However, only 3,000 pretzels were sold.

D.

The meaning of the word "conditional" in "conditional profit" is similar to the meaning of the word
"conditional" in "conditional probability." The conditional probability of two events is the probability that the
second event will occur when it is known that the first event has already occurred. Conditional profit is
conditional because a certain amount of profit (or loss) is associated with each event, such as purchasing a
certain amount of inventory and selling a certain amount of inventory.

In this problem, the first event is the purchase of 4,000 pretzels. So given that we know that 4,000 pretzels
have been purchased, what is the profit from that course of action if demand is only 3,000 pretzels? In other
words, in this problem, there are actually two conditions that are known: (1) 4,000 pretzels are supplied, and
(2) demand is 3,000 pretzels. Since the amount supplied is given and the amount demanded is given, the
frequency distribution of the possible demand levels (sales volumes) for pretzels is irrelevant.

Since unsold pretzels are discarded, to calculate the profit we need to use the cost of all 4,000 pretzels that
will be purchased to sell, not only the cost of the 3,000 pretzels that are sold. The cost is $1,200 (4,000 × $.30).
The revenue from selling 3,000 pretzels is $3,000 (3,000 × $1). The difference between the revenue of $3,000
and the cost of $1,200 is the profit, which is $1,800.

Question 60 - CMA 1291 3.25 - Budget Methodologies

A. This is purchases valued at the sales price, not the cost of goods sold (70% of purchased price). See the correct
answer for a complete explanation.

B. In December the cash payments will be for 25% of the purchases made in December and for 75% of the
purchase made in November. December purchases are for the budgeted January sales and are equal to
$168,000 ($240,000 total January sales × 70% COGS). Thus, 25% of the December purchases is $42,000
($168,000 × 25%). November purchases were for the December sales and are equal to $322,000 ($460,000 total
December sales × 70% COGS). Thus, 75% of November purchases is $241,500 ($322,000 × 75%). Adding these
two cash payments together, we get a total cash payment in December of $283,500 ($42,000 + $241,500).

C. This is the amount of purchases in December (of which only 25% are actually paid in December). See the correct
answer for a complete explanation.

D. This answer is calculated based on credit sales only, and not total sales. See the correct answer for a complete
explanation.

Question 61 - ICMA 10.P1.010 - Planning and Budgeting Concepts

A. Standards should never be set to result in favorable variances. To do that renders the standard setting process
useless from the beginning. Standards need to be based on expected reality, considering all appropriate factors.

B. Discussions between relevant managers to determine the exact quantities required and the timing of those
purchases will help determine what price level would be most appropriate for the standard. This is the
participative method of standard setting, and it is more likely to be supported by the employees than an
authoritative method of standard setting.

(c) HOCK international, page 99


Part 1 : 07/07/14 10:28:01

C. Standards are set using either an authoritative process or a participative process. Standards based on anticipated
costs may be a starting point for discussion using either process, but there is no absolute standard that should be
used in this situation. Setting a standard in order to keep pressure on an employee or group of employees is not a
participative process. Furthermore, pressure to obtain the lowest price may encourage the purchase of excess
inventory, which creates increased inventory carrying costs.

D. Standards are set using either an authoritative process or a participative process. Standards based on historical
costs plus an expected price increase may be a starting point for discussion using either process, but there is no
absolute standard that should be used in this situation.

Question 62 - ICMA 10.P1.022 - Learning Curves

A.

The total number of direct labor hours required to produce 8 units using the Cumulative Average-Time
Learning Model is:

10,000 (2 × .80) (2 × .80) (2 × .80), or 10,000 (2 × .80)3, which equals 40,960 hours.

B. This is 80% of the total number of hours that would be required to produce 8 units if no learning took place. This is
not the way to calculate the cumulative direcdt labor hours required for producing a total of eight units using the
Cumulative Average-Time Learning Model.

C. This is the number of direct labor hours that would be required if no learning took place.

D.

This answer results from multiplying $10,000 by .8 and getting $8,000; then multiplying $8,000 by .8 and getting
$6,400; then multiplying $6,400 by .8 and getting $5,120; then summing $10,000 + $8,000 + $6,400 + $5,120 =
$29,520.

Under the Incremental Unit-Time Learning Model, $10,000 is the cost of the first unit; $8,000 is the cost of the second
unit; $6,400 is the cost of the fourth unit; and $5,120 is the cost of the eighth unit. This is incorrect for two reasons: (1)
the problem says to use the Cumulative Average-Time Learning Model, not the Incremental Unit-Time Learning
Model; and (2) this would not be correct even for the Incremental Unit-Time Learning Model, because only the costs of
the first, second, fourth and eighth units are included in the total.

Question 63 - ICMA 10.P1.033 - Risk, Uncertainty and Expected Value

A.

Costs incurred up to this point are irrelevant to the decision to be made, because they are costs that have
already been incurred (called "sunk" costs), and they cannot be changed. So the only cash flows that make a
difference in this decisiion are the expected future cash flows.

Action 1, Invest in the Joint Venture, has one potential cash inflow if the investment is successful and a
different potential cash inflow if the investment is not successful. The expected value of the cash inflow is the
weighted average of the two possible cash inflows, weighted according to their probabilities. So the expected
value of the cash inflows from investing is ($15,000,000 × .60) + ($2,000,000 × .40), which equals $9,800,000.
The amount of the investment is $9,500,000. Subtracting the amount of the investment from the expected
value of the cash inflows, we get an expected value of investing of $9,800,000 − $9,500,000, which equals

(c) HOCK international, page 100


Part 1 : 07/07/14 10:28:01

$300,000.

Action 2, Do Not Invest in the Joint Venture, has only one potential cash flow, a cash outflow. If the company
decides not to invest, it will have additional costs to pay of $100,000. There is no need to calculate a weighted
average here, because the probability is 100% that if the company decides not to invest, it will have to pay out
$100,000.

So the answer to the question is $300,000 and $(100,000).

B. Costs incurred up to this point are irrelevant to the decision to be made, because they are costs that have already
been incurred (called "sunk" costs), and they cannot be changed. So the only cash flows that make a difference in this
decisiion are the expected future cash flows.

C. Costs incurred up to this point are irrelevant to the decision to be made, because they are costs that have already
been incurred (called "sunk" costs), and they cannot be changed. So the only cash flows that make a difference in this
decisiion are the expected future cash flows.

D. Costs incurred up to this point are irrelevant to the decision to be made, because they are costs that have already
been incurred (called "sunk" costs), and they cannot be changed. So the only cash flows that make a difference in this
decisiion are the expected future cash flows.

Question 64 - ICMA 10.P1.054 - Budget Methodologies

A.

We need to (1) determine the overhead application rate to be used, (2) calculate the amount of overhead to be
applied to each unit of laminated putter heads, and then (3) use that along with the other information given on
direct materials and direct labor cost to calculate the total cost for one laminated putter head.

(1) Determine the overhead application rate: We will use a combined overhead application rate (variable and
fixed OH), since the problem does not give enough information to split it out. Since the overhead is to be
applied to both products, we must have production amounts for both products in order to determine the
application rate per hour of direct labor to be used in producing both products.

(1a) Calculate the number of units to be manufactured of each product: The inventory equation is Beginning
Inventory in Units + Manufactured Units − Sold Units = Ending Inventory in Units

Forged: 300 + Manufactured Units − 8,200 = 100. Manufactured Forged Units = 8,000.

Laminated: 60 + Manufactured Units − 2,000 = 60. Manufactured Laminated Units = 2,000.

(1b) Calculate the number of direct labor hours required for production of both products:

Forged: 8,000 units × 1/4 hour per unit = 2,000 hours.

Laminated: 2,000 units × 1 hour per unit = 2,000 hours.

Total number of direct labor hours: 2,000 + 2,000 = 4,000.

(2) Calculate the amount of overhead to be applied to each unit of Laminated:

Total overhead of $25,000 variable plus $15,000 fixed = $40,000. $40,000 divided by the 4,000 total number of
direct labor hours = $10 per direct labor hour.

(c) HOCK international, page 101


Part 1 : 07/07/14 10:28:01

Each unit of Laminated requires 1 hour of direct labor. Therefore, the amount of overhead to be applied to
each unit of Laminated will be $10 × 1, or $10.

(3) Calculate the total cost for one Laminated Putter Head:
Raw materials:
1 pound steel @ $5/pound $ 5
1 pound copper @ $15/pound 15
Direct labor:
1 hour @ $22/hour 22
Overhead:
1 direct labor hour @ $10/DLH 10
Total $52

B. This amount includes applied overhead in the amount of $4 per unit, which is total overhead of $40,000 divided by
the 10,000 total units to be produced. However, the problem indicates that overhead should be based on direct labor
hours used, and the two products do not require the same number of direct labor hours to manufacture. The number
of hours required to produce each unit provides the basis for overhead allocation.

C. This amount includes applied overhead in the amount of $20, which is total overhead of $40,000 divided by the
2,000 total units to be produced of Laminated Putter Heads. This means that all of the overhead is being applied to
the Laminated Putter Heads and none to the Forged Putter Heads. The total overhead needs to be allocated to both
products on the basis of direct labor hours used by each product.

D. This is the material and labor cost for a laminated putter, but it does not include any applied overhead.

Question 65 - CMA 694 3.22 - Manufacturing Input Variances - Materials and Labor

A. If the standard labor rate was set using a single average standard rate, it may cause a labor rate variance. If the
rate doesn't reflect the proportion of hours worked of each wage rate group of workers, a variance will result.

B. The assignment of different skill levels of workers than was planned in most cases will cause a labor price
variances. The labor rate variance is calculated as: (Actual Rate − Standard Rate) × Actual Hours. When there is a
difference between the rates of assigned (with lower or higher working skills) and planned workers, a labor rate
variance will most likely occur.

C. The labor price or rate variance is calculated as: (Actual Rate − Standard Rate) × Actual Hours. The only
figure union contracts can influence is the actual labor rate. The standard rate is set by the budget at the
beginning of the year. As union contracts are approved before the budgeting cycle begins, the information
about potential changes in wages and salaries is already included in the budget and standards. Therefore, a
union contract approved before the budgeting cycle cannot be the cause of a labor rate variance.

D. As with all future predictions, labor rate predictions can have inaccuracies. If the standard labor rate was set using
these predicted numbers, it may cause a labor rate variance.

Question 66 - ICMA 10.P1.097 - Manufacturing Input Variances - Materials and Labor

A. When circumstances change, standards should be adjusted to reflect the changed situation. In this case,
there is nothing the company can do about the price increase because it has only one available supplier.
Therefore, the company has no choice but to pay the higher price and it should adjust its standards
accordingly.

(c) HOCK international, page 102


Part 1 : 07/07/14 10:28:01

B. The production manager has no control over the price that the company's sole supplier charges for the raw
material. The company cannot change suppliers, because there is no other supplier to change to; and the production
manager would not have any control over the supplier used even if the company could change suppliers. Therefore,
the production manager should not be held responsible for the price variance.

C. The purchasing manager has no control over the price that the company's sole supplier charges for the raw
material. The company cannot change suppliers, because there is no other supplier to change to. Therefore, the
purchasing manager should not be held responsible for the price variance.

D. The material usage standard should not be adjusted to compensate for changes in raw material costs. To do so
would compromise the quality of the product.

Question 67 - CMA 1291 3.3 - Manufacturing Input Variances - Materials and Labor

A. The labor rate/price variance is: (Actual Rate − Standard Rate) × Actual Hours. The actual direct labor cost
was $37,800 ($42,000 × 90%) for the month of November. The actual rate was $7.56 ($37,800 ÷ 5,000). The
actual number of hours was 5,000. The labor rate/price variance was ($7.56 − $8.00) × 5,000 = $(2,200)
favorable. A negative variance for a cost is a favorable variance.

B. The labor rate/price variance is favorable as the actual direct labor rate is lower than the standard. See the correct
answer for a complete explanation.

C. This answer results from using the total factory wages of $42,000 to calculate the direct labor actual rate. However,
only 90% of the total factory wages were direct labor. See the correct answer for a complete explanation.

D. The labor rate/price variance is calculated as: (Actual Rate − Actual Rate) × Actual Hours. This answer results from
using the hours allowed for the actual output (19,000 × .25) instead of the actual hours. See the correct answer for a
complete explanation.

Question 68 - ICMA 10.P1.085 - Variance Analysis Concepts

A. Variances are not more difficult to compute when a static budget rather than when a flexible budget is used to
compare actual to budgeted amounts.

B. Variances would probably tend to be larger when actual is compared to the static budget, because the variances
include variances due to variations between budgeted activity levels and actual activity levels. When actual amounts
are compared with flexible budget amounts, the variances will not include variances caused by variations between
actual and budgeted activity, whereas when the static budget is used, the variances will include variances caused by
variations between actual and budgeted activity.

C. A static budget is not more difficult to develop than a flexible budget. A flexible budget is based on the static
budget, but sales and production activity assumptions used to develop the static budget are changed to actual sales
and production activity for the flexible budget, and the budgeted amounts that vary with activity are adjusted
accordingly in the flexible budget.

D.

A static budget is developed for one specific activity (sales or production) level. When variance reports
comparing actual results to budgeted results in the static budget are prepared and causes for the variances
are reported, one of the causes will always be that actual volume was different from planned sales volume,
because actual activity will never be exactly equal to the budgeted activity.

(c) HOCK international, page 103


Part 1 : 07/07/14 10:28:01

Since variances due to volume variations are expected, it does not make much sense to continue reporting
them on the variance report as causes of variances. It is more important to focus on variances caused by
other factors.

For example, a variance caused by an increase in the cost of direct labor above what is expected for the
actual production could signal a problem in production and should be investigated. But an increase in the
cost of direct labor above what is expected for the budgeted production that is caused by increased
production is not a production problem, if the cost of the direct labor per unit is equal to the budgeted amount
per unit for the number of units actually produced.

The use of a static budget can create difficulty in isolating the causes of variances that need to be
investigated from those that need no investigation.

Question 69 - CIA 594 III.72 - Manufacturing Input Variances - Materials and Labor

A. This is the materials efficiency variance. However, the question asks for the materials price variance. See the
correct answer for a complete explanation.

B. This is the direct labor rate variance. However, the question asks for the direct materials price variance. See the
correct answer for a complete explanation.

C. The variance is unfavorable because the actual price ($28) is higher than standard ($24). See the correct answer
for a complete explanation.

D. The direct materials price variance is calculated as follows: (Actual Price − Standard Price) × Actual
Quantity. All the components of the formula are in the data given. The actual price is $28. The standard price
is $24. The actual quantity is 190,000. Therefore, the direct materials price variance is ($28 − $24) × 190,000 =
$760,000 unfavorable. Because the actual price was higher than the standard, the variance is unfavorable.

Question 70 - CMA 687 4.18 - Manufacturing Input Variances - Materials and Labor

A. This is calculated using the budgeted amount of units of production (5,000) instead of the actual units production
(4,500).

B. Since indirect labor is treated as a variable cost, the unit labor cost is $2.40 ($144,000 ÷ (5,000 × 12)). The
flexible budget amount of variable overhead (indirect labor) therefore equals $10,800 ($2.40 × 4,500). The
flexible budget variance is equal to the actual amount of variable overhead minus the flexible budget amount
of variable overhead. This gives us a favorable variance of $700 ($10,100 − $10,800).

C. The variance is favorable as the actual amount of indirect labor is less than the flexible budget amount.

D. The variance is favorable as the actual amount of indirect labor is less than flexible budget amount.

Question 71 - CIA 1191 IV.16 - Manufacturing Input Variances - Materials and Labor

A.

The purchase price variance is calculated as follows: (Actual Price − Standard Price) × Actual Quantity. Since

(c) HOCK international, page 104


Part 1 : 07/07/14 10:28:01

the purchase price variance is required, the price to use for the actual price is the price per unit of the units
purchased instead of the price per unit of the units used; and the actual quantity is the number of units
purchased, not the number of units that were put into production. The actual price is $3.96 per pound
($297,000 ÷ 75,000). The standard price is $4.00 per pound. The actual quantity purchased is 75,000 pounds.
The purchase price variance is ($3.96 − $4.00) × 75,000 = $(3,000) favorable.

The quantity variance is calculated as follows: (Actual Quantity − Standard Quantity for Actual Output ) ×
Standard Price. The actual quantity is the actual quantity used, not the quantity purchased. The standard
quantity is the standard quantity allowed for the actual output. The actual quantity is 70,000. The standard
quantity allowed for the actual level of output is 69,000 pounds (3 lb.× 23,000 units). The standard price is $4.
Hence, the quantity variance is (70,000 − 69,000) × $4 = $4,000 unfavorable.

B.

The price variance is calculated as follows: (Actual Price − Standard Price) × Actual Quantity. Since the purchase
price variance is required, the price to use for the actual price is the price per unit of the units purchased instead of the
price per unit of the units used; and the actual quantity is the number of units purchased. This price variance is
incorrect because the figure used in the formula is not the actual quantity of units purchased but is instead the
standard quantity for the planned production (26,000 units x 3 pounds per unit).

The quantity variance is calculated as follows: (Actual Quantity − Standard Quantity for Actual Output ) × Standard
Price. The actual quantity is the actual quantity used, not the quantity purchased. The standard quantity is the
standard quantity allowed for the actual output. This quantity variance is incorrect because it uses the standard
quantity allowed for the planned production (26,000 × 3 pounds) instead of the standard quantity allowed for the
actual production.

C. The purchase price variance is incorrect because it is calculated using the quantity used in production instead of
the quantity purchased. When a question asks for the purchase price variance, the actual quantity to use is the
quantity purchased, and the actual price is the actual price per unit for the quantity purchased.

D.

The quantity variance is incorrect because it is calculated as (the actual quantity purchased of 75,000 minus the actual
quantity used of 70,000) multiplied by the standard price per unit of $4.

The correct formula to use for the quantity variance is (Actual Quantity − Standard Quantity for Actual Output ) ×
Standard Price. The actual quantity is the actual quantity used, not the quantity purchased. The standard quantity is
the standard quantity allowed for the actual output.

Question 72 - CMA 692 3.21 - Manufacturing Input Variances - Materials and Labor

A. The actual hours exceed the standard amount, which means the variance is unfavorable. See the correct answer
for a complete explanation.

B. The actual hours exceed the standard amount, which means the variance is unfavorable. See the correct answer
for a complete explanation.

C. The labor efficiency variance is calculated as: (Actual Hours − Standard Hours for Actual Output) × Standard Rate.
This answer results from multiplying by the actual rate ($11.70) instead of the standard rate.

D. The direct labor efficiency variance is calculated as: (Actual Hours − Standard Hours for Actual Output) ×
Standard Rate. Actual hours is 28,000. The standard hours allowed for the actual level of output is 27,500
hours (1.25 hours per unit × 22,000 units produced). The standard labor rate is $12. Therefore, the direct labor
efficiency variance is (28,000 − 27,500) × $12 = $6,000 unfavorable. The variance is unfavorable because the

(c) HOCK international, page 105


Part 1 : 07/07/14 10:28:01

actual hours exceed the standard hours, and this is a cost variance.

Question 73 - CMA 692 3.18 - Manufacturing Input Variances - Materials and Labor

A. The price variance is (AP − SP) × AQ. The actual price is $3.80 per pound ($475,000 ÷ 125,000). The
standard price is $3.60 per pound. The actual quantity purchased is 125,000 pounds. Note that we use the
actual quantity purchased in the formula because we need to determine the purchase price variance. The
purchase price variance is ($3.80 − $3.60) × 125,000 = $25,000 unfavorable. Since the actual purchase price
was higher than the standard price, the variance is positive. A positive variance is unfavorable for a cost item
because it means the actual cost was greater than the planned cost.

B. The formula for the price variance is (AP − SP) × AQ. The actual price is $3.80 per pound. ($475,000 ÷ 125,000).
The standard price is $3.60 per pound. This answer results from using the quantity consumed by production (108,000
lb.) as the Actual Quantity (AQ) in the formula. However, the question asks for the purchase price variance.
Therefore, we need to use the actual quantity purchased in the formula (125,000 lb.), not the quantity consumed by
production.

C. The formula for the price variance is (AP − SP) × AQ. The actual price is $3.80 per pound. ($475,000 ÷ 125,000).
The standard price is $3.60 per pound. This answer results from using the quantity consumed by production (108,000
lb.) as the Actual Quantity (AQ) in the formula. However, the question asks for the purchase price variance.
Therefore, we need to use the actual quantity purchased in the formula (125,000 lb.), not the quantity consumed by
production. Furthermore, the variance is positive. A positive variance is unfavorable for a cost item because it means
the actual cost was greater than the planned cost.

D. The variance is unfavorable because the actual price – $3.80 per pound ($475,000 ÷ 125,000) – was greater than
the standard price of $3.60 per pound.

Question 74 - ICMA 10.P1.102 - Manufacturing Input Variances - Materials and Labor

A. This is the materials price variance, calculated using the amount of direct materials used during the period.
However, the question asks for the materials purchase price variance. The materials purchase price variance is
calculated using the quantity of materials purchased during the period, not the quantity of materials used.

B. This is the materials quantity variance, not the materials purchase price variance.

C.

The formula for the price variance is (AP – SP) × AQ. Because this is asking for the purchase price variance,
we use the actual quantity purchased, not the actual quantity used, for AQ.

The actual price is $583,200 ÷ 108,000 = $5.40 per unit of raw material.
The standard price is $16.50 per completed product unit ÷ 3 units of raw material used per completed product
unit = $5.50
The actual quantity purchased is 108,000.

The purchase price variance = ($5.40 – $5.50) × 108,000 = $(10,800) favorable.

D. The variance is unfavorable, not favorable.

(c) HOCK international, page 106


Part 1 : 07/07/14 10:28:01

Question 75 - CMA 1295 3.25 - Manufacturing Input Variances - Materials and Labor

A. The variable overhead spending variance is related to the difference between the actual variable overhead cost per
unit (this is calculated as the actual overhead costs divided by the actual usage of the allocation base) and the
standard application rate. It is the difference between the actual amount of variable overhead incurred and the
standard amount of variable overhead allowed for the actual quantity of the variable overhead allocation base used for
the actual output produced. The production control supervisor does not control this.

B. The material usage variance is the difference between the actual material usage and the standard usage for
this level of output, multiplied by the standard material price. This variance occurs during the production
process and is therefore most controllable by the production control supervisor. There are a number of
reasons that could cause this variance: poor production employees' performance, product design, waste,
theft, and poor material quality, etc.

C. The purchasing department is the most responsible for the material price variance, not the production control
supervisor.

D. The fixed overhead budget variance cannot be controlled by the production control supervisor.

Question 76 - CMA 692 3.20 - Manufacturing Input Variances - Materials and Labor

A. Since the actual labor rate of $11.70 is lower than the standard labor rate of $12, the variance is favorable. See the
correct answer for a complete explanation.

B. This result is the direct labor efficiency variance, but this question asks for the direct labor rate variance. See the
correct answer for a complete explanation.

C. The labor price/rate variance is calculated as: (Actual Rate − Standard Rate) × Actual Hours. The actual
total direct labor cost is $327,600 ($364,000 × 90%), and the actual labor rate is $11.70 ($327,600 ÷ 28,000
hours used). The standard labor rate is $12. The actual hours used is 28,000. Therefore, the labor rate
variance is ($11.70 − $12.00) × 28,000 = ($8,400) favorable. Since the actual labor rate of $11.70 is lower than
the standard labor rate of $12, the variance is favorable.

D. Since the actual labor rate of $11.70 is lower than the standard labor rate of $12, the variance is favorable. See the
correct answer for a complete explanation.

Question 77 - CIA 594 III.73 - Manufacturing Input Variances - Materials and Labor

A. The direct materials efficiency variance is favorable because the actual quantity used in production is less than the
standard quantity allowed for the actual output. See the correct answer for a complete explanation.

B. The quantity variance (also called the efficiency or usage variance) is calculated as: (Actual Quantity −
Standard Quantity for Actual Output) × Standard Price. The actual quantity is 190,000. The standard quantity
allowed for the output of 20,000 units of product is 200,000 lb. (20,000 × 10 lb. standard quantity of material
per unit of finished product). The standard price is $24. The direct materials efficiency variance is (190,000 −
200,000) × $24 = $(240,000) favorable. The variance is favorable because the actual quantity used in
production is less than the standard quantity allowed for the actual output.

C. This is the direct labor rate variance. The question asks for the direct materials efficiency variance. See the correct
answer for a complete explanation.

(c) HOCK international, page 107


Part 1 : 07/07/14 10:28:01

D. This is the direct materials price variance. However, the question asks for the direct materials efficiency variance.
See the correct answer for a complete explanation.

Question 78 - CIA 593 IV.14 - Sales and Market Variances

A. There is no difference between the flexible budget and the actual sales volume, as the actual sales volume is used
in calculating the flexible budget figures because the flexible budget uses the actual sales level.

B. The sales volume variance formula is: (Actual Sales Volume − Budgeted Sales Volume) × Budgeted Contribution
per Unit. As we can see from the formula, the budgeted unit contribution margin is used in the calculation, not the
actual contribution margin.

C. The sales volume variance formula is: (Actual Sales Volume − Budgeted Sales Volume) × Budgeted
Contribution per Unit. The budgeted contribution per unit is defined at the beginning of the year using the
master budget. The budgeted sales volume is also defined at the beginning of the year and it is a master
budget figure. Actual sales volume is the same thing as the flexible budget sales volume, because the flexible
budget uses the actual level of output. Thus, the sales volume variance is the difference between the flexible
budget and master budget sales volume, times master budget unit contribution margin.

D. The sales volume variance basic formula is: (Actual Sales Volume − Budgeted Sales Volume) × Budgeted
Contribution per Unit. As we can see from the formula, the budgeted unit contribution margin is used in the calculation,
not the actual unit contribution margin. The actual sales volume and the flexible budget sales volume are the same
amount, because the flexible budget uses the actual level of output.

Question 79 - CIA 582 IV.22 - Manufacturing Input Variances - Materials and Labor

A. An unfavorable materials quantity (usage) variance means that more materials were consumed by
production than was scheduled by the standard. This can happen due to the number of reasons: poor worker
performance, spoilage, shrinkage, theft, design of the product, poor quality of materials, machine downtime,
etc. Labor with skills equal to those required by the standards is most likely not a cause of a materials
efficiency variance, because worker performance should be adequate.

B. An unfavorable materials quantity (usage) variance means that more materials were consumed by production than
was scheduled by the standard. This can happen due to the number of reasons: poor worker performance, spoilage,
shrinkage, theft, design of the product, poor quality of materials, machine downtime, etc. Thus, materials that do not
meet specifications may be the cause of a materials efficiency variance.

C. An unfavorable materials quantity (usage) variance means that more materials were consumed by production than
was scheduled by the standard. This can happen due to the number of reasons: poor worker performance, spoilage,
shrinkage, theft, design of the product, poor quality of materials, machine downtime, etc. Substantial overtime may
affect the performance of workers and be the cause of a materials efficiency variance.

D. An unfavorable materials quantity (usage) variance means that more materials were consumed by production than
was scheduled by the standard. This can happen due to the number of reasons: poor worker performance, spoilage,
shrinkage, theft, design of the product, poor quality of materials, machine downtime, etc. Thus, machinery that has not
been maintained properly may be the cause of a materials efficiency variance.

Question 80 - CMA 1295 3.8 - Manufacturing Input Variances - Materials and Labor

(c) HOCK international, page 108


Part 1 : 07/07/14 10:28:01

A. The total material quantity (efficiency) and labor efficiency variances can be broken down into the two
subvariances, but the spending variance is not one of them.

B. The total material quantity (efficiency) and labor efficiency variances can be broken down into the two
subvariances, but the volume variance is not one of them.

C. The total material quantity (efficiency) and labor efficiency variances can be broken down into two
subvariances: the mix and the yield variances. The mix variance is the part of the quantity variance that
results because the mix of material actually used was different from the mix that was supposed to have been
used. (For example, including more corn and less wheat in the cereal than the standard called for). The yield
variance results from the difference between the total quantity of the inputs that were actually used to
produce the actual output and the total standard quantity that should have been used to produce the actual
output.

D. The total material quantity (efficiency) and labor efficiency variances can be broken down into the two
subvariances, but the price variance is not one of them.

Question 81 - CIA 1192 IV.20 - Manufacturing Input Variances - Materials and Labor

A. This is the material price variance, not the purchase price variance. To calculate the purchase price variance we
need to use the purchased quantity of material (25,000), not the quantity consumed by production (23,000).

B. This result is calculated using the standard quantity of materials, 24,000 pounds (3 lb. per unit of finished product
and 8,000 units budgeted for the production). However, to calculate the purchase price variance, we need to use the
purchased quantity of material (25,000 pounds). See the correct answer for a complete explanation.

C. The purchase price variance is calculated as follows: (AP − SP) × AQ, where AQ is the actual quantity
purchased. The actual price for the pound of material is $1.55 ($38,750 ÷ 25,000). The standard price is $1.60.
The quantity purchased is 25,000. The direct materials purchase price variance is ($1.55 − $1.60) × 25,000 =
$(1,250) favorable. The variance is favorable because the actual price the materials were purchased for is less
than the standard price.

D. This result is calculated using the quantity of materials allowed for the production, 22,500 pounds (3 lb. Per unit of
finished product × 7,500 units produced). However, to calculate the purchase price variance, we need to use the
purchased quantity of material (25,000 pounds). See the correct answer for a complete explanation.

Question 82 - CMA 1290 3.7 - Manufacturing Input Variances - Overhead

A.

This is not the correct answer. Please see the correct answer for a complete explanation.

We have been unable to determine how to calculate this incorrect answer choice. If you have calculated it, please let
us know how you did it so we can create a full explanation of why this answer choice is incorrect. Please send us an
email at support@hockinternational.com. Include the full Question ID number and the actual incorrect answer choice
-- not its letter, because that can change with every study session created. The Question ID number appears in the
upper right corner of the ExamSuccess screen. Thank you in advance for helping us to make your HOCK study
materials better.

B.

This is not the correct answer. Please see the correct answer for a complete explanation.

(c) HOCK international, page 109


Part 1 : 07/07/14 10:28:01

We have been unable to determine how to calculate this incorrect answer choice. If you have calculated it, please let
us know how you did it so we can create a full explanation of why this answer choice is incorrect. Please send us an
email at support@hockinternational.com. Include the full Question ID number and the actual incorrect answer choice
-- not its letter, because that can change with every study session created. The Question ID number appears in the
upper right corner of the ExamSuccess screen. Thank you in advance for helping us to make your HOCK study
materials better.

C.

This is not the correct answer. Please see the correct answer for a complete explanation.

We have been unable to determine how to calculate this incorrect answer choice. If you have calculated it, please let
us know how you did it so we can create a full explanation of why this answer choice is incorrect. Please send us an
email at support@hockinternational.com. Include the full Question ID number and the actual incorrect answer choice
-- not its letter, because that can change with every study session created.The Question ID number appears in the
upper right corner of the ExamSuccess screen. Thank you in advance for helping us to make your HOCK study
materials better.

D.

The variable overhead spending variance is the difference between the actual variable overhead cost per unit
of the allocation base actually used (the actual overhead costs divided by the actual usage of the allocation
base) and the standard variable overhead application rate, multiplied by the actual quantity used of the
application base. (Actual Cost per Hour [MH or DLH] Actually Used − Standard Application Rate) × Actual
Quantity [MH or DLH] Used.)

This variance is also the difference between the actual amount of variable overhead incurred and the
standard amount of variable overhead allowed for the actual quantity of the variable overhead allocation base
used for the output produced.

To use the formula — (Actual Cost per Hour [MH or DLH] Actually Used − Standard Application Rate) × Actual
Quantity [MH or DLH] Used — we need to find the actual application rate and the standard application rate.
The actual quantity of the application base (direct labor hours) used is given in the problem as 440,000.

Actual Cost per Hour Actually Used: The actual variable overhead incurred was $352,000. The actual quantity
of direct labor hours used was 440,000. Therefore, the actual cost per hour actually used was $352,000 ÷
440,000 = $.80.

Standard Application Rate: This one is a little more complicated. Total budgeted overhead is given in the
problem as $900,000, but that includes both fixed and variable overhead. We need to figure out what the
budgeted variable overhead was. We have enough information to figure out what budgeted fixed overhead
was, so we can calculate budgeted fixed overhead and subtract it from budgeted total overhead to find
budgeted variable overhead. The fixed overhead application rate is given in the problem as $3.00 per unit, and
the budgeted production is given as 200,000 units. Therefore, total budgeted fixed overhead must have been
$3.00 × 200,000 units, which is equal to $600,000.

Since total budgeted overhead was $900,000 and total budgeted fixed overhead was $600,000, total budgeted
variable overhead must have been $900,000 − $600,000, or $300,000. Since production was budgeted to be
200,000 units, standard (budgeted) variable overhead per unit produced was $300,000 ÷ 200,000, which is
$1.50 per unit. Overhead is applied on the basis of direct labor hours, and 2 direct labor hours are allowed for
each unit produced. So the standard variable overhead rate per direct labor hour allowed for production is
$1.50 ÷ 2, which is $.75.

Actual quantity of application base used: This is given as 440,000 direct labor hours.

The variable overhead spending variance is therefore:

(c) HOCK international, page 110


Part 1 : 07/07/14 10:28:01

(.80 − .75) × 440,000 = 22,000

Since overhead is a cost, a positive variance is an Unfavorable variance. We said that the variable overhead
spending variance is also the difference between the actual amount of variable overhead incurred and the
standard amount of variable overhead allowed for the actual quantity of the variable overhead allocation base
used for the output produced. Actual variable overhead incurred is given in the problem as $352,000. The
standard amount of variable overhead allowed for the actual quantity of the variable overhead allocation base
used for the output produced is the standard variable overhead application rate of $.75 (calculated above)
multiplied by the actual direct labor hours used, which is 440,000. The result, $330,000, is subtracted from
$352,000 to calculate the variance: $22,000 Unfavorable.

Question 83 - CIA 597 3.18 - Manufacturing Input Variances - Materials and Labor

A. An inadequately trained and supervised labor force will have more material waste and spoilage than an adequately
trained and supervised labor force.

B. Poorly functioning machines will have more material waste and spoilage.

C. Rush orders disrupt the manufacturing process by interfering with normal work routines, practices, and procedures.
These disruptions will adversely affect each of the manufacturing processes, including the efficient use of material,
labor, and overhead.

D. Producing more units than planned in the master budget will not affect the quantity of the materials used
for each unit.

Question 84 - CMA 692 3.17 - Manufacturing Input Variances - Materials and Labor

A.

The labor efficiency variance is calculated as follows: (Actual Hours − Standard Hours for Actual Output) × Standard
Rate. There are two mistakes in this variance calculation.

One, the standard hours for the actual output was used in place of the actual hours. And two, the number of hours in
the fixed budget was used in place of the standard hours for the actual output. See the correct answer for a complete
explanation.

B. The labor efficiency variance is calculated as follows: (Actual Hours − Standard Hours for Actual Output) ×
Standard Rate. Actual hours are given as 192,000. The standard hours for the actual output are 190,000 (20
hours per one article × 9,500 articles that were actually reviewed and edited). The standard rate is $1,000 per
article ($10,000,000 ÷ 10,000) and $50 per labor hour ($1,000 ÷ 20). Putting all of this into the formula, the labor
efficiency variance is (192,000 − 190,000) × $50 = $100,000 unfavorable.

C. This is the actual labor cost of $9,120,000 minus the flexible budget labor cost of $9,500,000 (9,500 articles
actually reviewed and edited at $1,000 standard cost per article). So this is the total flexible budget variance for labor.
The question asks for only the labor efficiency variance, which is one part of the total flexible budget variance.

D. This is the difference between the flexible budget standard labor costs (variable costs) and total actual costs
(including fixed and variable costs), which does not mean anything.

(c) HOCK international, page 111


Part 1 : 07/07/14 10:28:01

Question 85 - CMA 1291 3.14 - Sales and Market Variances

A. This is the sales price variance. See the correct answer for a complete explanation.

B. This answer is the total sales variance, which includes variances caused by differences in both the sales price and
in the quantity sold. The total sales variance on the contribution margin is the difference between actual and budgeted
amount of contribution margin. The sales volume/quantity variance is calculated as follows: (Actual Sales Volume −
Budgeted Sales Volume) × Budgeted Contribution per Unit. See the correct answer for a complete explanation.

C. This question is asking for the sales volume/quantity variance on the contribution margin that is calculated
as follows: (Actual Sales Volume − Budgeted Sales Volume) × Standard Contribution per Unit. The total
budgeted contribution margin was $120,000, which gives us a $20 contribution margin per unit ($120,000 ÷
6,000). Now we can calculate the sales volume variance: (5,000 − 6,000) × $20 = ($20,000) unfavorable. The
actual sales volume was lower than budgeted, and that caused the negative impact of $20,000 on the
contribution margin.

D. This answer results from using the actual unit contribution margin instead of the budgeted unit contribution margin.
See the correct answer for a complete explanation.

Question 86 - ICMA 10.P1.118 - Manufacturing Input Variances - Materials and Labor

A. New production equipment can explain an unfavorable material efficiency (quantity) variance, because workers
working on new equipment experience a learning curve. They could make mistakes that could cause their work,
including the direct materials used, to have to be discarded; and more of this could occur than normal.

B. Inferior materials in the production process could explain a significant unfavorable material efficiency (quantity)
variance, because an increased amount of the materials would be defective and thus unusable.

C. This, by itself, would not be likely to explain an unfavorable material efficiency (quantity) variance. There is
nothing about the production level that should unfavorably impact the amount of direct materials used for
each unit produced. However, inferior materials, less-skilled workers and workers learning to use new
production equipment could explain an unfavorable materials quantity variance.

D. Less-skilled workers can explain an unfavorable material efficiency (quantity) variance, because less-skilled
workers make more mistakes and can cause more of the work to have to be discarded, including the direct materials
that have gone into the discarded work.

Question 87 - CMA 694 3.19 - Introduction to Variance Analysis and Standard Costs

A. Industrial engineers are involved in establishing standard costs for evaluation purposes.

B. Top management is primarily involved in formulating strategy plans and budgets. Establishing standard
costs for evaluation purposes is a task of management of a lower level. These standards are used to estimate
what costs should be under normal conditions of operations. Industrial engineers, budgetary accountants,
quality control personnel and employees who will be evaluated using these criteria are involved in the
process.

C. Budgetary accountants are involved in establishing standard costs for evaluation purposes.

D. Quality control personnel are involved in establishing standard costs for evaluation purposes.

(c) HOCK international, page 112


Part 1 : 07/07/14 10:28:01

Question 88 - ICMA 10.P1.100 - Variance Analysis Concepts

A.

The company's policy is to investigate any variance that is greater than $1,000 or 10% of budget, whichever is
larger. In other words, if 10% of the budget amount is greater than $1,000, they are to investigate any variance
greater than 10%. Or, if $1,000 is greater than 10% of the budget amount, they are to investigate any variance
greater than $1,000.

The raw material budget amount is $100,000, so 10% of that is $10,000. $10,000 is greater than $1,000.
Therefore, they are to investigate any variance that is greater than $10,000. Actual raw material was $89,000,
which is $11,000 less than the budgeted amount. $11,000 is greater than $10,000, so they are to investigate
that variance.

The direct labor budget amount is $50,000, so 10% of that is $5,000. $5,000 is greater than $1,000. Therefore,
they are to investigate any variance that is greater than $5,000. Actual direct labor was $54,000, which is
$4,000 greater than the budgeted amount. $4,000 is less than $5,000. Therefore, they do not need to
investigate that variance.

So they will investigate the raw material variance only.

B.

The company's policy is to investigate any variance that is greater than $1,000 or 10% of budget, whichever is larger.
In other words, if 10% of the budget amount is greater than $1,000, they are to investigate any variance greater than
10%. Or, if $1,000 is greater than 10% of the budget amount, they are to investigate any variance greater than $1,000.

The raw material budget amount is $100,000, so 10% of that is $10,000. $10,000 is greater than $1,000. Therefore,
they are to investigate any variance that is greater than $10,000. The direct labor budget amount is $50,000, so 10%
of that is $5,000. $5,000 is greater than $1,000. Therefore, they are to investigate any variance that is greater than
$5,000.

Investigating neither variance would not conform to this policy.

C.

The company's policy is to investigate any variance that is greater than $1,000 or 10% of budget, whichever is larger.
In other words, if 10% of the budget amount is greater than $1,000, they are to investigate any variance greater than
10%. Or, if $1,000 is greater than 10% of the budget amount, they are to investigate any variance greater than $1,000.

The raw material budget amount is $100,000, so 10% of that is $10,000. $10,000 is greater than $1,000. Therefore,
they are to investigate any variance that is greater than $10,000. The direct labor budget amount is $50,000, so 10%
of that is $5,000. $5,000 is greater than $1,000. Therefore, they are to investigate any variance that is greater than
$5,000.

Investigating both the material variance and the labor variance would not conform to this policy.

D.

The company's policy is to investigate any variance that is greater than $1,000 or 10% of budget, whichever is larger.
In other words, if 10% of the budget amount is greater than $1,000, they are to investigate any variance greater than
10%. Or, if $1,000 is greater than 10% of the budget amount, they are to investigate any variance greater than $1,000.

The raw material budget amount is $100,000, so 10% of that is $10,000. $10,000 is greater than $1,000. Therefore,
they are to investigate any variance that is greater than $10,000. The direct labor budget amount is $50,000, so 10%
of that is $5,000. $5,000 is greater than $1,000. Therefore, they are to investigate any variance that is greater than

(c) HOCK international, page 113


Part 1 : 07/07/14 10:28:01

$5,000.

Investigating the labor variance only would not conform to this policy, because the labor variance is only $4,000.

Question 89 - CMA 692 3.16 - Manufacturing Input Variances - Overhead

A. The variance is unfavorable. This answer results from subtracting actual fixed overhead incurred from budgeted
fixed overhead instead of the reverse. See the correct answer for a complete explanation.

B. The fixed overhead spending variance is the actual fixed overhead incurred minus the budgeted fixed
overhead costs. Actual fixed overhead costs are $618,000 ($9,738,000 total cost incurred − $9,120,000 flexible
(variable) costs). The budgeted fixed overhead costs are $600,000. The fixed overhead spending variance is
$18,000 unfavorable ($618,000 actual fixed costs − $600,000 budgeted fixed costs). The actual costs exceed
the budgeted costs, so the variance is unfavorable.

C. The fixed overhead spending variance is the actual fixed overhead incurred minus the budgeted fixed overhead
costs. This incorrect result is the consequence of the wrong assumption that the budgeted amount of fixed overhead
should be adjusted to the level of production the way variable costs are in the flexible budget. In the calculation, we
have to use $600,000 of budgeted fixed costs, not $570,000 adjusted for the level of production of 9,500 articles.
Again, fixed costs do not vary with the level of output.

D. The variance is unfavorable. See the correct answer for a complete explanation.

Question 90 - CMA 1292 3.17 - Manufacturing Input Variances - Overhead

A.

This answer results from calculating the amount of variable overhead applied by multiplying the predetermined rate by
the amount of machine hours planned to be used for the planned output. However, under standard costing -- which
is being used here because the problem tells us that overhead is applied based on planned machine hours --
overhead to be applied is calculated by multiplying the predetermined rate by the amount of (in this case) machine
hours that should have been used for the amount actually produced.

B.

This answer results from two errors:

(1) Calculating the amount of variable overhead applied by multiplying the predetermined rate by the amount of
machine hours planned to be used for the planned output. However, under standard costing -- which is being used
here because the problem tells us that overhead is applied based on planned machine hours -- overhead to be
applied is calculated by multiplying the predetermined rate by the amount of (in this case) machine hours that should
have been used for the amount actually produced.

(2) Reversing or misinterpreting the calculation of the variance amount. The variance is the actual variable overhead
incurred minus the variable overhead applied. Because overhead is a cost, a positive result is an unfavorable
variance, and the variable overhead was underapplied (less was applied than was incurred). A negative result is a
favorable variance, and the variable overhead was overapplied (more was applied than was incurred).

C.

This answer results from applying the variable overhead using normal costing, which is incorrect. The problem says
that overhead is applied based on planned machine hours, which means standard costing is being used.

(c) HOCK international, page 114


Part 1 : 07/07/14 10:28:01

Under standard costing, overhead to be applied is calculated by multiplying the predetermined rate by the amount of
(in this case) machine hours that should have been used for the amount actually produced.

Under normal costing, overhead to be applied is calculated by multiplying the predetermined rate by the amount of the
allocation base that was actually used for the amount actually produced.

D.

Nanjones applies overhead based on planned machine hours using a predetermined annual rate. The amount
of planned variable manufacturing overhead was $2,400,000 and amount of planned machine hours were
240,000. Thus, the application rate for variable manufacturing overhead was $10 per hour ($2,400,000 /
240,000).

Under standard costing -- which is being used here because the problem tells us that overhead is applied
based on planned machine hours -- overhead to be applied is calculated by multiplying the predetermined
rate by the amount of (in this case) machine hours that should have been used for the amount actually
produced. The problem tells us that the planned machine hours based on output was 21,000, and therefore,
the amount of variable overhead applied was $10 × 21,000, or $210,000. The actual variable overhead incurred
was $214,000. Therefore, variable manufacturing overhead was underapplied by $4,000.

Question 91 - CMA 696 3.25 - Manufacturing Input Variances - Overhead

A.

The flexible budget overhead variance equals the difference between the total actual overhead incurred and
the flexible budget total overhead (variable and fixed).

The flexible budget fixed overhead equals the master budget amount of $27,000. The budgeted variable
factory overhead rate is $3 per labor hour, the standard hours to produce one unit of product is 3 hours, and
1,650 units were produced. Thus, the flexible budget variable factory overhead was $14,850.

The actual overhead costs were $39,930. The total flexible budget variable factory overhead is $41,850
($14,850 + $27,000). Therefore, the flexible budget overhead variance is ($1,920) favorable ($39,930 −
$41,850). Since the actual overhead is less than the budgeted overhead, the variance is favorable.

B. This is the flexible budget variance based on the master budget level of output of 1,800 units. Actual factory
overhead of $39,930 − ([$3 × 3 × 1,800] + $27,000) budgeted fixed overhead = ($3,270) favorable. However, the
actual level of production of 1,650 units should be used to calculate flexible budget amounts for variable costs, not the
master budget level. See the correct answer for a complete explanation.

C. The flexible budget overhead variance is favorable. The budgeted overhead is greater than the actual, which
means the variance is favorable. See the correct answer for a complete explanation.

D. The flexible budget overhead variance is favorable. The budgeted overhead is greater than actual which means the
variance is favorable. See the correct answer for a complete explanation.

Question 92 - CIA 1190 IV.18 - Sales and Market Variances

A. The sales volume variance measures the impact of the difference in sales volume between the actual results and
the static budget. The sales volume variance for the contribution margin is the flexible budget contribution margin

(c) HOCK international, page 115


Part 1 : 07/07/14 10:28:01

minus the static budget contribution margin. This answer is the flexible budget variance for revenue, which is not an
answer to the question.

B. The sales volume variance measures the impact of the difference in sales volume between the actual results and
the static budget. The sales volume variance for the contribution margin is the flexible budget contribution margin
minus the static budget contribution margin. The sales volume variance is unfavorable as the flexible budget
contribution margin is lower than the static (master) budget contribution margin.

C.

The sales volume variance measures the impact of the difference in sales volume between the actual results and the
static budget. The sales volume variance for the contribution margin is the flexible budget contribution margin minus
the static budget contribution margin. The sales volume variance is unfavorable as the flexible budget contribution
margin is lower than the static (master) budget contribution margin.

D.

The sales volume variance measures the impact of the difference in sales volume between the actual results
and the static budget. The sales volume variance for a single product (or for a single product firm) can be
calculated for each variable income and expense item as well as for the contribution margin. If a question
does not specify which line to use, as this one does not, use the contribution margin line.

For the contribution margin line, it is calculated as follows: (Actual Sales Volume − Budgeted Sales Volume) ×
Budgeted Contribution per Unit, or (AQ − SQ) × SP. The "AQ," actual quantity, is 11,000. The "SQ," budgeted
quantity, is 12,000. The "SP," budgeted price, is the budgeted contribution margin per unit. That is $10 per
unit ($110,000 ÷ 11,000 or $120,000 ÷ 12,000). Thus the variance is (11,000 − 12,000) × $10, or ($10,000). A
negative variance for an income line or for the contribution margin line is an unfavorable variance, because it
means the actual was lower than the budget.

We can also calculate the variance using the amounts given in the variance report. The sales volume variance
for the contribution margin is the flexible budget contribution margin minus the static budget contribution
margin. Actual sales volume times budgeted contribution per unit is the flexible budget contribution margin,
i.e. $110,000. Budgeted sales volume times budgeted contribution per unit is the static (master) budget
contribution margin, i.e. $120,000. $110,000 − $120,000 equals ($10,000) unfavorable.

Question 93 - CMA 1287 4.30 - Manufacturing Input Variances - Materials and Labor

A. A favorable materials purchase price variance and an unfavorable materials quantity variance do not relate to the
quantity of materials purchased and used in production.

B. An unfavorable materials quantity variance means that more materials were used in production than budgeted
(standard), not less.

C. A favorable materials purchase price variance and an unfavorable materials quantity variance do not relate to the
quantity of materials purchased and used in production.

D. A favorable materials purchase price variance means the actual purchase price was less than was
budgeted (the standard).

Question 94 - CMA 696 3.24 - Manufacturing Input Variances - Materials and Labor

(c) HOCK international, page 116


Part 1 : 07/07/14 10:28:01

A. This is the material quantity variance. The question asks for the labor rate variance. See the correct answer for a
complete explanation.

B. The labor rate variance formula is: (Actual Rate − Standard Rate) × Actual Hours. The labor rate variance is
zero as the standard rate equals the actual rate of $12.00 per hour.

C. The labor rate variance is not unfavorable. See the correct answer for a complete explanation.

D. This is the difference between the actual total overhead incurred and the flexible budget overhead. This is not the
labor rate variance. See the correct answer for a complete explanation.

Question 95 - CMA 1294 3.27 - Manufacturing Input Variances - Overhead

A. This is the variable overhead efficiency variance. See the correct answer for a complete explanation.

B.

The variable overhead spending variance is calculated as follows: (Actual VOH Cost/Unit of allocation base actually
used − Standard Application Rate) × Actual Quantity of variable overhead allocation base used for the actual output.

This answer results from using the budgeted hours (100,000), not the actual hours (94,000) to calculate the actual
variable overhead cost/unit of the allocation base actually used; and also to multiply by the actual quantity of the
variable overhead allocation base used for the actual output.

C.

The variable overhead spending variance is calculated as follows: (Actual VOH Cost/Unit of allocation base
actually used − Standard Application Rate) × Actual Quantity of variable overhead allocation base used for the
actual output.

The actual VOH cost/unit of the allocation base actually used was the total actual variable overhead cost of
$740,000 divided by the actual number of direct labor hours worked of 94,000, or $7.8723 per direct labor
hour. The standard application rate of variable overhead was $8 per direct labor hour. The actual quantity of
the variable overhead allocation base used for the actual output was 94,000.

Therefore, the variable overhead spending variance was ($7.8723 − $8) × 94,000, which equals ($12,000)
favorable.

D. This is the fixed overhead spending variance. See the correct answer for a complete explanation.

Question 96 - CMA 1294 3.26 - Manufacturing Input Variances - Overhead

A.

This is not the correct answer. Please see the correct answer for a complete explanation.

We have been unable to determine how to calculate this incorrect answer choice. If you have calculated it, please let
us know how you did it so we can create a full explanation of why this answer choice is incorrect. Please send us an
email at support@hockinternational.com. Include the full Question ID number and the actual incorrect answer choice
-- not its letter, because that can change with every study session created. The Question ID number appears in the
upper right corner of the ExamSuccess screen. Thank you in advance for helping us to make your HOCK study
materials better.

(c) HOCK international, page 117


Part 1 : 07/07/14 10:28:01

B. This amount is the difference between the actual variable overhead and the budgeted fixed overhead. See the
correct answer for a complete explanation.

C.

The fixed overhead budget/spending variance is the difference between the actual fixed overhead incurred
and the budgeted fixed overhead costs. The actual amount of fixed overhead costs was $540,000.

The budgeted amount of fixed overhead was $500,000 ($5 of fixed overhead per labor hour multiplied by
100,000 budgeted labor hours). We use the budgeted amount of labor hours (100,000) because we need to
calculate what the total budgeted fixed overhead amount was that the company used in its calculation of the
cost per direct labor hour. To calculate the budgeted cost per direct labor hour, the company divided the total
budgeted fixed cost by the budgeted number of labor hours. Therefore, to find the budgeted fixed
manufacturing cost, we reverse the process and multiply the budgeted cost per labor hour by the number of
budgeted labor hours. The amount of budgeted fixed cost is the same for any level of production. Budgeted
fixed cost is the same in the flexible budget as it is in the static budget. So in calculating the budgeted fixed
manufacturing overhead, it does not matter what the actual level of production was.

The fixed overhead budget/spending variance is $40,000 unfavorable ($540,000 − $500,000). The actual fixed
overhead costs incurred were higher than the budgeted amount, so the variance is unfavorable.

D.

This answer results from using the actual amount of labor hours used (94,000) to calculate the budgeted fixed
overhead costs instead of the budgeted labor hours (100,000).

We use the budgeted amount of labor hours (100,000) because we need to calculate what the total budgeted fixed
overhead amount was that the company used in its calculation of the cost per direct labor hour. To calculate the
budgeted cost per direct labor hour, the company divided the total budgeted fixed cost by the budgeted number of
labor hours. Therefore, to find the budgeted fixed manufacturing cost, we reverse the process and multiply the
budgeted cost per labor hour by the number of budgeted labor hours. The amount of budgeted fixed cost is the same
for any level of production. Budgeted fixed cost is the same in the flexible budget as it is in the static budget. So in
calculating the budgeted fixed manufacturing overhead, it does not matter what the actual level of production was.

Question 97 - ICMA 10.P1.091 - Manufacturing Input Variances - Overhead

A.

It is not likely that a labor rate variance and a variable overhead efficiency variance would be related. A labor rate
variance results from a difference between the actual hourly rate paid for direct labor used and the budgeted hourly
rate. The variable overhead efficiency variance is the amount of the total variance caused by a different usage of the
allocation base (either direct labor hours or machine hours) than was expected. Even if direct labor hours are being
used as the allocation base, the usage of the allocation base is a quantity variance whereas the labor rate variance is
a price variance. The labor rate variance and the variable overhead efficiency variance are not related, and it is very
unlikely that one would cause the other.

B.

It is not likely that a labor efficiency variance and a fixed overhead volume variance would be related. A labor
efficiency variance results from a difference between the actual number of labor hours required for the actual output
and the standard number of labor hours allowed for the actual output. The fixed overhead volume variance (also
called the fixed overhead production-volume variance) is a measure of capacity utilization. It is caused by the actual
production level being different from the production level used to calculate the budgeted fixed overhead rate. The
labor efficiency variance and the fixed overhead volume variance are not related, and it is very unlikely that one would

(c) HOCK international, page 118


Part 1 : 07/07/14 10:28:01

cause the other.

C. It is not likely that a material price variance and a variable overhead efficiency variance would be related. A material
price variance results from a difference between the actual price per unit of the direct materials used and the budgeted
price per unit. The variable overhead efficiency variance is the amount of the total variance caused by a different
usage of the allocation base (either direct labor hours or machine hours) than was expected. The material price
variance and the variable overhead efficiency variance are not related, and it is very unlikely that one would cause the
other.

D.

A material usage variance can be caused by labor factors, and a labor efficiency variance can be caused by
material quality factors.

If employees are new or untrained, an unfavorable labor efficiency variance can result. The untrained
employees may also cause more direct material spoilage, which will result in an unfavorable material usage
variance. It can work the other way, as well. Knowledgeable and efficient employees can create both a
favorable labor efficiency variance and a favorable material usage variance.

Furthermore, an unfavorable material usage variance can be caused by inferior materials. The inferior
materials that the employees have to work with can also cause an unfavorable labor efficiency variance as
well, because it may take longer to produce the product using inferior materials.

Question 98 - ICMA 10.P1.108 - Manufacturing Input Variances - Materials and Labor

A. The performance of the shipping employees is not connected with the production process. Therefore,
neither good nor bad performance of them can affect the materials efficiency variance.

B. Poor quality of the raw materials can cause an unfavorable material efficiency variance, because more of the direct
material will be wasted.

C. Poor design can cause many production problems. One of the problems it could cause is material waste, which will
lead to unfavorable materials efficiency variances.

D. Inadequate training of the direct labor employees could be a cause of the unfavorable materials efficiency (usage)
variance. Inadequately trained labor will make more mistakes, thereby wasting more direct materials than will
adequately trained labor.

Question 99 - HOCK CMA P3A H1 - Business Process Performance

A. The amount of money spent on advertising is not directly related to competitive advantage.

B. While it may be important for the company to quickly distribute its product, this is not directly related to competitive
advantage.

C. Distribution channels are not directly related to competitive advantage.

D. A company is said to have competitive advantage when it is more profitable than the average company in
its industry. Profitability does not create competitive advantage, though. It is the other way around.
Competitive advantage is required in order to have high profitability. Thus, in order to increase profitability
and sustain profit growth, managers need to formulate strategies that will give their company a competitive

(c) HOCK international, page 119


Part 1 : 07/07/14 10:28:01

advantage.

Question 100 - HOCK CMA P3A H14 - Business Process Performance

A. Marketing and Sales is a primary activity. Marketing and sales includes advertising, promotion and sales
activities. The other answer choices are all support activities.

B. Human Resources is a support activity.

C. Information Systems is a support activity.

D. Infrastructure is a support activity.

Question 101 - CMA 1293 3.3 H5 - Joint Products and Byproducts

A. This answer does not reduce the joint costs by the sales proceeds from the by-product.

B.

Net realizable value (NRV) is calculated as the selling price minus future costs to complete and dispose. For
Alfa, the NRV is $2 per pound ($4 selling price minus $2 additional processing cost), or $20,000 in total. For
Betters, the NRV is $8 per pound ($10 selling price minus $2 additional processing cost), or $40,000 in total.
Together, the total NRV of the two products is $60,000. Of this, 1/3 is Alfa and 2/3 is Betters. Therefore, Alfa
will get 1/3 of the joint allocable costs.

The joint costs to allocate are the $93,000 in joint costs reduced by the $3,000 of revenue that will be received
from the sale of the by-product. (The process of inventorying the by-product requires that the $3 per unit
sales value is debited to inventory and the corresponding credit is a reduction of the production costs.) So,
$90,000 needs to be allocated, and Alfa's 1/3 of this is $30,000.

C. This is the amount that should be allocated to Betters.

D. This is the value of the by-product.

Question 102 - CMA 690 4.7 - Joint Products and Byproducts

A. This is the amount that should be allocated to CBL using the relative sales value as the allocation basis.

B. In order to allocate the costs using the relative sales value method, we need to know the total relative sales
value. There are 60,000 units of MSB that will be sold at $2 each for a total value of $120,000. There are 90,000
units of CBL that will be sold for $4 each, for a total value of $360,000. The total sales value of both products
is $480,000, and MSB represents 25% of this total sales value. The joint costs are $300,000 and MSB should
be allocated 25%, or $75,000, of the joint costs.

C. This is the amount that should be allocated to CBL using the number of units as the allocation basis.

D. This is the amount that should be allocated to MSB using the number of units as the allocation basis.

(c) HOCK international, page 120


Part 1 : 07/07/14 10:28:01

Question 103 - CMA 693 3.1 - Activity-Based and Life-Cycle Costing

A. The allocation of costs to a cost object will not enable the company to analyze why sales of a particular
product increased. This is because the sales of the item are connected to so many other factors: the price,
the economy, and the actions of competitors, to name a few.

B. The allocation of costs will be part of the analysis of whether or not a department should be expanded. Without this
allocation, management will not know if the department is profitable.

C. The allocation of costs will be part of the analysis of whether or not a product line should be discontinued. Without
this allocation, management will not know if the product line is profitable.

D. The allocation of costs is a critical part of the make-or-buy decision that a company must make.

Question 104 - CMA 695 3.3 - Process Costing

A. This is the cost per unit under the weighted average method.

B.

Under the FIFO method of process costing we need to make three calculations to determine the EUP. These
are: calculate (1) how many EUP were required to finish BWIP, (2) how many units were started and
completed and (3) how many EUP were needed to start the EWIP. There were 16,000 units in BWIP and they
were 60% complete for materials, meaning that they needed to do 40%, or 6,400 EUP to finish BWIP. There
were a total of 76,000 units started and completed during the period (92,000 completed minus the 16,000 in
BWIP). There were 24,000 units in EWIP that were 90% complete for materials, meaning that 21,600 EUP of
materials had been done on the EWIP. Adding these three numbers together, we get 104,000 total equivalent
units.
Direct
Materials
Completion of BWIP 6,400
Started & Completed 76,000
Starting of EWIP 21,600
TOTAL EUP-FIFO 104,000

In order to calculate the cost per unit of materials under FIFO we simply divide the costs added for materials
this period by the total EUP for materials for the period. The cost added was $468,000, and the EUP for
materials was 104,000. $468,000 divided by 104,000 total equivalent units gives a per unit cost for materials of
$4.50.

C. This answer does not use the correct EUP for the period.

D. This answer uses the EUP as calculated for the weighted average method.

Question 105 - HOCK CMA P3A H43 - Operational Efficiency

A. This is a true statement.

B.

(c) HOCK international, page 121


Part 1 : 07/07/14 10:28:01

The main idea of JIT is that nothing is produced until the next process in the assembly line needs it. JIT is a
"pull system" rather than a "push system." In a push system, a department produces and sends all that it can
to the next step for further processing, which means that the manufacturer is producing something without
understanding consumer demand. This can result in large, useless stocks of inventory.

Because the idea of JIT is that nothing will be produced until the next process needs it, technically this
means that nothing will be produced until a customer orders it. However, this is not actually possible, so
production is driven by the expected demand for the product.

C. This is a true statement.

D. This is a true statement. JIT is a pull system, which is based on a manufacturing philosophy that combines
purchasing, production and inventory control into one function. This reduces the level of inventory that is held within
the company at all stages of production, and lowers the cost of carrying the inventory.

Question 106 - ICMA 10.P1.160 - Classifications of Costs

A.

The difference between normal and actual costing systems is in the way the amount of overhead to be applied to
production is calculated.

When normal costing is being used, a predetermined overhead rate is calculated (called an estimated normalized
rate), using the expected overhead costs divided by the expected usage of the allocation base (machine hours or
direct labor hours). When the overhead is applied to production, this predetermined overhead rate is multiplied by the
amount of the allocation base (machine hours or direct labor hours) that was actually used to produce the product
during the period. (Note: this is in contrast to standard costing, where the predetermined overhead rate is multiplied by
the amount of the allocation base allowed for the actual output.)

When actual costing is being used, no predetermined overhead rate is calculated. Instead, at the end of each period
(month, quarter, whatever), the total actual overhead costs incurred for the period are calculated and the total is
divided by the total actual number of units produced to calculate the amount of overhead to be allocated to each unit
produced.

Since with actual costing a unique overhead application rate is calculated for each period based on the actual incurred
overhead costs and the actual production volume for the period, actual costing requires more people hours to
calculate the unique overhead rate for each reporting period. Normal costing is more economical because it uses a
pre-set rate that is usually used all year.

B.

The difference between normal and actual costing systems is in the way the amount of overhead to be applied to
production is calculated.

When normal costing is being used, a predetermined overhead rate is calculated, using the expected overhead costs
divided by the expected usage of the allocation base (machine hours or direct labor hours). When the overhead is
applied to production, this predetermined overhead rate is multiplied by the amount of the allocation base (machine
hours or direct labor hours) that was actually used to produce the product during the period. (Note: this is in contrast to
standard costing, where the predetermined overhead rate is multiplied by the amount of the allocation base allowed
for the actual output.)

When actual costing is being used, no predetermined overhead rate is calculated. Instead, at the end of each period
(month, quarter, whatever), the total actual overhead costs incurred for the period are calculated and the total is
divided by the total actual number of units produced to calculate the amount of overhead to be allocated to each unit

(c) HOCK international, page 122


Part 1 : 07/07/14 10:28:01

produced.

Since the totals of actual overhead costs incurred and actual units produced cannot be known until after the end of the
period, normal costing has an advantage over actual costing in that it is more timely. Overhead can be allocated to
products without having to wait for the end-of-period actual data on overhead costs incurred to become available.

C.

The difference between normal and actual costing systems is in the way the amount of overhead to be applied to
production is calculated.

When normal costing is being used, a predetermined overhead rate is calculated (called an estimated normalized
rate), using the expected overhead costs divided by the expected usage of the allocation base (machine hours or
direct labor hours). When the overhead is applied to production, this predetermined overhead rate is multiplied by the
amount of the allocation base (machine hours or direct labor hours) that was actually used to produce the product
during the period. (Note: this is in contrast to standard costing, where the predetermined overhead rate is multiplied by
the amount of the allocation base allowed for the actual output.)

When actual costing is being used, no predetermined overhead rate is calculated. Instead, at the end of each period
(month, quarter, whatever), the total actual overhead costs incurred for the period are calculated and the total is
divided by the total actual number of units produced to calculate the amount of overhead to be allocated to each unit
produced.

Since overhead costs are not incurred smoothly during the year and production varies during the year, allocating
overhead using actual costing will result in an amount of overhead being allocated to each unit that will vary each
reporting period. Thus, normal costing has an advantage in that it smooths the product costs throughout the period by
smoothing the amount of overhead applied to units produced each period.

D.

The difference between normal and actual costing systems is in the way the amount of overhead to be
applied to production is calculated.

When normal costing is being used, a predetermined overhead rate is calculated (called an estimated
normalized rate), using the expected overhead costs divided by the expected usage of the allocation base
(machine hours or direct labor hours). When the overhead is applied to production, this predetermined
overhead rate is multiplied by the amount of the allocation base (machine hours or direct labor hours) that
was actually used to produce the product during the period. (Note: this is in contrast to standard costing,
where the predetermined overhead rate is multiplied by the amount of the allocation base allowed for the
actual output.)

When actual costing is being used, no predetermined overhead rate is calculated. Instead, at the end of each
period (month, quarter, whatever), the total actual overhead costs incurred for the period are calculated and
the total is divided by the total actual number of units produced to calculate the amount of overhead to be
allocated to each unit produced.

Since normal costing uses a predetermined overhead application rate and actual costing uses the actual
overhead application rate, it is not true that normal costing provides improved accuracy of job and product
costing. In fact, it is the other way around. Actual costing provides improved accuracy because it uses actual
allocation rates.

Question 107 - CMA 1293 3.9 - Job-Order and Operation Costing

A. Operation costing is a combination of process costing and job-order costing. The job-order costing part
comes through in the way materials costs are usually allocated: on the basis of batches. Operation costing is

(c) HOCK international, page 123


Part 1 : 07/07/14 10:28:01

used in a situation in which a company produces similar items that differ mostly in the materials that are used.

B. In operation costing the materials are accounted for in batches, and everything else is usually accounted for as in
process costing.

C. Operation costing is a combination of process costing and job-order costing. Operation costing does not use actual
cost for the allocation of overhead.

D. Operation costing is a combination of process costing and job-order costing. The method used for overhead
allocation is not prescribed for operation costing. However, actual costing is practical only for job-order costing, not for
operation costing.

Question 108 - ICMA 10.P1.200 - Process Costing

A. This is the number of physical units in ending work-in-process inventory multiplied by 20%. 20% completion is the
point at which 40% of Material B is added; it is not the amount of Material B that is added at that point.

B. This is 50% of the number of physical units in ending work-in-process inventory. 50% is the stage of completion
that the units are at the end of the quarter. It is not the amount of Material B that has been added at that point.

C. This is the number of physical units in ending work-in-process inventory. The number of equivalent units for
Material B is the percentage of the total amount of Material B that has been added to the units in ending
work-in-process inventory by the end of the quarter. That is dependent upon the stage of completion the units are in at
the end of the quarter.

D.

There are 22,000 shirms in ending work-in-process inventory at the end of the quarter. They are all 50%
completed. The first addition of Material B takes place when the units are 20% completed, and 40% of Material
B is added at that time. The units in ending work-in-process inventory have had 40% of Material B added,
because at 50% complete, they are more than 40% complete, which is where the first addition of Material B
takes place.

However, the units in ending work-in-process inventory have not yet had the remaining 60% of Material B
added, because it is not added until they are 80% complete. The units are only 50% complete.

Therefore, the number of equivalent units of shirms in ending work-in-process inventory for Material B is
22,000 × .40, which equals 8,800.

Question 109 - CIA 586 IV.6 - Process Costing

A. This is the cost of good baseballs that were produced.

B. This answer calculates normal spoilage as 3% of the number of baseballs completed, not of the baseballs that pass
inspection.

C. The first thing that we need to do is to determine how many units are abnormal spoilage. Since 3% of the
good units is considered normal, this is 60 baseballs (3% × 2,000 units passing inspection). Since there were
100 spoiled units and 60 is the normal spoilage, abnormal spoilage was 40 units. After this, we need to
determine the cost per unit. Remember that we allocate costs to the spoiled units. Because there was no
beginning or ending work in progress, we can treat all of the costs the same, using a total cost of $1,155. This

(c) HOCK international, page 124


Part 1 : 07/07/14 10:28:01

is allocated to each of the 2,100 units produced, giving a rate of $.55 per unit. This is the cost that is allocated
to the 40 abnormally spoiled units, giving a total cost of $22 for abnormal spoilage.

D. This is the amount of normal spoilage.

Question 110 - CMA 1293 3.3 H2 - Joint Products and Byproducts

A. This is the amount that is allocated to Alfa.

B. This is the amount that would be allocated to Alfa if the joint costs are not reduced by the sales value of the
by-product.

C. This is the amount that would be allocated to Alfa if we included the calories from the Morefeed into the calculation.

D. The total calories of Alfa are 44,000,000 and the total calories of Betters are 56,000,000. In total, this is
100,000,000 calories, of which Better is 56%. The total joint costs to allocate are $90,000. This is made up of
the $93,000 in joint costs reduced by the inventoried sales value of the by-product. Betters is to receive 56%
of this $90,000, or $50,400.

Question 111 - CMA 689 4.11 - Classifications of Costs

A. Opportunity costs are the lost benefits from the next best use of a resource. These R & D costs have already been
incurred, and are therefore not opportunity costs.

B. Relevant costs are costs that are different between options. Since there are no options in this question, these R &
D costs are not relevant costs.

C. Conversion costs are the costs of direct labor and overhead, i.e., the costs of converting raw materials to the
finished goods.

D. Because these R & D costs have already been spent, they are sunk costs.

Question 112 - CMA 689 4.10 - Classifications of Costs

A. Because there are both fixed and variable overhead costs, the total overhead cost is a mixed cost.

B. A carrying cost is the cost of keeping inventory.

C. A committed cost is a cost for the company's infrastructure. Committed costs are costs that are required in order to
establish and maintain the readiness to do business. Examples are fixed assets such as property, plant and
equipment and intangible assets such as the purchase of a franchise. They are usually on the balance sheet as
assets and become expenses in the form of amortization and depreciation.

D. A sunk cost is a cost that has already been spent and cannot be changed by any current or future decisions.

(c) HOCK international, page 125


Part 1 : 07/07/14 10:28:01

Question 113 - CMA 1290 3.4 - Overhead Allocation

A. The only time when units of production is an appropriate allocation base is when the company produces
only one product. Since each unit produced of that one product will require the same amount of inputs,
allocating overhead according to units of production will produce an equitable allocation.

B. Whether direct labor cost is high or low is not relevant in determining whether units of production is an appropriate
overhead allocation base.

C. The relationship between direct material costs and direct labor costs is not relevant in determining whether units of
production is an appropriate overhead allocation base.

D. Units of production is not an appropriate overhead allocation base when several well-differentiated products are
manufactured. Each individual product will require a different amount of inputs, and allocating overhead evenly to all
units produced would produce an inequitable allocation.

Question 114 - CIA 1192 IV.6 - Process Costing

A.

Under the FIFO method, we need to make 3 calculations to determine the number of equivalent units of
conversion costs.

(1) We need to determine the equivalent units of production required to finish the beginning work-in-process.

(2) We need to calculate the number of units that were started and completed during the period.

(3) And we need to determine the number of equivalent units of production done to start the ending
work-in-process.
How Calculated EUP-Conv. Costs
1) Completion of BWIP 20,000 × 50% 10,000
2)+ Started & Completed 180,000 − 20,000 160,000
3)+ Starting of EWIP 10,000 × 50% 5,000
EUP (FIFO) 175,000

There were 20,000 units in beginning work-in-process and they were 50% complete. This means that 50% of
the work, or 10,000 equivalent units, was performed this period to complete the units in BWIP.

180,000 units were transferred out, but as there were 20,000 units in beginning work-in-process, only 160,000
units were started and completed.

There were 10,000 units in ending work-in-process that were also 50% complete. This means that 50% of the
work, or 5,000 equivalent units, was done this period to start those units.

Adding together these 3 numbers, we get 175,000 equivalent units of conversion costs.

B. This is the number of units completed during the period.

C. This is the number of equivalent units of conversion costs under the weighted average method. When FIFO is
used, only costs added during the period are allocated between units completed and units in ending work-in-process
inventory. Costs in beginning work-in-process inventory are allocated 100% to BWIP. Since equivalent units of
production are used to allocate costs between units completed and units in EWIP, the units in BWIP are not included
in the calculation of the total equivalent units of production under FIFO.

(c) HOCK international, page 126


Part 1 : 07/07/14 10:28:01

D. This is the number of equivalent units of material during December.

Question 115 - ICMA 10.P1.188 - Overhead Allocation

A.

This is not the correct answer. Please see the correct answer for an explanation.

We have been unable to determine how to calculate this incorrect answer choice. If you have calculated it, please let
us know how you did it so we can create a full explanation of why this answer choice is incorrect. Please send us an
email at support@hockinternational.com. Include the full Question ID number and the actual incorrect answer choice
-- not its letter, because that can change with every study session created. The Question ID number appears in the
upper right corner of the ExamSuccess screen. Thank you in advance for helping us to make your HOCK study
materials better.

B. This is the amount of overhead applied in the Tooling Department only.

C. The predetermined overhead rates are: Tooling, $8,625 ÷ 460 direct labor hours = $18.75/DLH. Fabricating:
$16,120 ÷ 620 direct labor hours = $26.00/DLH. Budgeted direct labor hours on Job #231 in the Tooling
Department were 12 hours, so $18.75 × 12, or $225 of overhead was applied in the Tooling Department.
Budgeted direct labor hours on Job #231 in the Fabricating Department were 3 hours, so $26.00 × 3, or $78.00
of overhead was applied in the Fabricating Department. The total applied for both departments was $225 +
$78, or $303.

D.

This is not the correct answer. Please see the correct answer for an explanation.

We have been unable to determine how to calculate this incorrect answer choice. If you have calculated it, please let
us know how you did it so we can create a full explanation of why this answer choice is incorrect. Please send us an
email at support@hockinternational.com. Include the full Question ID number and the actual incorrect answer choice
-- not its letter, because that can change with every study session created. The Question ID number appears in the
upper right corner of the ExamSuccess screen. Thank you in advance for helping us to make your HOCK study
materials better.

Question 116 - HOCK CMA P3A H20 - Operational Efficiency

A.

Toys 4 Us has 100 completed units in finished goods inventory at the end of January. It needs to increase
finished goods inventory to 200 units. In addition, it needs to meet demand of 1,200 units during the month of
February. Therefore, production during February needs to be 1,300 units, and this production will take place
evenly throughout the month, which consists of four weeks. Thus, weekly production will be 325 units.

Each toy truck requires 18 wheels: 6 wheels for the cab (1 × 2) + (1 × 4) and 12 for the trailer (3 × 4). Therefore,
the weekly requirement will be 325 × 18, or 5,850 wheels. Beginning raw materials inventory is 1,350 wheels.
On Feb. 1, an order of 5,000 wheels will be received. During the week beginning Feb. 1, the company will use
5,850 wheels in manufacturing, leaving it with only 500 wheels on hand in raw materials inventory. Therefore,
on Feb. 8, the company will need to receive enough wheels to meet its weekly production schedule requiring
5,850 wheels, plus it will need to receive enough wheels to increase its raw materials inventory of wheels
from 500 units to the minimum required 2,000 units, a 1,500 unit increase. Thus, the company will require

(c) HOCK international, page 127


Part 1 : 07/07/14 10:28:01

7,350 wheels to be delivered on Feb. 8. Since only lots of 1,000 can be ordered, the order placed on Feb. 1 for
Feb. 8 delivery should be for 8,000 units.

B. An answer of 7,000 results from failing to recognize the need to increase the finished goods inventory.

C. An answer of 6,000 results from failing to recognize the need to increase the finished goods inventory and the raw
materials inventory.

D. 5,400 is the number of wheels needed per week to fulfill the projected sales demand. However, it does not
recognize the need to increase the inventories on hand, nor does it recognize the limitation on the size of the lot that
can be ordered.

Question 117 - CMA 696 3.29 - Classifications of Costs

A. This answer includes selling costs. Cost of goods sold includes direct materials, direct labor and overhead applied.
Selling costs are not part of the cost of goods sold.

B. This answer includes administrative costs. Cost of goods sold includes direct materials, direct labor and overhead
applied. Administrative costs are not part of the cost of goods sold.

C. This answer includes selling and administrative costs. Cost of goods sold includes direct materials, direct labor and
overhead applied. Selling and administrative costs are not part of the cost of goods sold.

D. The production costs incurred to complete Job ICU2 were: direct materials ($13,700), direct labor ($4,800)
and overhead applied ($20,000, or 800 hours × $25 per hour). In total this is $38,500. Allocated to the 7,000
good units produced, this is a cost per unit of $5.50. Administrative and selling costs are not part of the cost
of goods sold. Only production costs are included in cost of goods sold.

Question 118 - CMA 1286 4.18 H2 - Variable and Absorption Costing

A. This answer includes variable selling and other expenses which are not included in inventoriable costs under the
variable method because they are not production costs.

B. This answer does not include variable manufacturing overhead, which is inventoried under variable costing.

C. Under variable costing, the variable costs of production are inventoried while the fixed manufacturing
costs are expensed as incurred. The variable costs of production include the prime costs (direct labor and
direct materials) and variable manufacturing overhead. In total, this is $900,000.

D. This answer includes fixed manufacturing overhead, which is not inventoried under variable costing.

Question 119 - CMA 678 4.10 - Classifications of Costs

A. This is the definition of a controllable cost.

B. Committed costs are costs for the company's infrastructure. They are costs that are required in order to
establish and maintain the readiness to do business. Examples are fixed assets such as property, plant and
equipment and intangible assets such as the purchase of a franchise. They are usually on the balance sheet

(c) HOCK international, page 128


Part 1 : 07/07/14 10:28:01

as assets and become expenses in the form of amortization and depreciation. Therefore, they are governed
mainly by past decisions that established the current levels of operating and organizational capacity, and they
change slowly in response to small changes in capacity.

C. This is the definition of a variable cost.

D. This is the definition of a discretionary cost.

Question 120 - ICMA 10.P1.159 - Classifications of Costs

A. This answer results from incorrectly classifying the $27,700 uninsured factory fire loss as a product cost. Even
though it is a factory cost, it should not be included in the product costs because it is not a usual cost of
manufacturing. The cost of the product should not be increased because of this.

B. This answer results from classifying all variable costs as product costs and all fixed costs plus the uninsured fire
loss as period costs. All variable costs are not product costs, and all fixed costs are not period costs.

C. This answer results from incorrectly classifying manufacturing overhead as period cost.

D.

Product costs include: direct materials $56,000; direct labor $179,100; variable manufacturing overhead
$154,000; and fixed manufacturing oveerhead $267,000, for a total of $656,100.

Period costs include: variable selling costs $108,400; fixed selling costs $121,000; fixed administrative costs
$235,900; and the uninsured factory fire loss of $27,700, for a total of $493,000.

Question 121 - ICMA 10.P1.228 - Operational Efficiency

A. JIT systems produce goods as they are needed. Utilizing a JIT system would reduce the size of the finished goods
inventory, as items theoretically go directly from finished production to sales.

B. The main idea of JIT is that nothing is produced until the next process in the assembly line needs it. Therefore, JIT
is a "pull system" rather than a "push system." Nothing is produced until it is needed, so essentially nothing is
produced until a customer orders it. Therefore, JIT is actually more reliant on sales orders as a trigger for production
runs.

C. With a JIT system a company works with a few suppliers very closely to ensure the quality of materials as
well as the timing of many smaller shipments.

D. JIT relies on receiving many smaller shipments of raw materials rather than on large orders. Large orders increase
inventory costs as well as storage and handling expenses.

Question 122 - HOCK CMA P3A H38 - Operational Efficiency

A. Kanban was developed at Toyota in the 1960s.

B. Kanban is part of a JIT system and its purpose is to manage the flow on a manufacturing assembly line. This is a

(c) HOCK international, page 129


Part 1 : 07/07/14 10:28:01

chain process where orders flow from one process to another, so production of components is pulled to the production
line, rather than the pushed (as is done in the traditional forecast-oriented system).

C. Kanban is a Japanese term for "visual record." It is a simple parts movement system or an inventory
system in which 'cards' or 'tickets' are used to keep track of inventory and its movement. The term "kaizen" is
the Japanese term for "continuous improvement."

D. Kanban is a simple parts movement system or an inventory system in which 'cards' or 'tickets' are used to keep
track of inventory and its movement. The supplier delivers components to the production line on an "as needed" basis,
signaled by receipt of a card and empty container, eliminating storage in the production area.

Question 123 - CMA 1292 3.2 - Activity-Based and Life-Cycle Costing

A. The amount of materials used does not provide a good basis for allocating factory service department costs.

B. The amount of power consumed is the best of the choices given for allocating factory service department
costs.

C. The use of units sold is usually not a good allocation basis for anything except shipping costs or sales related costs.

D. Salaries of service department employees is not a good allocation basis for anything.

Question 124 - CMA 697 3.8 - Service Cost Allocation

A. Under the direct method the costs of one service department are not allocated to the other service
departments.

B. Under the direct method the costs of one service department are not allocated to the other service departments.

C. Under the direct method the costs of one service department are not allocated to the other service departments.

D. Under the direct method the costs of one service department are not allocated to the other service departments.

Question 125 - CMA 1293 3.15 - Overhead Allocation

A. When the cost drivers are the same in all processes, then only one allocation basis is needed. .

B. When there are different products that use resources in the departments at different rates, departmental
overhead rates are preferable to a single factory wide rate.

C. When there is only one product produced, the company can use one allocation rate, since all of the costs will go to
that product.

D. If it is not possible to determine the cause and effect relationship between cost drivers and the cost objects, then
departmental rates may not be accurate or cost efficient.

(c) HOCK international, page 130


Part 1 : 07/07/14 10:28:01

Question 126 - CMA 690 4.3 - Overhead Allocation

A. This is the cost of goods sold. Cost of goods sold is beginning finished goods inventory + cost of goods
manufactured − ending finished goods inventory. The question asks only for the cost of goods manufactured.

B.

Cost of goods manufactured is total manufacturing costs adjusted for the change in work-in-process
inventory. Total manufacturing costs are total prime costs (direct materials used + direct labor used) +
manufacturing overhead applied. We are told that the direct labor was $300,000 but will need to calculate the
direct materials used and the manufacturing overhead applied for January.

The beginning direct materials inventory was $134,000. During the period, they purchased $189,000 of direct
materials and also incurred $3,000 in transportation in costs. However, they also returned $1,000 of direct
materials during the period. Adding these together we can calculate the total direct materials available during
January as $325,000. Since there was an ending inventory of $124,000, they must have used $201,000 of direct
materials during the period.

Overhead is applied as 60% of direct labor, which totals $180,000 ($300,000 × 60%). Adding the prime costs
(direct materials and direct labor) plus the overhead, we get $681,000 of total manufacturing costs.
Work-in-process inventory increased during the period by $16,000. This means that $16,000 of the work
performed during the period was in ending work-in-process inventory and not finished goods inventory, so
this amount needs to be subtracted from the total manufacturing cost to calculate the cost of goods
manufactured. The cost of goods manufactured is $681,000 − $16,000, or $665,000.

Another way to calculate the cost of goods manufactured is to use the beginning and ending WIP inventory
balances instead of the amount of change. Beginning WIP inventory was $235,000. Cost of manufacturing
was $681,000. Ending WIP Inventory was $251,000. Cost of goods manufactured is beginning WIP inventory of
$235,000 plus cost of manufacturing of $681,000 minus ending WIP Inventory of $251,000, which is $665,000.

C. This is the total cost of manufacturing. Cost of goods manufactured is total manufacturing costs adjusted for the
change in work-in-process inventory.

D.

The question asks for cost of goods manufactured. This answer results from an incorrect calculation of cost of goods
sold. Cost of goods manufactured is one component in the calculation of cost of goods sold.

Cost of goods sold is beginning finished goods inventory + cost of goods manufactured − ending finished goods
inventory. This answer results from turning around beginning and ending finished goods inventory in the calculation of
cost of goods sold. This is ending finished goods inventory + cost of goods manufactured − beginning finished goods
inventory. The question asks only for cost of goods manufactured.

Question 127 - ICMA 10.P1.232 - Business Process Performance

A. Distribution involves delivery of products or services to customers after the sales are made, and customer service
cannot be provided until the goods or services are in the hands of the purchasers.

B. Distribution involves delivery of products or services to customers after the sales are made. Marketing efforts are
required to make the sales. So marketing would logically have to come before distribution.

C. Production design would logically have to come before production.

D.

(c) HOCK international, page 131


Part 1 : 07/07/14 10:28:01

Research and development, marketing, and customer service is the correct order of those functions in the
value chain. The primary activities in the value chain, in order of their completion, are: research &
development, production, marketing & sales, and customer services.

Research provides a company with needed knowledge for creating new or improving existing products,
services or processes. Development uses those research findings to develop and improve the company’s
products, processes, or services. The next step is design, which is the detailed planning and engineering for
these efforts. Design is followed by production, which is the acquisition, coordination and assembly required
to produce a product or deliver a service. Once the product is available for sale, marketing and sales,
including advertising, promotion and sales activities, create the sales. Distribution involves delivery of
products or services to customers after the sales are made. Customer service includes customer support and
warranty services after a sale takes place.

Question 128 - ICMA 10.P1.161 - Overhead Allocation

A. This answer results from either dividing direct costs by direct labor hours or by dividing total indirect costs by total
direct and indirect labor hours. The question is asking for an allocation of indirect costs. The number of indirect hours
are irrelevant here, as indirect costs are allocated based on an activity base that is directly related to production, such
as direct labor hours. In a normal costing system, the activity base used is the actual amount of the allocation base
that was used in producing the product.

B.

In a normal costing system, the budgeted indirect cost allocation rate is calculated by dividing the total
indirect costs by the budgeted amount of the allocation base to be used in producing the product. As given in
this question, that is 250,000 direct labor hours. The indirect costs are $5,000,000 for labor-related costs and
$7,000,000 for non-labor-related costs, for total indirect costs of $12,000,000. Dividing $12,000,000 by 250,000
direct labor hours results in an indirect application rate of $48 per direct labor hour.

Note that in a normal costing system, costs are applied to production on the basis of the actual usage of the
allocation base multiplied by the application rate. This is different from standard costing, where costs are
applied to production on the basis of the allowed usage of the allocation base for the actual output multiplied
by the application rate.

C. This is the indirect non-labor related costs divided by the direct labor hours. This is partially correct, as the direct
labor hours are the activity base to use for allocation of indirect costs in a normal costing system. However, all of the
indirect costs, labor and non-labor, need to be included in the calculation.

D. This is the indirect labor costs divided by the direct labor hours. This is partially correct, as the direct labor hours
are the activity base to use for allocation of indirect costs in a normal costing system. However, all of the indirect
costs, labor and non-labor, need to be included in the calculation.

Question 129 - CIA 593 IV.5 - Process Costing

A.

This is number of units transferred in plus the number of units in ending work-in-process inventory. The units in ending
work-in-process inventory are counted twice, since they were also transferred-in units. Furthermore, not all of the units
transferred in have received the direct materials. The units in ending work-in-process inventory are only 20% complete
with respect to conversion costs, and the direct materials are not added until the units are 25% complete with respect
to conversion costs.

(c) HOCK international, page 132


Part 1 : 07/07/14 10:28:01

B. This is the number of units transferred in, but not all of them have received the direct materials. The units in ending
work-in-process inventory are only 20% complete with respect to conversion costs, and the direct materials are not
added until the units are 25% complete with respect to conversion costs.

C.

The equivalent units for direct materials are:


Direct
Materials
Completion of BWIP 0
Started & Completed 70,000
Starting of EWIP 0
Total EUP-FIFO 70,000

Because the materials are added when the units are 25% complete, we know that the units in BWIP had the
materials added during the previous period because they are 40% complete at the beginning of the current
period. This means that no materials were added during the current period to the units in BWIP.

There were 80,000 units transferred in (started on) during the period, so 80,000 is the maximum number of
EUP that could be used with respect to direct materials. However, since the 10,000 units in EWIP have not yet
reached the 25% stage, the materials have not yet been added to them.

Therefore, the number of EUP of materials used during the period is only 70,000, the number of units Started
& Completed.

D. This is the number of EUP for materials using the weighted average method.

Question 130 - CIA 594 III.99 - Estimating Fixed and Variable Costs

A. This is the per unit variable cost. The question is asking for the average total cost per unit, which includes both
variable and fixed.

B.

The question is asking for the average cost per unit of cost X at a production level of 8,000 units, a production
level not given in the table, which includes both fixed and variable costs. To solve this, we will first need to
estimate the total costs for cost X at a production level of 8,000 units and then find the average cost per unit.
To find the total of cost X at that level, we need to find how much of the historical cost is fixed cost and how
much is variable cost. Based on the given information, the High-Low Points Method is the best method to use
to separate these costs.

First, calculate the variable portion of the cost. This is done by dividing the difference between the highest
and the lowest total cost for X by the difference between the associated number of units produced. The result
is the variable cost per unit produced of X:

($178,260 − $23,700) / (35,000 − 3,000) = $4.83 variable cost per unit of X produced

The next step is to find the fixed cost using the following equation:

FC = Total Cost − Variable Cost

We can solve the above equation using the data from either the highest or the lowest observation.

Using the highest observation, this will be:

(c) HOCK international, page 133


Part 1 : 07/07/14 10:28:01

FC = $178,260 − ($4.83 × 35,000)

FC = $9,210

Next, we use this fixed cost and the variable cost per unit to find the total cost at a production level of 8,000:

TC = FC + VC

TC = $9,210 + ($4.83 × 8,000)

TC = $47,850

The final step is to calculate the average total cost at the production level of 8,000 by dividing the total cost by
8,000:

$47,850 ÷ 8,000 = $5.98

C. This is the average cost per unit of cost X at a production level of 9,000 units.

D. This is the average cost per unit of cost X at a production level of 3,000 units.

Question 131 - ICMA 10.P1.236 - Business Process Performance

A.

Benchmarks should be based on best practices. In benchmarking, the best performance anywhere can be used as a
standard to be attained. However, most companies do not permit outside organizations to review their processes
on-site — particularly if they are competitors.

A large corporation can send a team to its best division to review their processes. Personnel there can provide
information about lessons they have learned and potential problems and demonstrate the successful application of the
processes. Someone from that division can coach other departments in improving their processes. This is an
advantage that a large corporation would have that probably would not be available to a smaller organization.

B. Benchmarks should be based on best practices. In benchmarking, the best performance anywhere can be used as
a standard to be attained. The best performance of a competitor with a similar operation is a good "best practice" to
emulate.

C. Benchmarks should be based on best practices. In benchmarking, the best performance anywhere can be used as
a standard to be attained. The best performance of a unit in comparable past periods is a good "best practice" to
emulate.

D. Benchmarks should be based on best practices. In benchmarking, the best performance anywhere can be
used as a standard to be attained. The unit’s previous year's performance may have been outstanding, or it
may have been average, or it may have been a disaster. Unless the previous year's performance was the best
performance anywhere, it cannot be used as a benchmark, so the previous year’s performance is not an
example of a benchmarking standard.

Question 132 - ICMA 10.P1.170 - Overhead Allocation

A. This answer results from (1) assuming that current budgeted operating income is zero and operating income is to

(c) HOCK international, page 134


Part 1 : 07/07/14 10:28:01

be increased by the full $50,000 of desired operating income instead of by $30,000; and (2) misreading the question
as asking for the total number of units to be produced instead of the increase in the number of units to be produced.

B. This answer results from including fixed selling and administrative cost along with fixed manufacturing cost in the
calculation. Only fixed manufacturing cost is relevant.

C.

This answer results from assuming that all of the additional production will be sold. The problem does not say that the
additional production will be sold. The expected sales are still only 5,000 units, and only the revenues and costs for
units that are sold reach the income statement. Only the production increases, and that causes an increase to
inventory on hand.

Because the production increases, the amount of fixed overhead applied to production will be more than the actual
incurred fixed overhead. That variance will be closed out by crediting the amount of the overapplication to cost of
goods sold, which will increase gross profit and increase net income before taxes by the amount of the variance.

However, this answer would result in an increase in net income before tax of only $4,920, which would not accomplish
George's misguided goal.

D.

The company is expecting $20,000 in operating income, and George wants to increase that by $30,000 to
$50,000. Both sales and production are planned for 5,000 units. Budgeted fixed manufacturing cost is
$100,000, so the application rate for fixed overhead is $20 per unit ($100,000 ÷ 5,000 units expected to be
produced). If the company increases production above what it can sell, inventory will increase. As a result,
the fixed manufacturing cost applied to the unsold units in inventory will be on the balance sheet instead of in
Cost of Goods Sold on the income statement, and net income will be increased by the amount of fixed
manufacturing cost attached to the unsold units in ending inventory.

To increase net income by $30,000, the number of units in inventory will need to increase by $30,000 ÷ $20
fixed manufacturing cost applied per unit, or 1,500 units.

The company's practice is to write off under- or over-applied manufacturing cost to COGS. If production is
increased by 1,500 units, fixed manufacturing cost will be overapplied by 1,500 units × $20 per unit, or
$30,000, assuming that actual fixed manufacturing cost is the same as budgeted fixed manufacturing cost.
This overapplied fixed manufacturing cost will be cleared out at the end of the period with a credit to COGS of
$30,000. So fixed manufacturing cost in COGS will be $30,000 lower than it would have been if the company
had produced only the 5,000 units it planned to produce and that it was able to sell.

Note that because the whole variance is written off (credited) to Cost of Goods Sold, Finished Goods
Inventory will not be adjusted. That extra $30,000 will remain on the balance sheet in Inventory at year end,
still attached to the 1,500 extra unsold units that were produced.

Because of the overproduction which led to a variance and the resulting credit to Cost of Goods Sold
expense, only $70,000 of the actual $100,000 of fixed manufacturing cost incurred will reach the income
statement, whereas all $100,000 of it would reach the income statement if only 5,000 units were produced and
sold.

When in some future year the company cuts production in order to sell down the extra inventory it built up
this year, each unit produced in that future year will bear more of that year's fixed overhead. Cost of goods
sold for that future year will include the $30,000 from this year's fixed overhead (for the units sold out of
inventory) plus the greater amount of fixed overhead per unit for the units sold from the future year's
decreased production. As a result, cost of goods sold in that future year will be higher than it should be and
net income will be lower than it should be. Thus the result of the overproduction in the current year is to move
some of a future year's net income into the current year.

(c) HOCK international, page 135


Part 1 : 07/07/14 10:28:01

Question 133 - CMA 692 3.3 - Process Costing

A. This is the equivalent units of conversion costs for the period. It does not work for the materials because the
materials are all added at one time in the process, when the units are 60% complete with respect to conversion costs..

B.

The equivalent units in the assembly department for direct materials are:
Direct
Materials
Completion of BWIP 8,000
Started & Completed 30,000
Starting of EWIP 0
Total EUP-FIFO 38,000

Because the company uses the FIFO method, there are three numbers that we need to add together to
calculate equivalent units of production (EUP): (1) the number of EUP to finish beginning WIP + (2) the
number of units started and completed + (3) the number of EUP to start ending WIP. Materials are added to
the process when the unit is 60% complete. Because of this, none of the beginning WIP had materials, so
materials needed to be added to all of the beginning WIP — that is 8,000 EUP. The number of units started and
completed was 30,000 (38,000 units transferred out minus the 8,000 units in beginning WIP). And since the
ending WIP is not yet to the 60% completion stage, there was no material added to the units in EWIP. Adding
these numbers together, we get 38,000 EUP of materials.

C. This is the number of units transferred in during the period.

D. This is the number of units started and completed.

Question 134 - CMA 696 3.2 - Overhead Allocation

A. Because the master-budget capacity is less than theoretical, the master-budget capacity will allocate more
overhead to each unit. This means that there will be more overhead costs in inventory using the master-budget
capacity and therefore a higher net income.

B.

The most important fact given here is that variances are adjustments to cost of goods sold. If variances are
prorated among cost of goods sold, finished goods inventory and work in process inventory, the choice of a
denominator level will make no difference in net income, because net income will be the same no matter
which denominator level is chosen. However, when all variances are adjustments to cost of goods sold, the
choice of the denominator level will make a difference in net income.

By using the practical level, a smaller amount of overhead will go to each unit than if the master-budget level
is used. Therefore, less overhead will be included in ending inventory at the end of the period. Since there is
less overhead on the balance sheet, there will be more overhead on the income statement since variances go
to cost of goods sold. More overhead on the income statement will lead to a lower net income.

C. Because the normal capacity has the highest overhead rate, more overhead will be included in inventory using this
capacity than any other. Therefore, using normal capacity will lead to a higher income than any of the other methods.

D. Because the practical capacity is higher than the normal capacity, the normal capacity level will lead to higher
overhead costs per unit. Therefore, the normal capacity level will lead to more overhead being included in inventory

(c) HOCK international, page 136


Part 1 : 07/07/14 10:28:01

and a higher income than the practical capacity would give.

Question 135 - CIA 596 III.99 - Activity-Based and Life-Cycle Costing

A. The number of sales people would not be a good basis for the allocation of distribution costs.

B. The number of phone calls would not be a good basis for the allocation of distribution costs.

C. The number of shipments would be a good basis for the allocation of distribution costs because the more
shipments that are made, the higher the distribution costs will be.

D. The dollar sales volume would not be a good basis for the allocation of distribution costs because a very expensive
item might be very small and have low distribution costs.

Question 136 - CMA 680 4.5 - Process Costing

A. The costs of abnormal spoilage are not charged to inventory.

B. Abnormal spoilage is charged to a special loss account and presented on the income statement in the
period of the spoilage.

C. The costs of abnormal spoilage are not charged to a material variance account.

D. The costs of abnormal spoilage are not charged to manufacturing overhead.

Question 137 - CMA 697 3.4 - Process Costing

A. Operation costing is used when there is a similar process applied to units that differ in the materials that are input
into the process. This is not the case here.

B. Activity-based costing requires the identification and use of cost drivers, which are not present in the information
given.

C. Process costing is usually used in a system like this where the similar products move through a process
with different stages or departments.

D. Job-order costing is used when each product is different and unique from the others.

Question 138 - CIA 1185 IV.11 - Joint Products and Byproducts

A.

The net realizable value method is generally used instead of the sales value at splitoff method only when
selling prices for one or more products at splitoff do not exist. Since both products do have sales prices at
the splitoff point, we use their relative sales values at the splitoff point in order to allocate the joint costs,

(c) HOCK international, page 137


Part 1 : 07/07/14 10:28:01

even though one product will be processed further.

Using the relative sales value method, we will allocate the $10,000 of joint costs to the different products.
Since product X has a sales value at splitoff of $12,000 and Y has a sales value at splitoff of $8,000, the total
sales value is $20,000. 60% of this is from product X, so product X will receive 60%, or $6,000, of the joint
costs.

B. This is the total amount of joint costs that need to be allocated.

C. This answer results from using the sales value after further processing, net of further processing costs, for Product
X to allocate the joint cost. However, since Product X can be sold at the splitoff point, its sales value at the splitoff
point should be used to allocate the joint cost. Net realizable value of one product and the sales value at splitoff for the
other product can be used if the first product has no sales value at splitoff because it cannot be sold at splitoff. But
whenever a product can be sold at the splitoff point, that sales value at the splitoff point should be used for allocating
the joint cost, even if the product will be processed further.

D. This is how much would be allocated if the joint costs were allocated equally.

Question 139 - CIA 586 IV.11 - Joint Products and Byproducts

A. This is the amount that would be allocated to F1 if physical units were used to allocate the joint costs.

B. The NRV of an item is calculated as the selling price minus the costs to complete and dispose. In order to
allocate joint costs based on NRV, we need to calculate the total NRV that all of the joint products have at the
split off point. We will make these calculations based on one ton of raw materials. We can do this because the
answer is in %, not in the dollar amount. F1 has an NRV of $2 per unit and there are 5 units, so this is $10 of
NRV. F2 has an NRV of $5 per unit and there are 2 units, so this is $10 of NRV. F3 has an NRV of $10 per unit
and there are three units, so this is $30 of NRV. In total there is $50 of NRV and the NRV of F1 is 20% of this
amount.

C. This is the amount that would be allocated if the joint costs were allocated equally to the three products.

D. This is the answer if the allocation were done based on selling price, not NRV.

Question 140 - ICMA 13.P1.004 - Estimating Fixed and Variable Costs

A.

A semi-variable cost has both a fixed component and a variable component. There is a basic fixed amount
that must be paid regardless of activity, even if there is no activity. And added to that fixed amount is an
amount that varies with activity. Therefore, a semi-variable cost would graph as a straight line where the
straight line begins at the level of the fixed component and moves upward at a constant slope based on the
variable component. However, a negative amount of fixed cost at zero output (represented by −25,000 in the
cost function) is not possible. The only way that could occur is if someone is paying the company not to
produce, and that is not one of the available answer choices.

A cost function such as the one given is usable for measuring costs only within the relevant range. The
relevant range is the span of activity over which a certain cost behavior holds true. For example, the cost
behavior for the cost function given might hold true for production between 15,000 units and 20,000 units.
However, at any level below 15,000 units, the cost function does not hold true.

(c) HOCK international, page 138


Part 1 : 07/07/14 10:28:01

At 15,000 units, total cost according to the cost function would be −25,000 + (2.5 × 15,000) = $12,500.

At 20,000 units, total cost according to the cost function would be −25,000 + (2.5 × 15,000) = $25,000.

However, if the cost function is applied to a production leve of 10,000l, the result is −25,000 + (2.5 × 10,000) =
$0, which is not possible.

And at a zero production level, the result is −25,000 + (2.5 × 0) = −$25,000, which is also not possible.

Therefore, the zero level of output is outside of the relevant range.

B. A semi-variable cost has both a fixed component and a variable component. There is a basic fixed amount that
must be paid regardless of activity, even if there is no activity. And added to that fixed amount is an amount that varies
with activity. Therefore, a semi-variable cost would graph as a straight line where the straight line begins at the level of
the fixed component and moves upward at a constant slope based on the variable component. Based upon the cost
function given, this cost does exhibit semi-variable behavior.

C. The value of the independent variable in a simple regression determines the slope of the line on a graph. It does
not determine the level of the constant.

D. In statistics, an "outlier" is an observation that is far away from the remainder of the observations. It appears to
deviate markedly from the rest of the data. There is nothing in this question indicating that many outliers wee included
in the data.

Question 141 - CMA 1293 3.3 H4 - Joint Products and Byproducts

A. This is the answer if we use calories to allocate the costs.

B. Net realizable value (NRV) is calculated as the selling price minus future costs to complete and dispose.
For Alfa, the NRV is $2 per pound ($4 selling price minus additional processing costs of $2), or $20,000 in
total. For Betters, the NRV is $8 per pound ($10 selling price minus $2 additional processing costs), or
$40,000 in total. Together, the total NRV of the two products is $60,000. Of this, 1/3 is Alfa and 2/3 is Betters.
Therefore, Betters will get 2/3 of the joint allocable costs. The joint costs to allocate are the $93,000 and
because they do not inventory the by-product, all $93,000 of the joint costs need to be allocated. Betters is to
receive 2/3 of this, or $62,000.

C. This is the amount that would be allocated to Alfa.

D. This is the amount that would be allocated to Alfa is Atlas inventoried the by-product.

Question 142 - CIA 1194 III.47 H2 - Joint Products and Byproducts

A. This is the total cost of Product R.

B. This would be the total cost if the joint costs were allocated based on physical units.

C. This answer calculates the allocation of the joint costs by using the sales value at the split-off point based on actual
sales.

D. In order to calculate the total cost of Product S, we need to know how much of the joint costs will be
allocated to Product S. In order to allocate joint costs based on the sales value at the split off point, we need
to determine the sales value at the split off point for each item. For R it is $250,000 (2,500 units produced ×

(c) HOCK international, page 139


Part 1 : 07/07/14 10:28:01

$100). For S it is $400,000 (5,000 units × $80). For T it is $150,000 (7,500 × $20). In total this is $800,000, of
which Product S is 50%. Therefore, 50% of the joint costs should be allocated to Product S. Joint costs were
$720,000, so Product S receives $360,000. For the company it is more beneficial to process product S further
because it gains greater incremental revenue after further production. Thus, we need to add the $150,000 of
processing costs after the split off point to determine the total cost of Product S as $510,000.

Question 143 - ICMA 10.P1.189 - Process Costing

A.

The equivalent units of production using the weighted average method are:
Direct MaterialsConversion Costs
Units completed 27,000 27,000
Starting of EWIP 1,500 1,500
Total EUP - Weighted Average 28,500 28,500

Note: Since the equivalent units of production are the same (28,500) for both direct materials and conversion
costs, we can add the direct materials costs and conversion costs together and calculate the total inventory
cost per unit in a single calculation. If the number of equivalent units of production were not the same for
direct materials and conversion costs, we would need to calculate the inventory cost for direct materials
separately from the inventory cost for conversion costs and then add the two inventory costs together to get
the total. (The problem would have had to give us separate beginning work in process inventory costs for
direct materials and inventory, too, and it does not do that.)

The total direct materials and conversion costs are:


Beginning WIP inventory cost (direct materials & conversion costs) $ 4,300
Direct materials cost incurred 39,700
Conversion costs incurred 70,000
Total costs $114,000

Total cost of $114,000 ÷ 28,500 equivalent units worked on during the month = $4.00 cost per equivalent unit
worked on. That was also the cost per unit of the completed units.

B.

This is the costs incurred during the month (excluding the costs in beginning WIP inventory) divided by the number of
equivalent units worked on during the month (excluding the units in beginning WIP inventory).

Under the weighted average method, both the costs in beginning work-in process inventory and the equivalent units in
BWIP are included in the calculation of the cost per equivalent unit.

C. This is the total costs of $114,000 divided by 32,500 equivalent units. The number of equivalent units is incorrect.
The total equivalent units uner the weighted average method is completed units plus the equivalent units used to start
the work on the units in ending work-in-process inventory.

D.

This is not the correct answer. Please see the correct answer for an explanation.

We have been unable to determine how to calculate this incorrect answer choice. If you have calculated it, please let
us know how you did it so we can create a full explanation of why this answer choice is incorrect. Please send us an
email at support@hockinternational.com. Include the full Question ID number and the actual incorrect answer choice
-- not its letter, because that can change with every study session created. The Question ID number appears in the
upper right corner of the ExamSuccess screen. Thank you in advance for helping us to make your HOCK study

(c) HOCK international, page 140


Part 1 : 07/07/14 10:28:01

materials better.

Question 144 - ICMA 10.P1.225 - Service Cost Allocation

A. Under the direct method, the company simply allocates all of the service departments’ costs directly to the
production departments. The Information Systems Department is a service department and the Assembly
Department is a production department. Therefore, some of the costs of the Information Systems Department
would be allocated to the Assembly Department.

B. The Personnel Department and the Information Systems Department are both service departments. Under the
direct method, the services that are provided by the different service departments to each other are ignored. The
company simply allocates all of the service departments’ costs directly to the production departments. Therefore, no
service department costs are allocated to other service departments, and no Personnel Department costs would be
allocated to the Information Systems Department.

C. The Machining Department is a production department and the Information Systems Department is a service
department. Costs of production departments are never allocated to service departments. Instead, service department
costs are allocated to production departments.

D. The Assembly Department and the Machining Department are both production departments. Production
departments' costs are never allocated to other production departments. Only service departments' costs are allocated
to production departments.

Question 145 - CMA 691 3.48d - Job-Order and Operation Costing

A. This answer results from calculating the per unit conversion cost in the Trim operation by dividing the total costs of
$69,000 by the 5,000 units of Deluxe chairs worked on and then multiplying the per unit conversion cost by the 1,000
units in ending WIP inventory. However, the 1,000 units in ending WIP inventory were only 60% complete as to Trim
operations. Therefore, the number of units in the divisor for the calculation of per unit cost is incorrect, and it is
incorrect to multiply that per unit Trim operations cost by 1,000 units to calculate the ending WIP inventory.

B. This is the total conversion cost of the finish operation.

C. This is the total conversion cost of the trim operation.

D.

The 3,000 Deluxe units are 100% complete as to materials, 100% complete as to Extrusion Operation costs,
and 100% complete as to Form Operation costs. However, 1,000 of the 3,000 units are only 60% complete as
to Trim Operation costs.

The costs per unit for the Deluxe model are as follows:

Materials (all units are 100% complete as to materials):

Extrusion materials: Total costs $36,000 ÷ 3,000 units produced = $12 per unit
Form materials: Total costs $12,000 ÷ 3,000 units produced = $4 per unit
Trim materials: Total costs $9,000 ÷ 3,000 units produced = $3 per unit

Conversion costs:

In calculating conversion costs per unit, we must use the total costs of each operation (for all models) and

(c) HOCK international, page 141


Part 1 : 07/07/14 10:28:01

the total number of units of all models that went through each operation, because we do not have the costs
broken down by model.

Extrusion operation: Total costs $392,000 to produce 16,000 units = $24.50 per unit
Forming operation: Total costs $132,000 to produce 11,000 units = $12.00 per unit
Trim operation: Total costs $69,000 to produce 4,600 units (calculated as follows: 2,000 complete Executive
units + 2,000 complete Deluxe units + 600 Deluxe equivalent units [60% of 1,000 partially complete Deluxe
units]) = $69,000 ÷ 4,600 units = $15.00 per equivalent unit.

Total costs in EWIP inventory for 1,000 Deluxe units that are 100% complete as to materials, 100% complete
as to extrusion and form operations, and 60% complete as to trim operation are:

Materials: ($12 + $4 + $3) × 1,000 units = $19,000


Conversion costs: ($24.50 × 1,000) + ($12.00 × 1,000) + ($15.00 × 600) = $45,500
Materials costs of $19,000 + Conversion costs of $45,500 = Total cost of $64,500.

Question 146 - CIA 589 IV.4 - Job-Order and Operation Costing

A. Operations costing is used when the items produced share some, but not all costs and materials.

B. Job-order costing is used when the item (or in this case service) produced is unique, as is the case in a
consulting company.

C. Just-in-time costing would be used together with a JIT inventory system, which is not in use here.

D. Process costing is used when the units produced (or services provided) are similar.

Question 147 - CMA 696 3.4 - Process Costing

A.

This answer results from dividing the total costs of $50,000 by the 10,000 units started and multiplying the result by
4,000 units to represent 2,000 equivalent units in direct materials and 2,000 equivalent units in conversion costs in
ending work-in-process inventory.

The total cost allocated to units in ending work-in-process inventory must be calculated separately for direct materials
and conversion costs. The cost for direct materials in ending WIP inventory is then added to the conversion costs in
ending WIP inventory to find the total cost. Furthermore, the costs should be allocated on the basis of the number of
equivalent units of production in direct materials and the number of equivalent units of production in conversion costs,
which will usually be different. In this case, the number of equivalent units in conversion costs in ending
work-in-process inventory is different from the number in direct materials.

B.

This answer results from dividing the total costs of $50,000 by the 10,000 units started and multiplying the result by
the 500 equivalent units in conversion costs in ending work-in-process inventory.

The total cost allocated to units in ending work-in-process inventory must be calculated separately for direct materials
and conversion costs. The cost for direct materials in ending WIP inventory is then added to the conversion costs in
ending WIP inventory to find the total cost.

Furthermore, the costs should be allocated on the basis of the number of equivalent units of production in direct

(c) HOCK international, page 142


Part 1 : 07/07/14 10:28:01

materials and the number of equivalent units of production in conversion costs, which will usually be different.

C.

This question does not say whether the FIFO or the Weighted Average cost flow assumption is being used,
but since there is no beginning work-in-process inventory, it doesn't matter which one is being used. The
cost per equivalent unit for each cost element will be the costs added during the month divided by the total
equivalent units.

In order to answer this question, we need to determine the equivalent units of production for both materials
and conversion costs, then calculate the cost per EUP for each, multiply the cost per EUP for each by the
number of units of each in ending WIP inventory for the period, and sum the results.

There were 10,000 equivalent units of production for materials: the 8,000 bats that were completed plus 100%
of the 2,000 bats that had been started but not completed, since the question says that all of the Forming
Department's direct materials were placed in process.

There were a total of 8,500 equivalent units of production in conversion costs: 8,000 units that were
completed plus 25% or 500 of the 2,000 units in ending inventory that were 25% complete as to conversion
costs.

Here are the EUP:


Direct Conversion
Materials Costs
Units Completed 8,000 8,000
Starting of EWIP 2,000 500
Total EUP 10,000 8,500

The costs per EUP are:


Direct Conversion
Materials Costs
Total costs $33,000 $17,000
Total EUP 10,000 8,500
Cost/EUP $3.30 $2.00

The costs allocated to the units in ending WIP are:


Direct materials 2,000 × $3.30 = $6,600
Conversion costs 500 × $2.00 = 1,000
Total costs in ending WIP $7,600

D.

This answer results from dividing the total costs of $50,000 by the 10,000 units started and multiplying the result by
the 2,000 physical units in ending work-in-process inventory.

The total cost allocated to units in ending work-in-process inventory must be calculated separately for direct materials
and conversion costs. The cost for direct materials in ending WIP inventory is then added to the conversion costs in
ending WIP inventory to find the total cost.

Furthermore, the costs should be allocated on the basis of the number of equivalent units of production in direct
materials and the number of equivalent units of production in conversion costs, not on physical units.

(c) HOCK international, page 143


Part 1 : 07/07/14 10:28:01

Question 148 - CMA 696 3.21 - Overhead Allocation

A. If the amount is material, the underapplied or overapplied overhead is apportioned to work in process inventory,
finished goods inventory, and cost of goods sold.

B. Underapplied or overapplied overhead is never allocated only to finished goods inventory.

C. The treatment of the underapplied or overapplied overhead depends on the materiality of the amount. If it is
immaterial it can be moved to cost of goods sold. If it is material, it is apportioned among work in process
inventory, finished goods inventory, and cost of goods sold.

D. If the amount of underapplied or overapplied overhead is immaterial, it can be charged to cost of goods sold.
However, if the amount is material, it is prorated among work in process and finished goods inventories and cost of
goods sold.

Question 149 - CIA 594 III.46 - Variable and Absorption Costing

A. This is a true statement because an increase in inventory will keep more of the fixed factory overhead on the
balance sheet and off the income statement.

B. This is a true statement. Under absorption costing, an increase in inventory will keep more of the fixed factory
overhead on the balance sheet and off the income statement resulting in decreased expenses and higher operating
profit. A decrease in inventory will cause some of the fixed factory overhead that is on the balance sheet to be
expensed, resulting in an increase in expenses and lower operating profit.

C.

This is a true statement. When sales volume is greater than production volume, units are sold from inventory which
causes the fixed cost attached to those units to reach the income statement as cost of sales under absorption costing.
As a result, total fixed cost expensed would consist of all of the current year's fixed cost plus a portion of previous
years' fixed costs.

Under variable costing, all fixed costs would be expensed in the year incurred, so the fixed cost expensed for the
current year would be only the current year's fixed cost.

D. Under variable costing, the fixed factory overheads are expensed in the period incurred, so an increase in
production would not impact the income from the period, nor the manager's review.

Question 150 - ICMA 10.P1.193 - Process Costing

A. The cost of $152,000 is the total cost of $9.50 per unit multiplied by 16,000 units transferred to finished goods
inventory. The cost of the normally spoiled goods is a cost of the good units, as well.

B.

The costs per unit were $3.50 for materials and $6.00 for conversion costs, for a total cost per unit of $9.50.
Since normal spoilage is charged to good units, the total cost transferred was for 16,000 good units
completed plus the 300 units normally spoiled. So the total cost was $9.50 × 16,300, or $154,850. However,
only the good units were actually transferred to finished goods inventory, and that was 16,000 units.

Because of the normal spoilage cost included in the good units' cost, the cost per unit transferred was
$154,850 ÷ 16,000, or $9.678 per unit. Note that the unit cost of the units transferred to finished goods

(c) HOCK international, page 144


Part 1 : 07/07/14 10:28:01

inventory ($9.678) is higher than the $9.50 cost per unit calculated above. That is because each unit
transferred to finished goods inventory bears some of the cost of the normal spoilage.

C. The cost of $155,800 is the total cost of $9.50 per unit multiplied by 16,400 units, which is all the units worked on
including the good units completed and transferred to finished goods inventory, the normally spoiled units, and the
abnormally spoiled units. The costs transferred to finished goods inventory do not include the cost of abnormally
spoiled units, because that cost is charged to expense as it is incurred.

D. 16,300 units is the number of good units plus the number of abnormally spoiled units. Only the good units were
transferred to finished goods inventory.

Question 151 - CIA 1188 IV.6 - Process Costing

A. This is the number of physical units worked on during the period.

B. This is the total equivalent units for conversion costs using the FIFO cost flow method; but the question says the
weighted average method is to be used.

C. This answer includes 30% of the BWIP in addition to including all of the BWIP in the 7,400 units completed. It is
double counting the work that had been done last period.

D.

Under the weighted average method, we pretend the work in beginning work-in-process inventory was done
and the costs for it were incurred during the current period. Therefore, the total equivalent units of production
for conversion costs will equal all the units completed during the period (including the number of physical
units that were in beginning work-in-process inventory and were completed this period) plus the equivalent
units of production that were in ending work-in-process inventory.

The question tells us that 7,400 units were completed during the period. Ending work-in-process inventory
had 2,300 units that were 60% complete as to conversion costs, or 1,380 equivalent units.

Thus the total equivalent units of conversion costs for the period were 7,400 + 1,380, or 8,780.

Question 152 - ICMA 10.P1.226 - Service Cost Allocation

A. This answer results from calculating the cost allocations by dividing each individual department's usage of the
allocation base by the total usage by all the departments (9,300 computer usage hours or 11,600 square feet
occupied). The proration of the costs should be based not on the total usage by all departments but on the total usage
by only the departments to which the costs are being allocated.

B. In the step-down method, the costs of one of the service departments are allocated first, and those costs
are allocated to the other service department(s) and the production departments. When the second service
department’s costs are allocated, those costs are allocated to any service department(s) other than the first
service department allocated and to the production departments. Any service department whose costs have
already been allocated does not receive an allocation from any other service department.

Beginning with the Systems Department, its overhead cost of $200,000 is allocated to the Facilities,
Machining, Assembly and Finishing departments on the basis of their computer usage hours. The usage of
those four departments totals 900 + 3,600 + 1,800 + 2,700 = 9,000 hours. The usage of the Facilities
Department is 900 hours, or 10% of that. Thus, the Facilities Department is allocated 10% of the Systems

(c) HOCK international, page 145


Part 1 : 07/07/14 10:28:01

Department’s overhead cost of $200,000, or $20,000.

The Facilities Department has overhead cost to allocate of $100,000 plus the $20,000 allocated to it from
Systems, for a total of $120,000. Facilities costs are allocated on the basis of square feet occupied by the
Machining, Assembly and Finishing departments. The square feet occupied by those three departments totals
2,000 + 3,000 + 5,000 = 10,000 square feet. The square feet occupied by the Machining Department is 2,000
square feet, or 20% of that. Thus, the Machining Department is allocated 20% of the Facilities Department’s
overhead cost of $120,000, or $24,000.

C.

This is not the correct answer. Please see the correct answer for an explanation.

We have been unable to determine how to calculate this incorrect answer choice. If you have calculated it, please let
us know how you did it so we can create a full explanation of why this answer choice is incorrect. Please send us an
email at support@hockinternational.com. Include the full Question ID number and the actual incorrect answer choice
-- not its letter, because that can change with every study session created. The Question ID number appears in the
upper right corner of the ExamSuccess screen. Thank you in advance for helping us to make your HOCK study
materials better.

D. This answer results from beginning the allocations with the Facilities Department and also from not allocating any
costs of the Facilities Department to the Systems Department, as would be appropriate if the allocation began with
Facilities. But the question says to begin with the Systems Department, not with the Facilities Department.

Question 153 - ICMA 10.P1.152 - Classifications of Costs

A.

Variable marketing costs are costs which are the same per unit sold and which vary in total with sales
volume. The only variable marketing costs are the sales commissions and the marketing manager's incentive.
Based on projected sales of $100,000, sales commissions are .08 × $100,000, or $8,000. The marketing
manager's incentive is .005 × $100,000, or $500 based on the projected sales. Since both these costs will vary
with sales volume, they are both variable costs. Total variable marketing costs for the new product are thus
$8,500. The other costs are all fixed costs.

B. This is the cost of the sales commissions, which are a variable marketing cost. However, the sales commissions
are not the only variable marketing cost. The marketing manager's incentive is also tied to sales performance and will
therefore vary with the sales volume.

C. This includes the cost of the sales commissions as well as the cost of the salaried marketing staff's time to produce
the brochure. The cost of the brochure is not directly dependent on sales, making it a fixed cost. This calculation also
does not take into consideration the marketing manager's incentive, which is tied to sales and so is a variable
marketing cost.

D. This includes the sales commissions and the marketing manager's incentive expenses, which are both variable
marketing costs since they are tied directly to sales. However, it also includes the cost of the marketing staff in
producing the brochure. The cost to produce the brochure will not change based on the sales volume. Therefore, it is
not a variable marketing cost.

Question 154 - ICMA 10.P1.156 - Classifications of Costs

A. "Linear" means that when a function is graphed, the relationship between the X value and the Y value will

(c) HOCK international, page 146


Part 1 : 07/07/14 10:28:01

graph as a straight line. The cost function is assumed to be linear. Thus, when cost and level of activity are
plotted on a graph, the result will be a straight line. The straight line begins at the level of fixed costs on the Y
axis (where the activity level is zero) and moves upward by the amount of the variable cost per unit for each
one-unit increase in activity on the X axis.

B. This is not an assumption concerning cost behavior. The time period used can be any time period management
wants analyzed.

C. General and administrative costs may be either fixed or variable.

D. This is not an assumption concerning cost behavior.

Question 155 - CMA 1277 5.5 - Classifications of Costs

A. This is the definition of a sunk cost.

B. This is the definition of a fixed cost.

C. This is the definition of an opportunity cost.

D. An imputed cost is one that does not need to be paid out in cash, but it still relevant for decision making.

Question 156 - CMA 1290 H2 - Variable and Absorption Costing

A.

This problem does not specify whether Valyn is using a standard cost system (in which standard, or planned,
costs are used to account for production) or an actual cost system (in which actual costs are used). However,
it does say that Valyn uses a predetermined manufacturing overhead rate for applying manufacturing
overhead to its product. And since the actual, incurred per unit costs for direct materials, direct labor and
variable manufacturing overhead are exactly the same as the planned per unit costs for those items, we do
not need to know whether standard costing or actual costing is being used in order to answer this question.

Under variable costing, the cost per unit in ending inventory was $25 ($12 direct materials per unit + $9 direct
labor per unit + $4 variable manufacturing overhead per unit). Fixed manufacturing cost per unit is not
included, since under variable costing, fixed manufacturing cost is expensed as incurred. Given an ending
inventory of 40,000 units, the value of ending inventory was $1,000,000 (40,000 units × $25 per unit).

B.

This is not the correct answer. Please see correct answer for an explanation.

We have been unable to determine how to calculate this incorrect answer choice. If you have calculated it, please let
us know how you did it so we can create a full explanation of why this answer choice is incorrect. Please send us an
email at support@hockinternational.com. Include the full Question ID number and the actual incorrect answer choice
-- not its letter, because that can change with every study session created. The Question ID number appears in the
upper right corner of the ExamSuccess screen. Thank you in advance for helping us to make your HOCK study
materials better.

C. This answer results from incorrectly calculating ending inventory as 30,000 units. Ending inventory is beginning
inventory plus production minus sold units, which is 40,000 units.

(c) HOCK international, page 147


Part 1 : 07/07/14 10:28:01

D. This answer results from including variable selling and variable administrative expenses as inventoriable costs.
Only variable product costs are inventoriable costs on the variable costing basis, and selling and administrative
expenses are not product costs.

Question 157 - ICMA 10.P1.223 - Service Cost Allocation

A. Systems overhead is allocated on the basis of hours used. Under the direct method, overhead is allocated
only to production departments, not to other service departments. Therefore, Systems overhead will be
allocated only to the Machine, Assembly and Finishing departments. The total hours used by these
departments is 3,600 + 1,800 + 2,700 = 8,100. So the total overhead of the Systems department, which is
$200,000, will be allocated by dividing that amount by 8,100 to calculate the allocation rate per hour used by
the production departments.

B. 1,200 hours is the total of computer usage hours for the Systems and Facilities departments. The Systems costs
will be allocated only to the production departments. So the divisor used in calculating the allocation rate should be the
total of the production departments' usage hours.

C. 9,000 hours is the total of computer usage hours for all of the departments other than the Systems department. The
Systems costs will be allocated only to the production departments. So the divisor used in calculating the allocation
rate should be the total of the production departments' usage hours.

D. 9,300 hours is the total of computer usage hours for all of the departments, including the Systems department. The
Systems costs will be allocated only to the production departments. So the divisor used in calculating the allocation
rate should be the total of the production departments' usage hours.

Question 158 - ICMA 10.P1.177 - Variable and Absorption Costing

A. In all cases when ending finished goods inventory exists, ending finished goods inventory will be higher under
absorption costing, not variable costing.

B. This would not cause ending finished goods inventory to be higher under direct (variable) costing than under
absorption costing. If more units were sold than were produced during a given year, that would cause operating
income to be higher under variable costing, but it would not cause ending finished goods inventory to be higher under
variable costing.

C. This would not cause ending finished goods inventory to be higher under direct (variable) costing than under
absorption costing. If more units were produced than were sold during a given year, that would cause operating
income to be higher under absorption costing, but it would not cause ending finished goods inventory to be higher
under variable costing.

D. In all cases, ending finished goods inventory will be higher under absorption costing, not variable costing.
An allocation of fixed manufacturing cost is included in inventoried goods under absorption costing whereas
under variable costing it is not, because the fixed manufacturing cost is expensed as incurred under variable
costing.

Question 159 - ICMA 10.P1.066 - Classifications of Costs

A. The cost of glue used to secure the attachment of the legs to the tables is an overhead cost and so would appear in

(c) HOCK international, page 148


Part 1 : 07/07/14 10:28:01

the company's overhead budget.

B. The production supervisor's salary as well as his or her fringe (employee) benefits are overhead costs and thus
would appear in the company's overhead budget.

C. Overtime paid to the workers who perform production scheduling is an overhead cost and so would appear in the
company's overhead budget.

D. Freight charges for the delivery of raw materials to the company are transportation-in costs.
Transportation-in costs are landing costs, and they are inventoried along with the inventory they are related
to. They are a part of direct materials cost and thus are direct costs, not overhead costs.

Question 160 - CMA 1286 H1 - Variable and Absorption Costing

A. This answer does not include the fixed factory overhead. Under absorption costing, all factory overheads are
allocated to the units and inventoried.

B. This answer includes all of the selling and administrative costs. Selling and administrative costs are not inventoried
under either absorption costing or variable costing.

C. This answer does not include any of the factory overhead. Under absorption costing all factory overheads are
allocated to the units and inventoried.

D. Under the absorption costing method, all costs of production — variable and fixed — are included in
inventory. Therefore, the inventoriable costs are: direct materials ($300,000), direct labor ($100,000), variable
factory overhead ($50,000) and fixed factory overhead ($80,000). This is a total of $530,000.

Question 161 - CMA 694 3.6 - Variable and Absorption Costing

A. By definition, gross margin is sales price minus cost of goods sold.

B. This answer is incorrect because fixed manufacturing costs are included in the gross margin calculation.

C. This answer is incorrect because fixed costs are included in gross margin, and selling and administrative costs are
not.

D. This answer is incorrect because fixed indirect manufacturing costs are included in the gross margin calculation.

Question 162 - ICMA 10.P1.197 - Process Costing

A. This answer results from including 10 equivalent units in ending work-in-process inventory for conversion costs and
using 110 as the total equivalent units for conversion costs. The units in ending work-in-process inventory were not
complete, so the number of equivalent units in ending WIP inventory is less than 10.

B. This is the conversion costs incurred during the month. The total conversion costs, including both the costs incurred
during the month and the costs in beginning work-in-process inventory, must be allocated between completed units
and units in ending work-in-process inventory.

C. This is the total of the conversion costs in beginning work-in-process inventory and the conversion costs incurred

(c) HOCK international, page 149


Part 1 : 07/07/14 10:28:01

during the month. This total conversion costs must be allocated between completed units and units in ending
work-in-process inventory.

D.

In order to answer this question, we need to determine the equivalent units of production for conversion
costs, calculate the cost per EUP for conversion costs, and then multiply the cost per EUP for conversion
costs by the number of units completed and transferred out during the period.

The equivalent units of production using the weighted average method are:
Conversion
Costs
Units Completed 100
Starting of EWIP 4
Total EUP - WAVG 104

Conversion costs are added uniformly throughout the process, and EWIP was 40% complete as to
conversion. Therefore, the equivalent units of conversion costs were 40% of the 10 units in EWIP, or 4 EUP.
Total EUP were thus 104 for conversion costs.

The cost per EUP for conversion costs was the total of the conversion costs in beginning WIP inventory and
the conversion costs incurred during the period, divided by the total EUP for conversion costs.

The total conversion costs are:


Beginning WIP inventory cost $ 180
Conversion costs incurred 1,484
Total costs $1,664

Total conversion costs of $1,664 ÷ 104 equivalent units for conversion costs = $16 cost per equivalent unit
worked on. A total of 100 units were completed and transferred to the next department, so the total
conversion cost assigned to the units transferred out was $16 × 100 units = $1,600.

Question 163 - CMA 695 3.6 - Process Costing

A. This is the answer when the FIFO method is used.

B. This answer does not include the costs of materials that were in BWIP.

C.

The cost per equivalent unit using the weighted average method is the total costs in both beginning WIP
inventory and costs added during the period, divided by the total EUP during the period.

Under the weighted average method to calculate the equivalent units, we need to add together the number of
units completed and the number of EUP that were done to start the EWIP. In this question 92,000 units were
completed and the 24,000 units in EWIP are 90% complete for materials, giving us 21,600 EUP in EWIP. This
gives a total of 113,600 EUP of materials for the period.
Direct
Materials
Units Completed 92,000
Starting of EWIP 21,600
TOTAL EUP-WAVG 113,600

Here are the total costs for direct materials:

(c) HOCK international, page 150


Part 1 : 07/07/14 10:28:01

Beginning WIP costs $ 54,560


Costs added 468,000
Total costs $522,560

The total material cost is $522,560 and this is divided by the 113,600 EUP, giving us a cost per unit for
materials of $4.60.

D. This answer results from omitting the Beginning Work in Process inventory from the calculation of Total EUP. When
the weighted average method is being used, Total EUP must include the Beginning Work in Process inventory. The
cost per EUP is the total costs, including costs in BWIP and costs incurred for the period, divided by the Total EUP,
including BWIP in the calculation of Total EUP.

Question 164 - CIA 578 IV.1 - Job-Order and Operation Costing

A. Other costing methods may also be used to estimate overhead costs included in transfer prices.

B. Job-order costs are useful to determine the cost of a specific project. Job-order costing is used when a
company has unique and identifiable projects or jobs.

C. The LIFO inventory method may be used with other costing methods besides job-order costing.

D. Job-order costs do not influence future prices any more than any other method.

Question 165 - ICMA 10.P1.241 - Business Process Performance

A. Internal failure occurs when a problem is detected before a defective product is shipped to a customer.
Reworking defective units is done when they are found to be defective before they are sold or shipped, so the
cost of rework is an internal failure cost.

B.

Internal failure occurs when a problem is detected before a defective product is shipped to a customer.

Product testing may be done before a new product enters into production, so that would be part of the product design
cost. Product design is a prevention cost, because flaws in design become defective products. Quality design and
testing prevent defective products from ever being produced.

Product testing might also be considered an appraisal cost, if it refers to testing or inspecting the finished product at
the completion of production.

Both prevention costs and appraisal costs are conformance costs, whereas internal failure costs are nonconformance
costs.

C. Internal failure occurs when a problem is detected before a defective product is shipped to a customer. Inspection
of units must occur before a problem can be detected. The cost to inspect units produced is an appraisal cost, which
is a cost incurred in order to determine if an individual unit is defective.

D. Internal failure occurs when a problem is detected before a defective product is shipped to a customer. Warranty
repair costs are costs to correct a defect after the customer has received the product, so they are externala failure
costs.

(c) HOCK international, page 151


Part 1 : 07/07/14 10:28:01

Question 166 - CMA 696 3.19 - Classifications of Costs

A. This question is like a cost of goods sold question and the formula that we need to use is: beginning inventory +
purchases − materials used = ending inventory. This answer does not take into account the beginning inventory.

B. This question is like a cost of goods sold question and the formula that we need to use is: beginning
inventory + purchases − materials used = ending inventory. Putting the numbers into the formula, we get
$27,500 + X − $128,900 = $28,750. Solving for X, we get $130,150 of materials purchased during the period.

C. This question is like a cost of goods sold question and the formula that we need to use is: beginning inventory +
purchases − materials used = ending inventory. This answer does not take into account the ending inventory.

D. This question is like a cost of goods sold question and the formula that we need to use is: beginning inventory +
purchases − materials used = ending inventory. This answer switches the beginning and ending inventory amounts in
the formula.

Question 167 - CMA 685 5.6 - Classifications of Costs

A. Common costs are the costs that are shared by more than one cost object. A cost object is anything for
which costs are accumulated for managerial purposes: a specific product, job, product line, a market or
certain customers.

B. Prime costs are direct material and direct labor and they are traceable to one cost object.

C. Variable costs are costs that vary as production changes. They may or may not benefit more than one cost object.

D. Conversion costs are the costs of converting direct material to the finished goods. They include direct labor and
overhead. They may or may not benefit more than one cost object.

Question 168 - CMA 1285 4.14 H1 - Variable and Absorption Costing

A. This is the operating income as calculated under variable costing.

B. Under absorption costing the fixed manufacturing costs are allocated to the products produced. The
variable costs of production are $30 per unit and the fixed costs per unit are $3 ($600,000 ÷ 200,000 units
produced). In total, the cost per unit is $33. Since the sales price was $40 per unit, this is a gross profit of $7
per unit. With 120,000 units sold, that is $840,000 in total gross profit. Subtracting from this the selling and
administration costs of $400,000, we get an operating income of $440,000.

C. This answer results from expensing all of the fixed manufacturing cost as would be done under variable costing and
also not deducting the selling and administrative costs.

D. This answer results from not deducting the selling and administrative costs that are expensed as period costs.

Question 169 - CIA 1193 IV.5 e - Overhead Allocation

(c) HOCK international, page 152


Part 1 : 07/07/14 10:28:01

A. The amount of the overtime wage that is in excess of the regular wage rate should be charged to overhead.

B. Nothing should be charged to repair and maintenance, as the labor costs are split between direct labor and
overhead.

C. Because the incurrence of overtime is not directly related to a specific product or order, the costs related
to the overtime should be charged to overhead. However, only the overtime premium (the amount of the
salary in excess of the normal wage rate), is a cost related to overtime and is overhead. The standard wage
rate will be charged to direct labor for each hour of overtime that was worked.

D. Only the excess of the overtime wage over the regular wage should be charged to overhead.

Question 170 - CMA 697 3.7 - Service Cost Allocation

A. This is the amount that is allocated to Department B if only a single rate is used.

B. This answer allocates the fixed costs based on the actual quantities printed instead of the budgeted pages.

C. In the allocation of the fixed costs, Department A will receive 1/3 of the fixed costs since they use 1/3 of the
printing services. This is $33,333. In addition to the fixed costs, though, they will also receive a charge of $.03
per page based on their actual usage. Since they printed 1,400,000 pages, this variable charge is $42,000. So,
in total Department A will be charged $75,333.

D. This is the variable cost that is charged to Department A.

Question 171 - CMA 696 3.30 - Activity-Based and Life-Cycle Costing

A. Under the traditional system, overheads are allocated based on direct labor at a rate of 14.5% of direct
labor. Direct labor costs were $27,500 and 14.5% of this is $3,987.50. Under ABC we will need to make three
allocations, one for each activity. For incoming materials the amount charged to Satin Sheen is $138 (12 ×
$11.50). For in-process inspection it is $2,450 (17,500 × $.14). For product certification it is $1,925 (25 × $77).
Adding these three amounts together, we get $4,513, which is $525.50 more than what was allocated under
the traditional system.

B. This is the total of the amount that would be allocated using traditional costing and the amount that would be
allocated using activity-based costing. The total of the two options has no meaning, since one or the other will be used
but not both.

C. This is the total of the three cost rates: $11.50 + $0.14 + $77.00. Since all of those cost rates are for different units,
totalling them is meaningless.

D. The monthly quality control cost assigned to Satin Sheen makeup using activity-based costing will be higher than
the amount using the traditional system, not lower.

Question 172 - CIA 596 III.83 - Service Cost Allocation

A.

(c) HOCK international, page 153


Part 1 : 07/07/14 10:28:01

Under the step method we will first allocate the costs of the information services department to all of the
other departments (including building operations) and then we will allocate the building operations costs
(their own costs plus their share of the information services costs) to the production departments.

The information services costs are $1,200,000 and the usage of their services totaled 2,000 computer hours.
Of the total information services costs, 10% ($120,000) will be allocated to building operations and 30%
($360,000) to the finishing department. This allocation means that there is now $670,000 of costs in the
building operations department that need to be allocated to the production department. The finishing
department occupies 60% of the building space that is occupied by production departments (we do not
include the information services department since no costs will be allocated to it). The share of the costs that
is allocated to the finishing department from building operations is $402,000. This is added to the $360,000
that was allocated to the finishing department from the information services department, giving a total of
$762,000 allocated to the finishing department.

B. This answer allocates some of the building operations costs to the information services department. In the step
method, once a service department's costs have been allocated to other departments, no other service departments'
costs can be allocated to that service department.

C. This answer is a result of two mistakes: (1) allocating the Building Operations costs first instead of allocating the
Information Services' costs first when the problem says to allocate the Building Operations costs first; and (2)
allocating some of the Information Services' costs to Building Operations in the second allocation. In the step method,
once a service department's costs have been allocated to other departments, no other service departments' costs can
be allocated to that service department.

D. This answer uses the direct method to allocate the costs of both departments.

Question 173 - ICMA 10.P1.173 - Variable and Absorption Costing

A. Only variable manufacturing costs are inventoried when variable costing is used. This answer results from including
variable marketing costs, as well. Marketing costs are always period costs that are expensed as they are incurred,
regardless of whether variable costing or absorption costing is being used.

B. Only variable manufacturing costs are inventoried when variable costing is used. This answer results from including
fixed manufacturing costs of $2,000,000 and variable marketing costs of $1,000,000, as well. When variable costing is
used, fixed manufacturing costs and marketing costs are period costs that are expensed as they are incurred.

C. Only variable manufacturing costs are inventoried when variable costing is used, so the inventory value of
the new product is equal to the variable manufacturing costs incurred to produce it, which are $5,000,000.

D. Only variable manufacturing costs are inventoried when variable costing is used. This answer results from including
fixed manufacturing costs of $2,000,000, variable marketing costs of $1,000,000, and fixed marketing costs of
$3,000,000 as well. When variable costing is used, fixed manufacturing costs and marketing costs are period costs
that are expensed as they are incurred.

Question 174 - CMA 695 3.2 - Process Costing

A.

Using the FIFO method, the equivalent units of production for conversion costs are

(c) HOCK international, page 154


Part 1 : 07/07/14 10:28:01

Conversion
Costs
Completion of BWIP 12,800
Started & Completed 76,000
Starting of EWIP 9,600
TOTAL EUP-FIFO 98,400

Under the FIFO method of process costing we need to make three calculations to determine the EUP. These
are: calculate (1) how many EUP were required to finish BWIP, (2) how many units were started and
completed and (3) how many EUP were needed to start the EWIP. There were 16,000 units in BWIP and they
were 20% complete for conversion costs, meaning that they needed to do 80%, or 12,800 EUP to finish BWIP.
There were a total of 76,000 units started and completed during the period (92,000 completed minus the
16,000 in BWIP). There were 24,000 units in EWIP that were 40% complete for conversion costs, meaning that
9,600 EUP of conversion costs had been done on the EWIP. Adding these three numbers together, we get
98,400 units.

B. This answer does not include the work that was done to complete the units in beginning work-in-process inventory.

C. This answer does not include the work that was done to begin the units in ending work-in-process inventory.

D. This answer results from subtracting the number of equivalent units in beginning work-in-process inventory from the
total equivalent units completed from BWIP plus the number of units started and completed plus the number of
equivalent units in ending work-in-process inventory. The number of equivalent units in BWIP are not subtracted to
calculate equivalent units when FIFO is used. They are simply omitted from the calculation of total equivalent units.

Question 175 - CIA 1186 IV.8 - Variable and Absorption Costing

A. Fixed manufacturing overhead is treated as a period cost under variable (or direct) costing. This means
that the costs are expensed when incurred.

B. Fixed manufacturing overhead is not classified as an administrative cost under variable (or direct) costing.

C. Fixed manufacturing overhead is not classified as a selling cost under variable (or direct) costing.

D. Under variable (or direct) costing, fixed manufacturing overhead is not treated as an inventoriable cost.

Question 176 - ICMA 10.P1.221 - Service Cost Allocation

A.

In order to use the step-down method, there must be an order in which we allocate the costs of the service
departments. A popular method is to determine the order according to the percentage of each department’s services
that are provided to other service departments. The department that provides the highest percentage of its services to
other service departments is allocated first. Then the department that provides the next highest percentage of its
services to other service departments comes next, and so on.

The first service department will have its costs allocated to the other service departments and the production
departments. The second service department’s costs (which now include its share of the first service department’s
costs) will be allocated to the other service departments (but not to the first service department that has already been

(c) HOCK international, page 155


Part 1 : 07/07/14 10:28:01

allocated) and the production departments. Once a service department has had its costs allocated, no costs will be
allocated from other service departments to it.

The question tells us that company studies have shown that the Personnel Department provides support to a greater
number of departments than the Information Systems Department does. Therefore, we will assume that the Personnel
Department's costs will be allocated first. Thus, some of the Personnel Department's costs will be allocated to the
Assembly Department.

B.

In order to use the step-down method, there must be an order in which we allocate the costs of the service
departments. A popular method is to determine the order according to the percentage of each department’s services
that are provided to other service departments. The department that provides the highest percentage of its services to
other service departments is allocated first. Then the department that provides the next highest percentage of its
services to other service departments comes next, and so on.

The first service department will have its costs allocated to the other service departments and the production
departments. The second service department’s costs (which now include its share of the first service department’s
costs) will be allocated to the other service departments (but not to the first service department that has already been
allocated) and the production departments. Once a service department has had its costs allocated, no costs will be
allocated from other service departments to it.

The question tells us that company studies have shown that the Personnel Department provides support to a greater
number of departments than the Information Systems Department does. Therefore, we will assume that the Personnel
Department's costs will be allocated first. Thus, some of the Personnel Department's costs will be allocated to the
Assembly Department and the Machining Department.

C.

In order to use the step-down method, there must be an order in which we allocate the costs of the service
departments. A popular method is to determine the order according to the percentage of each department’s services
that are provided to other service departments. The department that provides the highest percentage of its services to
other service departments is allocated first. Then the department that provides the next highest percentage of its
services to other service departments comes next, and so on.

The first service department will have its costs allocated to the other service departments and the production
departments. The second service department’s costs (which now include its share of the first service department’s
costs) will be allocated to the other service departments (but not to the first service department that has already been
allocated) and the production departments. Once a service department has had its costs allocated, no costs will be
allocated from other service departments to it.

The question tells us that company studies have shown that the Personnel Department provides support to a greater
number of departments than the Information Systems Department does. Therefore, we will assume that the Personnel
Department's costs will be allocated first. Therefore, some of the Personnel Department's costs will be allocated to
the Information Systems Department.

D.

In order to use the step-down method, there must be an order in which we allocate the costs of the service
departments. A popular method is to determine the order according to the percentage of each department’s
services that are provided to other service departments. The department that provides the highest percentage
of its services to other service departments is allocated first. Then the department that provides the next
highest percentage of its services to other service departments comes next, and so on.

The first service department will have its costs allocated to the other service departments and the production
departments. The second service department’s costs (which now include its share of the first service
department’s costs) will be allocated to the other service departments (but not to the first service department
that has already been allocated) and the production departments. Once a service department has had its

(c) HOCK international, page 156


Part 1 : 07/07/14 10:28:01

costs allocated, no costs will be allocated from other service departments to it.

The question tells us that company studies have shown that the Personnel Department provides support to a
greater number of departments than the Information Systems Department does. Therefore, we will assume
that the Personnel Department's costs will be allocated first, and some of the Personnel Department's costs
will be allocated to the Information Systems Department. The Information Systems Department's costs,
including its allocation of Personnel Department costs, are then allocated to the production departments, but
not to the Personnel Department.

Thus, under the step-down method, none of the costs of the Information Systems Department would be
allocated to the Personnel Department.

Question 177 - CMA 1290 H5 - Variable and Absorption Costing

A. This is not the correct answer. This answer results from including fixed administrative expenses along with fixed
manufacturing overhead in calculating the fixed cost that would be included in ending inventory under absorption
costing but not included under variable costing. Fixed administrative costs (along with fixed selling costs) are
expensed in both methods, so the fixed administrative costs do not contribute to the difference between operating
income under absorption costing and under variable costing.

B. This is (Fixed Manufacturing Cost Per Unit of $5 × Number of units of change in inventory of 5,000) + Underapplied
Fixed Manufacturing Overhead of $65,000. The underapplied fixed manufacturing overhead is not relevant to the
calculation of the difference in operating income between absorption costing and variable costing, because it is
handled the same way under both methods. The $65,000 underapplied fixed manufacturing cost is charged to
expense under absorption costing because the problem says that underapplied or overapplied manufacturing
overhead is closed out to cost of goods sold. It is also charged to expense under variable costing, because under
variable costing, all fixed costs incurred are expensed as period costs as incurred. Since the amount of underapplied
fixed manufacturing overhead under absorption costing is handled the same way under both methods, it is not
relevant to a calculation of the difference in operating income between them.

C. This is the amount of underapplied fixed manufacturing overhead. It is not the difference in operating income
between the absorption costing and the variable costing methods.

D.

This problem does not specify whether Valyn is using a standard cost system (in which standard, or planned,
costs are used to account for production) or an actual cost system (in which actual costs are used). However,
it does say that Valyn uses a predetermined manufacturing overhead rate for applying manufacturing
overhead to its product. And since the actual, incurred per unit costs for direct materials, direct labor and
variable manufacturing overhead are exactly the same as the planned per unit costs for those items, we do
not need to know whether standard costing or actual costing is being used in order to answer this question.

The difference between the two methods is the treatment of fixed factory overheads. Under the absorption
method fixed manufacturing overheads are applied to the units produced at the predetermined rate, whereas
under the variable method, they are expensed. The fixed factory overhead cost per unit is $5 and we can
determine the difference between these two methods by multiplying this $5 per unit difference by the number
of units that were added to inventory during the period. Inventory increased by 5,000 units and 5,000 × $5
gives us a $25,000 difference in income between the two methods.

The difference in operating income can be calculated in this way because


1. The beginning finished goods inventory is valued at the same per unit manufacturing cost as the
current year's planned per unit manufacturing cost (which implies that the fixed manufacturing cost in
beginning inventory was the same as the current year's predetermined application rate per unit for fixed
manufacturing overhead);

(c) HOCK international, page 157


Part 1 : 07/07/14 10:28:01

2. Over- or under-applied manufacturing overhead is closed to the cost of goods sold account at the end
of the period; and
3. There is no beginning work-in-process inventory.

Question 178 - CIA 596 III.82 - Service Cost Allocation

A. This answer includes the other service department in the calculation of the total square feet. This would be done
under the step method.

B. This answer includes all of the departments (including building operations) in the calculation of total square feet.

C. Under the direct method we ignore services that are provided to the other service departments. In order to
allocate the building operation costs to fabricating, we simply need to determine what percentage of the
building space taken up by production departments is used by fabricating (we ignore the building usage by
the other service departments under direct costing). There is a total of 40,000 square feet and fabricating
occupies 40% of that so they will get 40% of the building costs, or $220,000.

D. This answer ignores the labor and benefit costs, which also need to be allocated.

Question 179 - CIA 1190 IV.6 - Overhead Allocation

A. Materials price variances are usually closed to the cost of goods sold or to the cost of goods sold and
work-in-process on pro-rata basis.

B. Abnormal spoilage is all spoilage in excess of the normal level of spoilage. The costs that have been allocated to
the abnormal spoiled units will be expensed on the income statement in that period as a loss from abnormal spoilage.
Thus, abnormal spoilage is not included in the overhead costs.

C. Prime costs are the costs of direct material and direct labor. These are the direct inputs and are not included in
overhead costs.

D. When overtime must be worked, the premium (this is the amount that the wage increases for overtime
work) that is paid to the workers for the overtime is usually considered to be factory overhead. However, if the
need to work overtime is the result of a specific job or customer request, the premium should be charged to
that specific job and not included in the overall overhead amount to allocate. Hence, in most cases overtime
premiums are included in overhead costs.

Question 180 - ICMA 10.P1.147 - Classifications of Costs

A. Ice cream is a product, and any costs for things used to make it are manufacturing costs. Property taxes on plant
property are a cost that is necessary to provide the physical plant and plant equipment needed to manufacture the ice
cream. Therefore, plant property taxes are manufacturing costs.

B. For an automobile manufacturer, any costs for things used to make automobiles are manufacturing costs. Cars
must have tires. Therefore, tires for an automobile manufacturer are a manufacturing cost.

C. For a car manufacturer, any costs for things used to make cars are manufacturing costs. Sales
commissions are not costs for things used to make cars. Therefore, they are not manufacturing costs.

(c) HOCK international, page 158


Part 1 : 07/07/14 10:28:01

D. Ice cream is a product, and any costs for things used to make it are manufacturing costs. Cream is a manufacturing
cost for an ice cream maker because it is used to make the ice cream.

Question 181 - ICMA 10.P1.164 - Variable and Absorption Costing

A.

This is not the correct answer. Please see the correct answer for an explanation.

We have been unable to determine how to calculate this incorrect answer choice. If you have calculated it, please let
us know how you did it so we can create a full explanation of why this answer choice is incorrect. Please send us an
email at support@hockinternational.com. Include the full Question ID number and the actual incorrect answer choice
-- not its letter, because that can change with every study session created. The Question ID number appears in the
upper right corner of the ExamSuccess screen. Thank you in advance for helping us to make the HOCK study
materials better.

B.

This answer results from calculating cost of goods sold for the operating income statement using the number of units
produced instead of the number of units sold. When absorption costing is used (and absorption costing is required for
external financial reporting under US GAAP), revenue and cost of goods sold are both based on the number of units
sold.

When absorption costing is used (and absorption costing is required for external financial reporting under US GAAP),
revenue and cost of goods sold are both based on the number of units sold.

C.

The problem tells us that the volume variance is written off to cost of goods sold in the year incurred. The
fixed overhead production-volume variance is budgeted fixed overhead minus the amount of applied fixed
overhead. The total budgeted fixed overhead was $15,000 ($20 per unit × 750 units, the denominator level of
activity that was used to calculate the per unit amount).

The amount of fixed overhead applied to production was $20 per unit × 700 units produced, or $14,000. The
amount produced (700 units) was lower than the expected amount of 750 units. Therefore, there is a fixed
overhead production-volume variance of $1,000 ($20 × 50 units). Since we are told there were no spending
variances, we know that actual fixed overhead was the same as budgeted fixed overhead. Therefore, the
production-volume variance and the total fixed overhead variance were the same, and the fixed overhead was
underapplied by $1,000. Since this variance is written off to cost of goods sold, $1,000 will be debited to cost
of goods sold, increasing the cost of goods sold on the operating income statement.

The standard variable manufacturing cost was $90 per unit, and the standard fixed manufacturing cost per
unit was $20, so the total standard cost per unit was $110. Since the standard costs for the year previous to
the last fiscal year were the same as those for the last fiscal year, we know that the cost of the 100 units in
beginning finished goods inventory was also $110 per unit.

Since there were no price, efficiency or spending variances, no further adjustments need to be made to cost
of goods sold other than the $1,000 for the fixed overhead production volume variance.

Therefore, net operating income was:


Sales: 750 units at $200/unit $150,000
Cost of Goods Sold: 750 units at $110/unit (82,500)
Adjustment: Fixed Overhead Volume Variance (1,000)
Gross Profit $ 66,500

(c) HOCK international, page 159


Part 1 : 07/07/14 10:28:01

Selling and Administrative Expense 45,000


Net Operating Income 21,500

D.

This answer results from two errors: (1) calculating cost of goods sold for the operating income statement using the
number of units produced instead of the number of units sold; and (2) omitting the adjustment to cost of goods sold for
the variance that is written off to cost of goods sold.

When absorption costing is used (and absorption costing is required for external financial reporting under US GAAP),
revenue and cost of goods sold are both based on the number of units sold.

The amount of fixed overhead applied to production was $20 per unit × 700 units produced, or $14,000. The amount
produced (700 units) was 50 units lower than the expected amount of 750 units. Therefore, there is a fixed overhead
production-volume variance of $1,000 ($20 × 50 units). Since we are told there were no spending variances, we know
that actual fixed overhead was the same as budgeted fixed overhead. Therefore, the production-volume variance and
the total fixed overhead variance were the same, and the fixed overhead was underapplied by $1,000. Since this
variance is written off to cost of goods sold, $1,000 needs to be debited to cost of goods sold, increasing the cost of
goods sold on the operating income statement.

Question 182 - CIA 591 IV.11 - Overhead Allocation

A. In this question overhead is allocated based on direct labor hours. The budgeted overhead is $500,000 and
the budgeted direct labor hours is 200,000. This gives us an application rate of $2.50 of overhead for every 1
direct labor hour. Since the actual direct labor hours were 210,000, the company applied $525,000 of
overhead. This is $10,000 more than the actual overhead and is the overapplied overhead.

B. This answer reverses the actual and estimated dollar amounts in the calculation.

C. This answer is simply the difference between the actual and estimated overhead costs. Also, in this question
overhead was overapplied, not underapplied.

D. This answer reverses the actual and estimated hours in the calculation.

Question 183 - ICMA 10.P1.215 - Activity-Based and Life-Cycle Costing

A. This is the total engineering cost allocated to Product A divided by the number of units produced of Product B.

B. Number of production orders is the cost driver. Product B used 18 ÷ (12 + 18), or 60% of the total number of
production orders. The total cost was $300,000, so 60% of that is $180,000, and $180,000 is the total
engineering cost allocated to Product B. $180,000 ÷ 12,000 units of Product B produced and sold = $15
engineering cost per unit.

C. This is the engineering cost per unit for Product A, not Product B.

D.

This is not the correct answer. Please see the correct answer for an explanation.

We have been unable to determine how to calculate this incorrect answer choice. If you have calculated it, please let

(c) HOCK international, page 160


Part 1 : 07/07/14 10:28:01

us know how you did it so we can create a full explanation of why this answer choice is incorrect. Please send us an
email at support@hockinternational.com. Include the full Question ID number and the actual incorrect answer choice
-- not its letter, because that can change with every study session created. The Question ID number appears in the
upper right corner of the ExamSuccess screen. Thank you in advance for helping us to make your HOCK study
materials better.

Question 184 - CIA 1195 III.93 - Activity-Based and Life-Cycle Costing

A. This answer assumes that 80 machine hours are required for the entire 20,000 units produced, not per batch.

B. This answer is based on the direct labor overhead rate.

C. This answer uses the machining overhead rate.

D. The traditional system will include the costs for direct materials, direct labor and overhead. Direct materials
and labor are $5.15 per unit so all we need to calculate is the overhead per unit. Under the traditional method,
overhead is applied based on machine hours. The rate is $60 per machine hour ($1,800,000 budgeted costs ÷
30,000 hours). Each unit requires .016 machine hours (80 machine hours in a batch ÷ 5,000 units in a batch).
This means that each unit will have $.96 of overhead applied. This gives a total cost of $6.11 per unit under
the traditional method.

Question 185 - CMA 1290 3.3 - Service Cost Allocation

A. This answer is incorrect because it is not a method of allocating service department costs.

B. Under the reciprocal method the costs of the service departments are allocated to each other, so there is a
recognition of reciprocal interdepartmental service.

C. Under the direct method no costs are allocated between the service departments.

D. Under the step method, the services that one department provides to the other departments are
recognized. However, in the allocation of the costs of the second service department, no costs are allocated
back to the first department. In the allocation of the costs of the third service department, no costs are
allocated back to the first or second service department, and so on.

Question 186 - ICMA 10.P1.150 - Classifications of Costs

A.

To answer this question correctly, look at the total cost for each cost and whether it changes or remains the same
when the activity level changes; then divide the total cost by the activity level for each cost and look at whether that
changes or remains the same as the activity level changes. If the cost remains the same in total when the activity level
changes, it is a fixed cost. If it changes in total but remains the same on a per unit basis, it is a variable cost. If it both
changes in total and on a per unit basis, it is a semi-variable cost.

Cost A is not a semi-variable cost and Cost B is not a variable cost. See correct answer for a full explanation.

B.

(c) HOCK international, page 161


Part 1 : 07/07/14 10:28:01

Cost A is a variable cost. At an activity level of 1,000, the cost per unit is $1.42. At activity levels of 1,500 and
2,000, the cost per unit remains $1.42.

Cost B is a semi-variable cost, a type of mixed cost. It has both a fixed component and a variable component.
There is a basic fixed amount that must be paid regardless of activity, even if there is no activity. And added
to that fixed amount is an amount which varies with activity. We can determine this because the total cost
changes as activity changes, but the per unit cost changes as well. At an activity level of 1,000, the total cost
is $1,550 while the per unit cost is $1.55; at a level of 1,500, total cost changes to $2,200 while the per unit
cost changes to $1.47; and at a level of 2,000, total cost changes to $2,900 while the per unit cost changes to
$1.45.

Cost C is a fixed cost, because it remains the same in total ($1,000).

Cost D is a variable cost. At an activity level of 1,000, the cost per unit is $1.63. At activity levels of 1,500 and
2,000, the cost per unit remains $1.63.

C.

To answer this question correctly, look at the total cost for each cost and whether it changes or remains the same
when the activity level changes; then divide the total cost by the activity level for each cost and look at whether that
changes or remains the same as the activity level changes. If the cost remains the same in total when the activity level
changes, it is a fixed cost. If it changes in total but remains the same on a per unit basis, it is a variable cost. If it both
changes in total and on a per unit basis, it is a semi-variable cost.

Cost B is not a fixed cost. See correct answer for a full explanation.

D.

To answer this question correctly, look at the total cost for each cost and whether it changes or remains the same
when the activity level changes; then divide the total cost by the activity level for each cost and look at whether that
changes or remains the same as the activity level changes. If the cost remains the same in total when the activity level
changes, it is a fixed cost. If it changes in total but remains the same on a per unit basis, it is a variable cost. If it both
changes in total and on a per unit basis, it is a semi-variable cost.

Cost B is not a semi-variable cost. See correct answer for a full explanation.

Question 187 - ICMA 10.P1.202 - Activity-Based and Life-Cycle Costing

A. Activity-based costing is very useful for allocation of costs to service functions.

B. There is no lack of software available to assist with recordkeeping related to activity-based software. A firm
considering implementation of an activity-based costing and management program would have many software options
to choose from.

C. With ABC, cost allocations are based on activities performed and what those activities cost. Some
nonmanufacturing costs that are necessary to produce the products are included with manufacturing costs in
determining the cost of each product. As a result, companies who use ABC normally gain added insights into
the causes of their costs.

D. More cost pools are used with ABC than under more traditional costing methods.

Question 188 - ICMA 10.P1.171 - Variable and Absorption Costing

(c) HOCK international, page 162


Part 1 : 07/07/14 10:28:01

A. Variable selling and administrative costs are not product costs and threfore are not inventoriable. This is true not
only for variable costing but for absorption costing as well.

B. All variable manufacturing costs are product costs that are inventoriable under variable costing, and that includes
more than just raw material and direct labor costs.

C. Fixed manufacturing costs are not inventoriable as product costs under variable costing; all variable manufacturing
costs are inventoriable, and that includes more than just raw material and direct labor costs.

D. Under variable costing, all variable manufacturing costs are product costs and are inventoriable, and fixed
manufacturing costs are classified as period costs.

Question 189 - ICMA 10.P1.209 - Activity-Based and Life-Cycle Costing

A. All manufacturing overhead being fixed would not cause reported net income to be the same for the first year
regardless of which overhead allocation method had been selected. See correct answer for a full explanation.

B. Production costs that are close to those that were budgeted would not cause reported net income to be the same
for the first year regardless of which overhead allocation method had been selected. See correct answer for a full
explanation.

C. A sales mix that does not vary from the mix that was budgeted would not cause reported net income to be the
same for the first year regardless of which overhead allocation method had been selected. See correct answer for a
full explanation.

D.

This is a new company just beginning operations. That fact tells us that the beginning inventory was zero. If
the ending inventory is also zero, as in this answer choice, that tells us that all of the units the company
produced during its first year of operation were sold during that year.

Under all the ways of allocating manufacturing overhead to units produced, there would have been some
variance between the actual incurred overhead and overhead allocated to production as the year passed.
However, at the end of the year, the variances between the actual amount incurred and the amount of costs
applied to the units produced would have been resolved, either by closing them out to COGS only or by
allocating them between COGS and Ending Inventory. If ending inventory was zero, though, all of the
variances would have been closed out only to COGS, regardless of which method of closing out variances
the company was using. Since all production for the year was sold, 100% of actual costs incurred during the
year would have been in COGS, regardless of whether a plantwide rate, a departmental rate, or activity based
costing had been used. The amounts applied under each of the methods would have been different. However,
when the amounts applied and the adjustments made to the COGS account to close out the variances were
combined, the total amount of overhead in COGS expense would have been the same under all the methods.

Sometimes it is easier to see something like this when you use numbers. So let’s say that the actual overhead
incurred by Young Company during the year was $1,000,000. And let’s say that overhead applied under the
various methods would have been as follows:

Plantwide rate, $993,000


Departmental rates, $995,000
Activity Based Costing, $997,000

Using a plantwide rate, with total actual overhead incurred of $1,000,000, $993,000 would have been applied
during production and all of that cost would have flowed to COGS because all of the production was sold.

(c) HOCK international, page 163


Part 1 : 07/07/14 10:28:01

The variance between actual incurred overhead of $1,000,000 and applied overhead of $993,000, or $7,000,
would have been closed out by debiting COGS for $7,000. Total overhead in COGS reported on the income
statement would have been equal to the actual overhead incurred, or $1,000,000.

Using departmental rates, with total actual overhead incurred of $1,000,000, $995,000 would have been
applied during production and all of that cost would have flowed to COGS because all of the production was
sold. The variance between actual incurred overhead of $1,000,000 and applied overhead of $995,000, or
$5,000, would have been closed out by debiting COGS for $5,000. Total overhead in COGS reported on the
income statement would have been equal to the actual overhead incurred, or $1,000,000.

Using Activity Based Costing, with total actual overhead incurred of $1,000,000, $997,000 would have been
applied during production and all of that cost would have flowed to COGS because all of the production was
sold. The variance between actual incurred overhead of $1,000,000 and applied overhead of $997,000, or
$3,000, would have been closed out by debiting COGS for $3,000. Total overhead in COGS reported on the
income statement would have been equal to the actual overhead incurred, or $1,000,000.

We can say this for certain only because all of the production was sold during the year. If some of the units
produced had remained in inventory, unsold at the end of the year, the three different ways of applying the
overhead to production may have resulted in three different divisions between COGS and Ending Inventory of
the $1,000,000 overhead incurred during the year.

Question 190 - ICMA 10.P1.158 - Classifications of Costs

A. This is total manufacturing cost at a production level of 8,000 units. The question asks for the total manufacturing
cost at a level 9,000 units.

B. This is the total cost per unit at an activity level of 8,000 multiplied by 9,000. This does not take into account the
nature of the fixed cost component of the total cost per unit.

C.

The cost function for total costs is FC + (VC per unit × P).

Since the expected fixed cost per unit of $15 was based on a planned activity level of 8,000 units, expected
fixed manufacturing overhead cost must have been $120,000 ($15 × 8,000 units). That amount will not change
with changes in the production level as long as production remains within the relevant range.

Variable costs are $55 per unit ($20 + $25 + $10).

The cost function for total costs is FC + (VC per unit × P). Total manufacturing cost expected to be incurred to
manufacture 9,000 units is $120,000 + ($55 × 9,000) = $615,000).

D.

This is total variable cost per unit multiplied by 8,000 units plus fixed manufacturing cost per unit at the level of 8,000
units multiplied by 9,000 units: ($55 × 8,000) + ($15 × 9,000) = $575,000.

To find the total manufacturing cost expected to be incurred at a production level of 9,000 units, we need to multiply
the total variable cost per unit by the 9,000 units to be produced; then we need to add that amount to total expected
fixed costs to be incurred. Total expected fixed costs to be incurred are not $15 × 9,000, because total fixed costs do
not increase with increases in production as long as production remains within the relevant range.

(c) HOCK international, page 164


Part 1 : 07/07/14 10:28:01

Question 191 - CMA 1295 3.23 - Overhead Allocation

A. This is incorrect because the account balance would be a credit and this would represent an overapplied amount.

B. There would be a credit balance in the account, but this would represent an overapplied amount, not an
underapplied amount.

C. When the actual costs were incurred, the factory overhead account was debited for $132,000. When the
overhead was applied, the account was credited for $140,000 (70% of the $200,000 labor cost). This leaves a
credit balance of $8,000 in the account and this represents an overapplied amount since the applied overhead
was greater than the actual overhead incurred.

D. Overhead was overapplied, but this would be a credit balance in the account.

Question 192 - ICMA 10.P1.238 - Business Process Performance

A. External failure occurs a defect is not detected until the product is already with the consumer. Inspection of
incoming raw materials takes place before production.

B. Appraisal costs are the costs that are incurred in order to determine if an individual unit is defective.
Inspection of incoming raw materials is done in order to determine if any of the raw materials are defective,
so it is an appraisal cost.

C. Internal failure occurs when a problem is detected after production but before shipment to the customer. Inspection
of incoming raw materials takes place before production.

D. Prevention costs are the costs that are incurred in order to prevent a defect from occurring. Evaluating suppliers on
an ongoing basis to make sure their work is of the proper quality is a prevention cost. But checking each unit of
incoming raw materials as it is received is done in order to determine whether an individual unit is defective, which
means the defect already occurred.

Question 193 - ICMA 10.P1.207 - Activity-Based and Life-Cycle Costing

A. Activity-based costing requires the use of a greater number of allocation bases; and activity-based costing
also results in more accurate costing results. Activity-based costing focuses on individual activities as the
fundamental cost objects. In ABC, the premise is that the cost of a finished product should include the cost of
direct materials, direct labor and overhead costs directly attributable to that product, and it should also
include a portion of the administrative costs necessary to produce the product. With ABC, the cost
allocations are not based on usage of resources. Instead, they are based on activities performed and what
those activities cost. ABC is much more detailed than traditional costing, because it uses many more cost
pools, and each cost pool has its own cost driver. But it results in a more accurate cost for products.

B. Departmental allocation does not result in a greater number of allocation bases; and departmental allocation does
not result in more accurate costing results. Activity-based costing focuses on individual activities as the fundamental
cost objects. In ABC, the premise is that the cost of a finished product should include the cost of direct materials,
direct labor and overhead costs directly attributable to that product, and it should also include a portion of the
administrative costs necessary to produce the product. With ABC, the cost allocations are not based on usage of
resources. Instead, they are based on activities performed and what those activities cost. ABC is much more detailed
than traditional costing, because it uses many more cost pools, and each cost pool has its own cost driver. But it
results in a more accurate cost for products.

(c) HOCK international, page 165


Part 1 : 07/07/14 10:28:01

C. Departmental allocation does not result in more accurate costing results than ABC does. Activity-based costing
focuses on individual activities as the fundamental cost objects. In ABC, the premise is that the cost of a finished
product should include the cost of direct materials, direct labor and overhead costs directly attributable to that product,
and it should also include a portion of the administrative costs necessary to produce the product. With ABC, the cost
allocations are not based on usage of resources. Instead, they are based on activities performed and what those
activities cost. ABC is much more detailed than traditional costing, because it uses many more cost pools, and each
cost pool has its own cost driver. But it results in a more accurate cost for products.

D. Departmental allocation does not result in a greater number of allocation bases. Activity-based costing focuses on
individual activities as the fundamental cost objects. In ABC, the premise is that the cost of a finished product should
include the cost of direct materials, direct labor and overhead costs directly attributable to that product, and it should
also include a portion of the administrative costs necessary to produce the product. With ABC, the cost allocations are
not based on usage of resources. Instead, they are based on activities performed and what those activities cost. ABC
is much more detailed than traditional costing, because it uses many more cost pools, and each cost pool has its own
cost driver. But it results in a more accurate cost for products.

Question 194 - CMA 689 4.13 - Classifications of Costs

A. Prime costs are the costs of direct material and direct labor.

B. Sunk costs are costs that have already been spent and cannot be changed by current or future decisions. Not all of
the fixed overhead costs are sunk costs, though it is possible that some of them are.

C. Because the costs for fixed overhead relate to items like rent, these are committed costs. A committed cost
is a cost for the company's infrastructure. Committed costs are costs that are required in order to establish
and maintain the readiness to do business. Examples are fixed assets such as property, plant and equipment
and intangible assets such as the purchase of a franchise. They are usually on the balance sheet as assets
and become expenses in the form of amortization and depreciation.

D. Discretionary costs are costs that do not need to be incurred in the short-term. Since these are production costs,
they do need to be incurred and are not discretionary costs.

Question 195 - ICMA 10.P1.162 - Variable and Absorption Costing

A.

This answer results from multiplying the standard variable manufacturing cost per unit and the standard fixed
manufacturing cost per unit by the number of units produced, not by the number of units sold.

Income statements are based on the number of units sold, not on the number of units produced. The company
produced 700 units and sold 750 units, so they sold 50 units from their beginning inventory, and the cost of those units
needs to be included in the cost of goods sold. Beginning inventory is costed at the previous year's costs; and since
the previous year's variable costs per unit were the same as the current year's budgeted and actual costs, we can use
the current year's variable manufacturing costs for all 750 units sold.

The problem says there were no spending variances, and therefore we know that the fixed manufacturing cost
incurred was the same as the fixed manufacturing cost budgeted. Therefore, we can use the standard fixed
manufacturing cost per unit and the denominator level of activity to calculate the total fixed manufacturing overhead
that was budgeted, and we will also have the actual fixed manufacturing overhead that was incurred. The full amount
of the actual fixed manufacturing cost is expensed.

B.

(c) HOCK international, page 166


Part 1 : 07/07/14 10:28:01

Since there were no price, efficiency or spending variances, that means actual costs per unit were the same
as the standard costs per unit for the year. Thus, we can use the standard costs per unit given as actual
amounts for purposes of calculating operating income under variable costing.

Income statements are based on the number of units sold, not on the number of units produced. The
company produced 700 units and sold 750 units, so they sold 50 units from their beginning inventory, and the
cost of those units needs to be included in the cost of goods sold. Beginning inventory is costed at the
previous year's costs; and since the previous year's variable costs per unit were the same as the current
year's budgeted and actual costs, we can use the current year's variable manufacturing costs for all 750 units
sold.

Under variable costing, only variable manufacturing costs are inventoried, while fixed manufacturing costs
are expensed as incurred.

The problem says there were no spending variances, and therefore we know that the fixed manufacturing
cost incurred was the same as the fixed manufacturing cost budgeted. Therefore, we can use the standard
fixed manufacturing cost per unit and the denominator level of activity to calculate the total fixed
manufacturing overhead that was budgeted, and we will also have the actual fixed manufacturing overhead
that was incurred. We know that the standard fixed manufacturing cost per unit ($20) was calculated by
dividing the total budgeted fixed manufacturing cost by the anticipated production of 750 units. Therefore, we
can multiply the $20 per unit standard cost by the anticipated production of 750 units, and we will have the
budgeted total fixed manufacturing cost. As we said, since there were no spending variances for the year, the
actual fixed manufacturing cost is the same as the budgeted fixed manufacturing cost, and the full amount of
the actual fixed manufacturing cost, or $15,000, is expensed.

Actual selling and administrative cost is the same as the budgeted selling and administrative cost, so that is
used as given.

And since there were no production variances, no variances needed to be written off to cost of goods sold, so
that is an adjustment that does not need to be made.

The income statement using variable costing was


Sales revenue: 750 units @ $200 $150,000
Variable cost: 750 units @ $90 67,500
Contribution margin $ 82,500

Fixed mfg. cost: $20 standard cost × 750 (denomin. level of activity)$ 15,000
Selling and administrative cost 45,000
Net operating income $ 22,500

C.

This is not the correct answer. Please see the correct answer for an explanation.

We have been unable to determine how to calculate this incorrect answer choice. If you have calculated it, please let
us know how you did it so we can create a full explanation of why this answer choice is incorrect. Please send us an
email at support@hockinternational.com. Include the full Question ID number and the actual incorrect answer choice
-- not its letter, because that can change with every study session created. The Question ID number appears in the
upper right corner of the ExamSuccess screen. Thank you in advance for helping us to make the HOCK study
materials better.

D. This is the operating income using absorption costing.

(c) HOCK international, page 167


Part 1 : 07/07/14 10:28:01

Question 196 - CIA 1195 III.94 - Activity-Based and Life-Cycle Costing

A. This answer assumes that that there is only one setup for each batch and the machine hours were therefore only
80.

B. This answer assumes the incorrect number of machine hours.

C. This answer uses only one setup per batch instead of two.

D.

Under the ABC method we will have $5.15 in direct materials and direct labor, which is the same as under the
traditional costing method; but we will need to calculate the overhead on the basis of activity-based costing.

In ABC, there are 4 calculations we will need to make as part of the overhead allocation. These are as follows
per activity:

1. Material Handling – $.12 per part ($720,000 ÷ 6,000,000) and there are 5 parts per unit. This is $.60.

2. Setup Costs – $420 per setup ($315,000 ÷ 750). There are 2 setups per batch, for a cost of $840 for each
batch of 5,000 units. This is $.168 per unit ($840 ÷ 5,000).

3. Machining Costs – $18 per machine hour. There are 80 machine hours per batch, giving us $1,440 per batch
of 5,000 units, or $.288 per unit.

4. Quality Control – $450 per batch. This is $.09 per unit ($450 ÷ 5,000).

In total, these costs add up to $6.30 per unit.

Question 197 - CIA 585 IV.5 - Variable and Absorption Costing

A. Gross margin is calculated as sales minus cost of goods sold. All prices and costs have remained the same for the
last 2 years and are expected to do so in Year 3. Sales in Year 3 are lower than in Year 2. Since prices and costs
have remained the same and sales have equaled production each year, the gross margin for Year 3 cannot be equal
to the gross margin for Year 2.

B. Gross margin is calculated as sales minus cost of goods sold. All prices and costs have remained the same for the
last 2 years and are expected to do so in Year 3. Sales in Year 3 are lower than sales in Year 2. Since prices and
costs have remained the same and sales have equaled production each year, the gross margin for Year 3 cannot be
greater than the gross margin for Year 2.

C.

Gross margin is calculated as sales minus cost of goods sold. Since all of the costs have remained the same
over the period and there has been no change in inventory for any period (since sales have been equal to
production each year), the gross margin for Year 3 will be equal to the gross margin from the year in which
sales were the same level, and this is Year 1.

Because this is a new company and for every year since its beginning, sales have been equal to production,
inventory at year end for each year has been zero. Because of this, we do not need to know whether the
company closes out under- and over-applied overhead to cost of goods sold only, or whether the company
prorates it between cost of goods sold and inventory. Since inventory is zero, all under- or over-applied
overhead will have been closed to cost of goods sold only. And for each year since its beginning, the
company has had under-applied fixed overhead, because actual production has been lower than planned

(c) HOCK international, page 168


Part 1 : 07/07/14 10:28:01

production. Unless fixed overhead has been very different in Year 3 than it was in Year 1, the gross margin for
the two years should be substantially the same.

D. Gross margin is calculated as sales minus cost of goods sold. All prices and costs have remained the same for the
last 2 years and are expected to do so in Year 3. Sales in Year 3 are expected to be the same as sales in Year 1.
Since prices and costs have remained the same and sales have equaled production each year, gross margin for Year
3 cannot be greater than the gross margin for Year 1.

Question 198 - CMA 1285 4.14 H2 - Variable and Absorption Costing

A. This answer results from not deducting the selling and administrative costs.

B. Under variable costing, the fixed manufacturing costs are expensed. With a selling price of $40 per unit and
a variable cost of $30 per unit, the manufacturing contribution margin is $10 per unit. With sales of 120,000
units, this is in total $1,200,000. Subtracting from this the fixed manufacturing costs of $600,000 and the
selling and administrative costs of $400, we get an operating income of $200,000.

C. This is the contribution margin minus the selling and administrative costs, but nothing is deducted for fixed
manufacturing costs.

D. This is the income under the absorption method.

Question 199 - CMA 695 3.7 - Process Costing

A. This is the equivalent unit conversion cost using the FIFO method.

B. This answer does not include the conversion costs that were in beginning work-in-process inventory.

C. This is the total conversion costs in both beginning work-in-process inventory and those incurred during the period,
divided by the number of equivalent units for conversion costs under FIFO. The question asks for equivalent unit
conversion cost using the weighted average method, so the total costs need to be divided by the number of equivalent
units calculated using the weighted average method.

D.

The cost per equivalent unit using the weighted average method is the total costs in both beginning WIP
inventory and costs added during the period divided by the total EUP during the period.

Under the weighted average method, to calculate the equivalent units we need to add together the number of
units completed and the number of EUP that were done to start the EWIP. In this question, 92,000 units were
completed and the 24,000 units in EWIP were 40% complete for conversion costs, giving us 9,600 EUP in
EWIP. This gives a total of 101,600 EUP of conversion costs for the period.
Conversion
Costs
Units Completed 92,000
Starting of EWIP 9,600
TOTAL EUP-WAVG 101,600

Here are the total conversion costs:


Beginning WIP costs $20,320 DL + $15,240 OH $ 35,560
Costs added $182,880 DL + $391,160 OH 574,040

(c) HOCK international, page 169


Part 1 : 07/07/14 10:28:01

Total costs $609,600

The total conversion cost is $609,600 and this is divided by the 101,600 EUP, giving us a cost per equivalent
unit for conversion costs of $6.00.

Question 200 - CIA 587 IV.6 - Joint Products and Byproducts

A. This is the amount that would be allocated if the net revenue from the by-product did not reduce the joint costs to
be allocated.

B. In this question, we can very easily see that the two-by-fours represent 2/3 of the total board feet that is
produced and as such will be allocated 2/3 of the joint costs. What makes this question a bit more difficult is
the need to reduce the joint costs by the net revenues from the sale of the sawdust (the by-product). There
are 1,000 pounds of sawdust produced and this will be sold for $2.00 a pound. However, there is a $.10
packaging charge per pound of sawdust and a 10% commission, which is $.20. So, for each pound of
sawdust, the company will receive $1.70. In total, this is $1,700. This will reduce the joint costs to $48,300, and
2/3 of this is $32,200.

C. This answer does not include the $.10 per pound packaging costs as a reduction of the net revenue from the
by-product.

D. This is the answer if the joint costs are reduced by the entire $2,000 selling price of the sawdust. The joint costs are
reduced only by the amount of the net revenue of the by-product.

(c) HOCK international, page 170

You might also like